Você está na página 1de 189

G.R. No. 81561 January 18, 1991 PEOPLE OF THE PHILIPPINES, plaintiff-appellee vs. ANDRE MARTI, accused-appellant.

The Solicitor General for plaintiff-appellee. Reynaldo B. Tatoy and Abelardo E. Rogacion for accused-appellant.

BIDIN, J.:p This is an appeal from a decision * rendered by the Special Criminal Court of Manila (Regional Trial Court, Branch XLIX) convicting accused-appellant of violation of Section 21 (b), Article IV in relation to Section 4, Article 11 and Section 2 (e) (i), Article 1 of Republic Act 6425, as amended, otherwise known as the Dangerous Drugs Act. The facts as summarized in the brief of the prosecution are as follows: On August 14, 1987, between 10:00 and 11:00 a.m., the appellant and his common-law wife, Shirley Reyes, went to the booth of the "Manila Packing and Export Forwarders" in the Pistang Pilipino Complex, Ermita, Manila, carrying with them four (4) gift wrapped packages. Anita Reyes (the proprietress and no relation to Shirley Reyes) attended to them. The appellant informed Anita Reyes that he was sending the packages to a friend in Zurich, Switzerland. Appellant filled up the contract necessary for the transaction, writing therein his name, passport number, the date of shipment and the name and address of the consignee, namely, "WALTER FIERZ, Mattacketr II, 8052 Zurich, Switzerland" (Decision, p. 6) Anita Reyes then asked the appellant if she could examine and inspect the packages. Appellant, however, refused, assuring her that the packages simply contained books, cigars, and gloves and were gifts to his friend in Zurich. In view of appellant's representation, Anita Reyes no longer insisted on inspecting the packages. The four (4) packages were then placed inside a brown corrugated box one by two feet in size (1' x 2'). Styro-foam was placed at the bottom and on top of the packages before the box was sealed with masking tape, thus making the box ready for shipment (Decision, p. 8). Before delivery of appellant's box to the Bureau of Customs and/or Bureau of Posts, Mr. Job Reyes (proprietor) and husband of Anita (Reyes), following standard operating procedure, opened the boxes for final inspection. When he opened appellant's box, a peculiar odor emitted therefrom. His curiousity aroused, he squeezed one of the bundles allegedly containing gloves and felt

dried leaves inside. Opening one of the bundles, he pulled out a cellophane wrapper protruding from the opening of one of the gloves. He made an opening on one of the cellophane wrappers and took several grams of the contents thereof (tsn, pp. 29-30, October 6, 1987; Emphasis supplied). Job Reyes forthwith prepared a letter reporting the shipment to the NBI and requesting a laboratory examination of the samples he extracted from the cellophane wrapper (tsn, pp. 5-6, October 6, 1987). He brought the letter and a sample of appellant's shipment to the Narcotics Section of the National Bureau of Investigation (NBI), at about 1:30 o'clock in the afternoon of that date, i.e., August 14, 1987. He was interviewed by the Chief of Narcotics Section. Job Reyes informed the NBI that the rest of the shipment was still in his office. Therefore, Job Reyes and three (3) NBI agents, and a photographer, went to the Reyes' office at Ermita, Manila (tsn, p. 30, October 6, 1987). Job Reyes brought out the box in which appellant's packages were placed and, in the presence of the NBI agents, opened the top flaps, removed the styro-foam and took out the cellophane wrappers from inside the gloves. Dried marijuana leaves were found to have been contained inside the cellophane wrappers (tsn, p. 38, October 6, 1987; Emphasis supplied). The package which allegedly contained books was likewise opened by Job Reyes. He discovered that the package contained bricks or cake-like dried marijuana leaves. The package which allegedly contained tabacalera cigars was also opened. It turned out that dried marijuana leaves were neatly stocked underneath the cigars (tsn, p. 39, October 6, 1987). The NBI agents made an inventory and took charge of the box and of the contents thereof, after signing a "Receipt" acknowledging custody of the said effects (tsn, pp. 2-3, October 7, 1987). Thereupon, the NBI agents tried to locate appellant but to no avail. Appellant's stated address in his passport being the Manila Central Post Office, the agents requested assistance from the latter's Chief Security. On August 27, 1987, appellant, while claiming his mail at the Central Post Office, was invited by the NBI to shed light on the attempted shipment of the seized dried leaves. On the same day the Narcotics Section of the NBI submitted the dried leaves to the Forensic Chemistry Section for laboratory examination. It turned out that the dried leaves were marijuana flowering tops as certified by the forensic chemist. (Appellee's Brief, pp. 9-11, Rollo, pp. 132-134). Thereafter, an Information was filed against appellant for violation of RA 6425, otherwise known as the Dangerous Drugs Act.

After trial, the court a quo rendered the assailed decision. In this appeal, accused/appellant assigns the following errors, to wit: THE LOWER COURT ERRED IN ADMITTING IN EVIDENCE THE ILLEGALLY SEARCHED AND SEIZED OBJECTS CONTAINED IN THE FOUR PARCELS. THE LOWER COURT ERRED IN CONVICTING APPELLANT DESPITE THE UNDISPUTED FACT THAT HIS RIGHTS UNDER THE CONSTITUTION WHILE UNDER CUSTODIAL PROCEEDINGS WERE NOT OBSERVED. THE LOWER COURT ERRED IN NOT GIVING CREDENCE TO THE EXPLANATION OF THE APPELLANT ON HOW THE FOUR PARCELS CAME INTO HIS POSSESSION (Appellant's Brief, p. 1; Rollo, p. 55) 1. Appellant contends that the evidence subject of the imputed offense had been obtained in violation of his constitutional rights against unreasonable search and seizure and privacy of communication (Sec. 2 and 3, Art. III, Constitution) and therefore argues that the same should be held inadmissible in evidence (Sec. 3 (2), Art. III). Sections 2 and 3, Article III of the Constitution provide: Sec. 2. The right of the people to be secure in their persons, houses, papers and effects against unreasonable searches and seizures of whatever nature and for any purpose shall be inviolable, and no search warrant or warrant of arrest shall issue except upon probable cause to be determined personally by the judge after examination under oath or affirmation of the complainant and the witnesses he may produce, and particularly describing the place to be searched and the persons or things to be seized. Sec. 3. (1) The privacy of communication and correspondence shall be inviolable except upon lawful order of the court, or when public safety or order requires otherwise as prescribed by law. (2) Any evidence obtained in violation of this or the preceding section shall be inadmissible for any purpose in any proceeding. Our present constitutional provision on the guarantee against unreasonable search and seizure had its origin in the 1935 Charter which, worded as follows: The right of the people to be secure in their persons, houses, papers and effects against unreasonable searches and seizures shall not be violated, and no warrants shall issue but upon probable cause, to be determined by the judge after examination under oath or affirmation of the complainant and the witnesses he may produce, and particularly describing the place to be searched, and the

persons or things to be seized. (Sec. 1 [3], Article III) was in turn derived almost verbatim from the Fourth Amendment ** to the United States Constitution. As such, the Court may turn to the pronouncements of the United States Federal Supreme Court and State Appellate Courts which are considered doctrinal in this jurisdiction. Thus, following the exclusionary rule laid down in Mapp v. Ohio by the US Federal Supreme Court (367 US 643, 81 S.Ct. 1684, 6 L.Ed. 1081 [1961]), this Court, in Stonehill v. Diokno (20 SCRA 383 [1967]), declared as inadmissible any evidence obtained by virtue of a defective search and seizure warrant, abandoning in the process the ruling earlier adopted in Moncado v. People's Court (80 Phil. 1 [1948]) wherein the admissibility of evidence was not affected by the illegality of its seizure. The 1973 Charter (Sec. 4 [2], Art. IV) constitutionalized the Stonehill ruling and is carried over up to the present with the advent of the 1987 Constitution. In a number of cases, the Court strictly adhered to the exclusionary rule and has struck down the admissibility of evidence obtained in violation of the constitutional safeguard against unreasonable searches and seizures. (Bache & Co., (Phil.), Inc., v. Ruiz, 37 SCRA 823 [1971]; Lim v. Ponce de Leon, 66 SCRA 299 [1975]; People v. Burgos, 144 SCRA 1 [1986]; Roan v. Gonzales, 145 SCRA 687 [1987]; See also Salazar v. Hon. Achacoso, et al., GR No. 81510, March 14, 1990). It must be noted, however, that in all those cases adverted to, the evidence so obtained were invariably procured by the State acting through the medium of its law enforcers or other authorized government agencies. On the other hand, the case at bar assumes a peculiar character since the evidence sought to be excluded was primarily discovered and obtained by a private person, acting in a private capacity and without the intervention and participation of State authorities. Under the circumstances, can accused/appellant validly claim that his constitutional right against unreasonable searches and seizure has been violated? Stated otherwise, may an act of a private individual, allegedly in violation of appellant's constitutional rights, be invoked against the State? We hold in the negative. In the absence of governmental interference, the liberties guaranteed by the Constitution cannot be invoked against the State. As this Court held in Villanueva v. Querubin (48 SCRA 345 [1972]: 1. This constitutional right (against unreasonable search and seizure) refers to the immunity of one's person, whether citizen or alien, from

interference by government, included in which is his residence, his papers, and other possessions. . . . . . . There the state, however powerful, does not as such have the access except under the circumstances above noted, for in the traditional formulation, his house, however humble, is his castle. Thus is outlawed any unwarranted intrusion by government, which is called upon to refrain from any invasion of his dwelling and to respect the privacies of his life. . . . (Cf. Schermerber v. California, 384 US 757 [1966] and Boyd v. United States, 116 US 616 [1886]; Emphasis supplied). In Burdeau v. McDowell (256 US 465 (1921), 41 S Ct. 547; 65 L.Ed. 1048), the Court there in construing the right against unreasonable searches and seizures declared that: (t)he Fourth Amendment gives protection against unlawful searches and seizures, and as shown in previous cases, its protection applies to governmental action. Its origin and history clearly show that it was intended as a restraint upon the activities of sovereign authority, and was not intended to be a limitation upon other than governmental agencies; as against such authority it was the purpose of the Fourth Amendment to secure the citizen in the right of unmolested occupation of his dwelling and the possession of his property, subject to the right of seizure by process duly served. The above ruling was reiterated in State v. Bryan (457 P.2d 661 [1968]) where a parking attendant who searched the automobile to ascertain the owner thereof found marijuana instead, without the knowledge and participation of police authorities, was declared admissible in prosecution for illegal possession of narcotics. And again in the 1969 case of Walker v. State (429 S.W.2d 121), it was held that the search and seizure clauses are restraints upon the government and its agents, not upon private individuals (citing People v. Potter, 240 Cal. App.2d 621, 49 Cap. Rptr, 892 (1966); State v. Brown, Mo., 391 S.W.2d 903 (1965); State v. Olsen, Or., 317 P.2d 938 (1957). Likewise appropos is the case of Bernas v. US (373 F.2d 517 (1967). The Court there said: The search of which appellant complains, however, was made by a private citizen the owner of a motel in which appellant stayed overnight and in which he left behind a travel case containing the evidence*** complained of. The search was made on the motel owner's own initiative. Because of it, he became suspicious, called the local police, informed them of the bag's contents, and made it available to the authorities.

The fourth amendment and the case law applying it do not require exclusion of evidence obtained through a search by a private citizen. Rather, the amendment only proscribes governmental action." The contraband in the case at bar having come into possession of the Government without the latter transgressing appellant's rights against unreasonable search and seizure, the Court sees no cogent reason why the same should not be admitted against him in the prosecution of the offense charged. Appellant, however, would like this court to believe that NBI agents made an illegal search and seizure of the evidence later on used in prosecuting the case which resulted in his conviction. The postulate advanced by accused/appellant needs to be clarified in two days. In both instances, the argument stands to fall on its own weight, or the lack of it. First, the factual considerations of the case at bar readily foreclose the proposition that NBI agents conducted an illegal search and seizure of the prohibited merchandise. Records of the case clearly indicate that it was Mr. Job Reyes, the proprietor of the forwarding agency, who made search/inspection of the packages. Said inspection was reasonable and a standard operating procedure on the part of Mr. Reyes as a precautionary measure before delivery of packages to the Bureau of Customs or the Bureau of Posts (TSN, October 6 & 7, 1987, pp. 15-18; pp. 7-8; Original Records, pp. 119-122; 167-168). It will be recalled that after Reyes opened the box containing the illicit cargo, he took samples of the same to the NBI and later summoned the agents to his place of business. Thereafter, he opened the parcel containing the rest of the shipment and entrusted the care and custody thereof to the NBI agents. Clearly, the NBI agents made no search and seizure, much less an illegal one, contrary to the postulate of accused/appellant. Second, the mere presence of the NBI agents did not convert the reasonable search effected by Reyes into a warrantless search and seizure proscribed by the Constitution. Merely to observe and look at that which is in plain sight is not a search. Having observed that which is open, where no trespass has been committed in aid thereof, is not search (Chadwick v. State, 429 SW2d 135). Where the contraband articles are identified without a trespass on the part of the arresting officer, there is not the search that is prohibited by the constitution (US v. Lee 274 US 559, 71 L.Ed. 1202 [1927]; Ker v. State of California 374 US 23, 10 L.Ed.2d. 726 [1963]; Moore v. State, 429 SW2d 122 [1968]). In Gandy v. Watkins (237 F. Supp. 266 [1964]), it was likewise held that where the property was taken into custody of the police at the specific request of the manager and where the search was initially made by the owner there is no

unreasonable search and seizure within the constitutional meaning of the term. That the Bill of Rights embodied in the Constitution is not meant to be invoked against acts of private individuals finds support in the deliberations of the Constitutional Commission. True, the liberties guaranteed by the fundamental law of the land must always be subject to protection. But protection against whom? Commissioner Bernas in his sponsorship speech in the Bill of Rights answers the query which he himself posed, as follows: First, the general reflections. The protection of fundamental liberties in the essence of constitutional democracy. Protection against whom? Protection against the state. The Bill of Rights governs the relationship between the individual and the state. Its concern is not the relation between individuals, between a private individual and other individuals. What the Bill of Rights does is to declare some forbidden zones in the private sphere inaccessible to any power holder. (Sponsorship Speech of Commissioner Bernas , Record of the Constitutional Commission, Vol. 1, p. 674; July 17, 1986; Emphasis supplied) The constitutional proscription against unlawful searches and seizures therefore applies as a restraint directed only against the government and its agencies tasked with the enforcement of the law. Thus, it could only be invoked against the State to whom the restraint against arbitrary and unreasonable exercise of power is imposed. If the search is made upon the request of law enforcers, a warrant must generally be first secured if it is to pass the test of constitutionality. However, if the search is made at the behest or initiative of the proprietor of a private establishment for its own and private purposes, as in the case at bar, and without the intervention of police authorities, the right against unreasonable search and seizure cannot be invoked for only the act of private individual, not the law enforcers, is involved. In sum, the protection against unreasonable searches and seizures cannot be extended to acts committed by private individuals so as to bring it within the ambit of alleged unlawful intrusion by the government. Appellant argues, however, that since the provisions of the 1935 Constitution has been modified by the present phraseology found in the 1987 Charter, expressly declaring as inadmissible any evidence obtained in violation of the constitutional prohibition against illegal search and seizure, it matters not whether the evidence was procured by police authorities or private individuals (Appellant's Brief, p. 8, Rollo, p. 62). The argument is untenable. For one thing, the constitution, in laying down the principles of the government and fundamental liberties of the people, does not govern relationships between individuals. Moreover, it must be emphasized that the modifications introduced in the 1987 Constitution (re: Sec. 2, Art. III) relate to

the issuance of either a search warrant or warrant of arrest vis-a-vis the responsibility of the judge in the issuance thereof (See Soliven v. Makasiar, 167 SCRA 393 [1988]; Circular No. 13 [October 1, 1985] and Circular No. 12 [June 30, 1987]. The modifications introduced deviate in no manner as to whom the restriction or inhibition against unreasonable search and seizure is directed against. The restraint stayed with the State and did not shift to anyone else. Corolarilly, alleged violations against unreasonable search and seizure may only be invoked against the State by an individual unjustly traduced by the exercise of sovereign authority. To agree with appellant that an act of a private individual in violation of the Bill of Rights should also be construed as an act of the State would result in serious legal complications and an absurd interpretation of the constitution. Similarly, the admissibility of the evidence procured by an individual effected through private seizure equally applies, in pari passu, to the alleged violation, non-governmental as it is, of appellant's constitutional rights to privacy and communication. 2. In his second assignment of error, appellant contends that the lower court erred in convicting him despite the undisputed fact that his rights under the constitution while under custodial investigation were not observed. Again, the contention is without merit, We have carefully examined the records of the case and found nothing to indicate, as an "undisputed fact", that appellant was not informed of his constitutional rights or that he gave statements without the assistance of counsel. The law enforcers testified that accused/appellant was informed of his constitutional rights. It is presumed that they have regularly performed their duties (See. 5(m), Rule 131) and their testimonies should be given full faith and credence, there being no evidence to the contrary. What is clear from the records, on the other hand, is that appellant refused to give any written statement while under investigation as testified by Atty. Lastimoso of the NBI, Thus: Fiscal Formoso: You said that you investigated Mr. and Mrs. Job Reyes. What about the accused here, did you investigate the accused together with the girl? WITNESS: Yes, we have interviewed the accused together with the girl but the accused availed of his constitutional right not to give any written statement, sir. (TSN, October 8, 1987, p. 62; Original Records, p. 240) The above testimony of the witness for the prosecution was not contradicted by

the defense on cross-examination. As borne out by the records, neither was there any proof by the defense that appellant gave uncounselled confession while being investigated. What is more, we have examined the assailed judgment of the trial court and nowhere is there any reference made to the testimony of appellant while under custodial investigation which was utilized in the finding of conviction. Appellant's second assignment of error is therefore misplaced. 3. Coming now to appellant's third assignment of error, appellant would like us to believe that he was not the owner of the packages which contained prohibited drugs but rather a certain Michael, a German national, whom appellant met in a pub along Ermita, Manila: that in the course of their 30-minute conversation, Michael requested him to ship the packages and gave him P2,000.00 for the cost of the shipment since the German national was about to leave the country the next day (October 15, 1987, TSN, pp. 2-10). Rather than give the appearance of veracity, we find appellant's disclaimer as incredulous, self-serving and contrary to human experience. It can easily be fabricated. An acquaintance with a complete stranger struck in half an hour could not have pushed a man to entrust the shipment of four (4) parcels and shell out P2,000.00 for the purpose and for appellant to readily accede to comply with the undertaking without first ascertaining its contents. As stated by the trial court, "(a) person would not simply entrust contraband and of considerable value at that as the marijuana flowering tops, and the cash amount of P2,000.00 to a complete stranger like the Accused. The Accused, on the other hand, would not simply accept such undertaking to take custody of the packages and ship the same from a complete stranger on his mere say-so" (Decision, p. 19, Rollo, p. 91). As to why he readily agreed to do the errand, appellant failed to explain. Denials, if unsubstantiated by clear and convincing evidence, are negative self-serving evidence which deserve no weight in law and cannot be given greater evidentiary weight than the testimony of credible witnesses who testify on affirmative matters (People v. Esquillo, 171 SCRA 571 [1989]; People vs. Sariol, 174 SCRA 237 [1989]). Appellant's bare denial is even made more suspect considering that, as per records of the Interpol, he was previously convicted of possession of hashish by the Kleve Court in the Federal Republic of Germany on January 1, 1982 and that the consignee of the frustrated shipment, Walter Fierz, also a Swiss national, was likewise convicted for drug abuse and is just about an hour's drive from appellant's residence in Zurich, Switzerland (TSN, October 8, 1987, p. 66; Original Records, p. 244; Decision, p. 21; Rollo, p. 93). Evidence to be believed, must not only proceed from the mouth of a credible witness, but it must be credible in itself such as the common experience and observation of mankind can approve as probable under the circumstances (People v. Alto, 26 SCRA 342 [1968], citing Daggers v. Van Dyke, 37 N.J. Eg. 130; see also People v. Sarda, 172 SCRA 651 [1989]; People v. Sunga, 123

SCRA 327 [1983]); Castaares v. CA, 92 SCRA 567 [1979]). As records further show, appellant did not even bother to ask Michael's full name, his complete address or passport number. Furthermore, if indeed, the German national was the owner of the merchandise, appellant should have so indicated in the contract of shipment (Exh. "B", Original Records, p. 40). On the contrary, appellant signed the contract as the owner and shipper thereof giving more weight to the presumption that things which a person possesses, or exercises acts of ownership over, are owned by him (Sec. 5 [j], Rule 131). At this point, appellant is therefore estopped to claim otherwise. Premises considered, we see no error committed by the trial court in rendering the assailed judgment. WHEREFORE, the judgment of conviction finding appellant guilty beyond reasonable doubt of the crime charged is hereby AFFIRMED. No costs. SO ORDERED. Fernan, C.J., Gutierrez, Jr. and Feliciano, JJ., concur.

Footnotes * Penned by Judge Romeo J. Callejo. ** It reads: "The right of the people to be secure in their persons, houses, papers and effects, against unreasonable searches and seizures, shall not be violated, and no warrants shall issue, but upon probable cause, supported by oath or affirmation, and particularly describing the place to be searched, and the persons or things to be seized." *** Forged checks.

G.R. No. L-19550

June 19, 1967

HARRY S. STONEHILL, ROBERT P. BROOKS, JOHN J. BROOKS and KARL BECK, petitioners, vs. HON. JOSE W. DIOKNO, in his capacity as SECRETARY OF JUSTICE; JOSE LUKBAN, in his capacity as Acting Director, National Bureau of Investigation; SPECIAL PROSECUTORS PEDRO D. CENZON, EFREN I. PLANA and MANUEL VILLAREAL, JR. and ASST. FISCAL MANASES G. REYES; JUDGE AMADO ROAN, Municipal Court of Manila; JUDGE ROMAN CANSINO, Municipal Court of Manila; JUDGE HERMOGENES CALUAG, Court of First Instance of Rizal-Quezon City Branch, and JUDGE DAMIAN JIMENEZ, Municipal Court of Quezon City, respondents. Paredes, Poblador, Cruz and Nazareno and Meer, Meer and Meer and Juan T. David for petitioners. Office of the Solicitor General Arturo A. Alafriz, Assistant Solicitor General Pacifico P. de Castro, Assistant Solicitor General Frine C. Zaballero, Solicitor Camilo D. Quiason and Solicitor C. Padua for respondents. CONCEPCION, C.J.: Upon application of the officers of the government named on the margin 1 hereinafter referred to as Respondents-Prosecutors several judges2 hereinafter referred to as Respondents-Judges issued, on different dates,3 a total of 42 search warrants against petitioners herein 4 and/or the corporations of which they were officers,5 directed to the any peace officer, to search the persons above-named and/or the premises of their offices, warehouses and/or residences, and to seize and take possession of the following personal property to wit: Books of accounts, financial records, vouchers, correspondence, receipts, ledgers, journals, portfolios, credit journals, typewriters, and other documents and/or papers showing all business transactions including disbursements receipts, balance sheets and profit and loss statements and Bobbins (cigarette wrappers). as "the subject of the offense; stolen or embezzled and proceeds or fruits of the offense," or "used or intended to be used as the means of committing the offense," which is described in the applications adverted to above as "violation of Central Bank Laws, Tariff and Customs Laws, Internal Revenue (Code) and the Revised Penal Code." Alleging that the aforementioned search warrants are null and void, as contravening the Constitution and the Rules of Court because, inter alia: (1) they do not describe with particularity the documents, books and things to be seized; (2) cash money, not mentioned in the warrants, were actually seized; (3) the warrants were issued to fish evidence against the aforementioned petitioners

in deportation cases filed against them; (4) the searches and seizures were made in an illegal manner; and (5) the documents, papers and cash money seized were not delivered to the courts that issued the warrants, to be disposed of in accordance with law on March 20, 1962, said petitioners filed with the Supreme Court this original action for certiorari, prohibition, mandamus and injunction, and prayed that, pending final disposition of the present case, a writ of preliminary injunction be issued restraining Respondents-Prosecutors, their agents and /or representatives from using the effects seized as aforementioned or any copies thereof, in the deportation cases already adverted to, and that, in due course, thereafter, decision be rendered quashing the contested search warrants and declaring the same null and void, and commanding the respondents, their agents or representatives to return to petitioners herein, in accordance with Section 3, Rule 67, of the Rules of Court, the documents, papers, things and cash moneys seized or confiscated under the search warrants in question. In their answer, respondents-prosecutors alleged, 6 (1) that the contested search warrants are valid and have been issued in accordance with law; (2) that the defects of said warrants, if any, were cured by petitioners' consent; and (3) that, in any event, the effects seized are admissible in evidence against herein petitioners, regardless of the alleged illegality of the aforementioned searches and seizures. On March 22, 1962, this Court issued the writ of preliminary injunction prayed for in the petition. However, by resolution dated June 29, 1962, the writ was partially lifted or dissolved, insofar as the papers, documents and things seized from the offices of the corporations above mentioned are concerned; but, the injunction was maintained as regards the papers, documents and things found and seized in the residences of petitioners herein.7 Thus, the documents, papers, and things seized under the alleged authority of the warrants in question may be split into two (2) major groups, namely: (a) those found and seized in the offices of the aforementioned corporations, and (b) those found and seized in the residences of petitioners herein. As regards the first group, we hold that petitioners herein have no cause of action to assail the legality of the contested warrants and of the seizures made in pursuance thereof, for the simple reason that said corporations have their respective personalities, separate and distinct from the personality of herein petitioners, regardless of the amount of shares of stock or of the interest of each of them in said corporations, and whatever the offices they hold therein may be. 8 Indeed, it is well settled that the legality of a seizure can be contested only by the party whose rights have been impaired thereby,9 and that the objection to an unlawful search and seizure is purely personal and cannot be availed of by third parties. 10 Consequently, petitioners herein may not validly object to the use in evidence against them of the documents, papers and things seized

from the offices and premises of the corporations adverted to above, since the right to object to the admission of said papers in evidence belongs exclusively to the corporations, to whom the seized effects belong, and may not be invoked by the corporate officers in proceedings against them in their individual capacity. 11 Indeed, it has been held: . . . that the Government's action in gaining possession of papers belonging to the corporation did not relate to nor did it affect the personal defendants. If these papers were unlawfully seized and thereby the constitutional rights of or any one were invaded, they were the rights of the corporation and not the rights of the other defendants. Next, it is clear that a question of the lawfulness of a seizure can be raised only by one whose rights have been invaded. Certainly, such a seizure, if unlawful, could not affect the constitutional rights of defendants whose property had not been seized or the privacy of whose homes had not been disturbed; nor could they claim for themselves the benefits of the Fourth Amendment, when its violation, if any, was with reference to the rights of another. Remus vs. United States (C.C.A.)291 F. 501, 511. It follows, therefore, that the question of the admissibility of the evidence based on an alleged unlawful search and seizure does not extend to the personal defendants but embraces only the corporation whose property was taken. . . . (A Guckenheimer & Bros. Co. vs. United States, [1925] 3 F. 2d. 786, 789, Emphasis supplied.) With respect to the documents, papers and things seized in the residences of petitioners herein, the aforementioned resolution of June 29, 1962, lifted the writ of preliminary injunction previously issued by this Court, 12 thereby, in effect, restraining herein Respondents-Prosecutors from using them in evidence against petitioners herein. In connection with said documents, papers and things, two (2) important questions need be settled, namely: (1) whether the search warrants in question, and the searches and seizures made under the authority thereof, are valid or not, and (2) if the answer to the preceding question is in the negative, whether said documents, papers and things may be used in evidence against petitioners herein.1wph1.t Petitioners maintain that the aforementioned search warrants are in the nature of general warrants and that accordingly, the seizures effected upon the authority there of are null and void. In this connection, the Constitution 13 provides: The right of the people to be secure in their persons, houses, papers, and effects against unreasonable searches and seizures shall not be violated, and no warrants shall issue but upon probable cause, to be determined by the judge after examination under oath or affirmation of the complainant and the witnesses he may produce, and particularly describing the place to be searched, and the persons or things to be seized.

Two points must be stressed in connection with this constitutional mandate, namely: (1) that no warrant shall issue but upon probable cause, to be determined by the judge in the manner set forth in said provision; and (2) that the warrant shall particularly describe the things to be seized. None of these requirements has been complied with in the contested warrants. Indeed, the same were issued upon applications stating that the natural and juridical person therein named had committed a "violation of Central Ban Laws, Tariff and Customs Laws, Internal Revenue (Code) and Revised Penal Code." In other words, no specific offense had been alleged in said applications. The averments thereof with respect to the offense committed were abstract. As a consequence, it was impossible for the judges who issued the warrants to have found the existence of probable cause, for the same presupposes the introduction of competent proof that the party against whom it is sought has performed particular acts, or committed specific omissions, violating a given provision of our criminal laws. As a matter of fact, the applications involved in this case do not allege any specific acts performed by herein petitioners. It would be the legal heresy, of the highest order, to convict anybody of a "violation of Central Bank Laws, Tariff and Customs Laws, Internal Revenue (Code) and Revised Penal Code," as alleged in the aforementioned applications without reference to any determinate provision of said laws or To uphold the validity of the warrants in question would be to wipe out completely one of the most fundamental rights guaranteed in our Constitution, for it would place the sanctity of the domicile and the privacy of communication and correspondence at the mercy of the whims caprice or passion of peace officers. This is precisely the evil sought to be remedied by the constitutional provision above quoted to outlaw the so-called general warrants. It is not difficult to imagine what would happen, in times of keen political strife, when the party in power feels that the minority is likely to wrest it, even though by legal means. Such is the seriousness of the irregularities committed in connection with the disputed search warrants, that this Court deemed it fit to amend Section 3 of Rule 122 of the former Rules of Court 14 by providing in its counterpart, under the Revised Rules of Court 15 that "a search warrant shall not issue but upon probable cause in connection with one specific offense." Not satisfied with this qualification, the Court added thereto a paragraph, directing that "no search warrant shall issue for more than one specific offense." The grave violation of the Constitution made in the application for the contested search warrants was compounded by the description therein made of the effects to be searched for and seized, to wit: Books of accounts, financial records, vouchers, journals, correspondence, receipts, ledgers, portfolios, credit journals, typewriters, and other documents and/or papers showing all business transactions including disbursement receipts,

balance sheets and related profit and loss statements. Thus, the warrants authorized the search for and seizure of records pertaining to all business transactions of petitioners herein, regardless of whether the transactions were legal or illegal. The warrants sanctioned the seizure of all records of the petitioners and the aforementioned corporations, whatever their nature, thus openly contravening the explicit command of our Bill of Rights that the things to be seized be particularly described as well as tending to defeat its major objective: the elimination of general warrants. Relying upon Moncado vs. People's Court (80 Phil. 1), Respondents-Prosecutors maintain that, even if the searches and seizures under consideration were unconstitutional, the documents, papers and things thus seized are admissible in evidence against petitioners herein. Upon mature deliberation, however, we are unanimously of the opinion that the position taken in the Moncado case must be abandoned. Said position was in line with the American common law rule, that the criminal should not be allowed to go free merely "because the constable has blundered," 16 upon the theory that the constitutional prohibition against unreasonable searches and seizures is protected by means other than the exclusion of evidence unlawfully obtained, 17 such as the common-law action for damages against the searching officer, against the party who procured the issuance of the search warrant and against those assisting in the execution of an illegal search, their criminal punishment, resistance, without liability to an unlawful seizure, and such other legal remedies as may be provided by other laws. However, most common law jurisdictions have already given up this approach and eventually adopted the exclusionary rule, realizing that this is the only practical means of enforcing the constitutional injunction against unreasonable searches and seizures. In the language of Judge Learned Hand: As we understand it, the reason for the exclusion of evidence competent as such, which has been unlawfully acquired, is that exclusion is the only practical way of enforcing the constitutional privilege. In earlier times the action of trespass against the offending official may have been protection enough; but that is true no longer. Only in case the prosecution which itself controls the seizing officials, knows that it cannot profit by their wrong will that wrong be repressed.18 In fact, over thirty (30) years before, the Federal Supreme Court had already declared: If letters and private documents can thus be seized and held and used in evidence against a citizen accused of an offense, the protection of the 4th Amendment, declaring his rights to be secure against such searches and seizures, is of no value, and, so far as those thus placed are concerned, might as well be stricken from the Constitution. The efforts of the courts and their officials

to bring the guilty to punishment, praiseworthy as they are, are not to be aided by the sacrifice of those great principles established by years of endeavor and suffering which have resulted in their embodiment in the fundamental law of the land.19 This view was, not only reiterated, but, also, broadened in subsequent decisions on the same Federal Court. 20 After reviewing previous decisions thereon, said Court held, in Mapp vs. Ohio (supra.): . . . Today we once again examine the Wolf's constitutional documentation of the right of privacy free from unreasonable state intrusion, and after its dozen years on our books, are led by it to close the only courtroom door remaining open to evidence secured by official lawlessness in flagrant abuse of that basic right, reserved to all persons as a specific guarantee against that very same unlawful conduct. We hold that all evidence obtained by searches and seizures in violation of the Constitution is, by that same authority, inadmissible in a State. Since the Fourth Amendment's right of privacy has been declared enforceable against the States through the Due Process Clause of the Fourteenth, it is enforceable against them by the same sanction of exclusion as it used against the Federal Government. Were it otherwise, then just as without the Weeks rule the assurance against unreasonable federal searches and seizures would be "a form of words," valueless and underserving of mention in a perpetual charter of inestimable human liberties, so too, without that rule the freedom from state invasions of privacy would be so ephemeral and so neatly severed from its conceptual nexus with the freedom from all brutish means of coercing evidence as not to permit this Court's high regard as a freedom "implicit in the concept of ordered liberty." At the time that the Court held in Wolf that the amendment was applicable to the States through the Due Process Clause, the cases of this Court as we have seen, had steadfastly held that as to federal officers the Fourth Amendment included the exclusion of the evidence seized in violation of its provisions. Even Wolf "stoutly adhered" to that proposition. The right to when conceded operatively enforceable against the States, was not susceptible of destruction by avulsion of the sanction upon which its protection and enjoyment had always been deemed dependent under the Boyd, Weeks and Silverthorne Cases. Therefore, in extending the substantive protections of due process to all constitutionally unreasonable searches state or federal it was logically and constitutionally necessarily that the exclusion doctrine an essential part of the right to privacy be also insisted upon as an essential ingredient of the right newly recognized by the Wolf Case. In short, the admission of the new constitutional Right by Wolf could not tolerate denial of its most important constitutional privilege, namely, the exclusion of the evidence which an accused had been forced to give by reason of the unlawful seizure. To hold otherwise is to grant the right but in reality to withhold its privilege and enjoyment. Only last year the Court itself recognized that the purpose of the exclusionary rule to "is to deter to compel respect for the constitutional guaranty in the only effectively

available way by removing the incentive to disregard it" . . . . The ignoble shortcut to conviction left open to the State tends to destroy the entire system of constitutional restraints on which the liberties of the people rest. Having once recognized that the right to privacy embodied in the Fourth Amendment is enforceable against the States, and that the right to be secure against rude invasions of privacy by state officers is, therefore constitutional in origin, we can no longer permit that right to remain an empty promise. Because it is enforceable in the same manner and to like effect as other basic rights secured by its Due Process Clause, we can no longer permit it to be revocable at the whim of any police officer who, in the name of law enforcement itself, chooses to suspend its enjoyment. Our decision, founded on reason and truth, gives to the individual no more than that which the Constitution guarantees him to the police officer no less than that to which honest law enforcement is entitled, and, to the courts, that judicial integrity so necessary in the true administration of justice. (emphasis ours.) Indeed, the non-exclusionary rule is contrary, not only to the letter, but also, to the spirit of the constitutional injunction against unreasonable searches and seizures. To be sure, if the applicant for a search warrant has competent evidence to establish probable cause of the commission of a given crime by the party against whom the warrant is intended, then there is no reason why the applicant should not comply with the requirements of the fundamental law. Upon the other hand, if he has no such competent evidence, then it is not possible for the Judge to find that there is probable cause, and, hence, no justification for the issuance of the warrant. The only possible explanation (not justification) for its issuance is the necessity of fishing evidence of the commission of a crime. But, then, this fishing expedition is indicative of the absence of evidence to establish a probable cause. Moreover, the theory that the criminal prosecution of those who secure an illegal search warrant and/or make unreasonable searches or seizures would suffice to protect the constitutional guarantee under consideration, overlooks the fact that violations thereof are, in general, committed By agents of the party in power, for, certainly, those belonging to the minority could not possibly abuse a power they do not have. Regardless of the handicap under which the minority usually but, understandably finds itself in prosecuting agents of the majority, one must not lose sight of the fact that the psychological and moral effect of the possibility 21 of securing their conviction, is watered down by the pardoning power of the party for whose benefit the illegality had been committed. In their Motion for Reconsideration and Amendment of the Resolution of this Court dated June 29, 1962, petitioners allege that Rooms Nos. 81 and 91 of Carmen Apartments, House No. 2008, Dewey Boulevard, House No. 1436, Colorado Street, and Room No. 304 of the Army-Navy Club, should be included among the premises considered in said Resolution as residences of herein

petitioners, Harry S. Stonehill, Robert P. Brook, John J. Brooks and Karl Beck, respectively, and that, furthermore, the records, papers and other effects seized in the offices of the corporations above referred to include personal belongings of said petitioners and other effects under their exclusive possession and control, for the exclusion of which they have a standing under the latest rulings of the federal courts of federal courts of the United States. 22 We note, however, that petitioners' theory, regarding their alleged possession of and control over the aforementioned records, papers and effects, and the alleged "personal" nature thereof, has Been Advanced, not in their petition or amended petition herein, but in the Motion for Reconsideration and Amendment of the Resolution of June 29, 1962. In other words, said theory would appear to be readjustment of that followed in said petitions, to suit the approach intimated in the Resolution sought to be reconsidered and amended. Then, too, some of the affidavits or copies of alleged affidavits attached to said motion for reconsideration, or submitted in support thereof, contain either inconsistent allegations, or allegations inconsistent with the theory now advanced by petitioners herein. Upon the other hand, we are not satisfied that the allegations of said petitions said motion for reconsideration, and the contents of the aforementioned affidavits and other papers submitted in support of said motion, have sufficiently established the facts or conditions contemplated in the cases relied upon by the petitioners; to warrant application of the views therein expressed, should we agree thereto. At any rate, we do not deem it necessary to express our opinion thereon, it being best to leave the matter open for determination in appropriate cases in the future. We hold, therefore, that the doctrine adopted in the Moncado case must be, as it is hereby, abandoned; that the warrants for the search of three (3) residences of herein petitioners, as specified in the Resolution of June 29, 1962, are null and void; that the searches and seizures therein made are illegal; that the writ of preliminary injunction heretofore issued, in connection with the documents, papers and other effects thus seized in said residences of herein petitioners is hereby made permanent; that the writs prayed for are granted, insofar as the documents, papers and other effects so seized in the aforementioned residences are concerned; that the aforementioned motion for Reconsideration and Amendment should be, as it is hereby, denied; and that the petition herein is dismissed and the writs prayed for denied, as regards the documents, papers and other effects seized in the twenty-nine (29) places, offices and other premises enumerated in the same Resolution, without special pronouncement as to costs. It is so ordered. Reyes, J.B.L., Dizon, Makalintal, Bengzon, J.P., Zaldivar and Sanchez, JJ.,

concur. CASTRO, J., concurring and dissenting: From my analysis of the opinion written by Chief Justice Roberto Concepcion and from the import of the deliberations of the Court on this case, I gather the following distinct conclusions: 1. All the search warrants served by the National Bureau of Investigation in this case are general warrants and are therefore proscribed by, and in violation of, paragraph 3 of section 1 of Article III (Bill of Rights) of the Constitution; 2. All the searches and seizures conducted under the authority of the said search warrants were consequently illegal; 3. The non-exclusionary rule enunciated in Moncado vs. People, 80 Phil. 1, should be, and is declared, abandoned; 4. The search warrants served at the three residences of the petitioners are expressly declared null and void the searches and seizures therein made are expressly declared illegal; and the writ of preliminary injunction heretofore issued against the use of the documents, papers and effect seized in the said residences is made permanent; and 5. Reasoning that the petitioners have not in their pleadings satisfactorily demonstrated that they have legal standing to move for the suppression of the documents, papers and effects seized in the places other than the three residences adverted to above, the opinion written by the Chief Justice refrains from expressly declaring as null and void the such warrants served at such other places and as illegal the searches and seizures made therein, and leaves "the matter open for determination in appropriate cases in the future." It is precisely the position taken by the Chief Justice summarized in the immediately preceding paragraph (numbered 5) with which I am not in accord. I do not share his reluctance or unwillingness to expressly declare, at this time, the nullity of the search warrants served at places other than the three residences, and the illegibility of the searches and seizures conducted under the authority thereof. In my view even the exacerbating passions and prejudices inordinately generated by the environmental political and moral developments of this case should not deter this Court from forthrightly laying down the law not only for this case but as well for future cases and future generations. All the search warrants, without exception, in this case are admittedly general, blanket and roving warrants and are therefore admittedly and indisputably outlawed by the Constitution; and the searches and seizures made were therefore unlawful. That the petitioners, let us assume in gratia argumente, have no legal standing to ask

for the suppression of the papers, things and effects seized from places other than their residences, to my mind, cannot in any manner affect, alter or otherwise modify the intrinsic nullity of the search warrants and the intrinsic illegality of the searches and seizures made thereunder. Whether or not the petitioners possess legal standing the said warrants are void and remain void, and the searches and seizures were illegal and remain illegal. No inference can be drawn from the words of the Constitution that "legal standing" or the lack of it is a determinant of the nullity or validity of a search warrant or of the lawfulness or illegality of a search or seizure. On the question of legal standing, I am of the conviction that, upon the pleadings submitted to this Court the petitioners have the requisite legal standing to move for the suppression and return of the documents, papers and effects that were seized from places other than their family residences. Our constitutional provision on searches and seizures was derived almost verbatim from the Fourth Amendment to the United States Constitution. In the many years of judicial construction and interpretation of the said constitutional provision, our courts have invariably regarded as doctrinal the pronouncement made on the Fourth Amendment by federal courts, especially the Federal Supreme Court and the Federal Circuit Courts of Appeals. The U.S. doctrines and pertinent cases on standing to move for the suppression or return of documents, papers and effects which are the fruits of an unlawful search and seizure, may be summarized as follows; (a) ownership of documents, papers and effects gives "standing;" (b) ownership and/or control or possession actual or constructive of premises searched gives "standing"; and (c) the "aggrieved person" doctrine where the search warrant and the sworn application for search warrant are "primarily" directed solely and exclusively against the "aggrieved person," gives "standing." An examination of the search warrants in this case will readily show that, excepting three, all were directed against the petitioners personally. In some of them, the petitioners were named personally, followed by the designation, "the President and/or General Manager" of the particular corporation. The three warrants excepted named three corporate defendants. But the "office/house/warehouse/premises" mentioned in the said three warrants were also the same "office/house/warehouse/premises" declared to be owned by or under the control of the petitioners in all the other search warrants directed against the petitioners and/or "the President and/or General Manager" of the particular corporation. (see pages 5-24 of Petitioners' Reply of April 2, 1962). The searches and seizures were to be made, and were actually made, in the "office/house/warehouse/premises" owned by or under the control of the petitioners. Ownership of matters seized gives "standing."

Ownership of the properties seized alone entitles the petitioners to bring a motion to return and suppress, and gives them standing as persons aggrieved by an unlawful search and seizure regardless of their location at the time of seizure. Jones vs. United States, 362 U.S. 257, 261 (1960) (narcotics stored in the apartment of a friend of the defendant); Henzel vs. United States, 296 F. 2d. 650, 652-53 (5th Cir. 1961), (personal and corporate papers of corporation of which the defendant was president), United States vs. Jeffers, 342 U.S. 48 (1951) (narcotics seized in an apartment not belonging to the defendant); Pielow vs. United States, 8 F. 2d 492, 493 (9th Cir. 1925) (books seized from the defendant's sister but belonging to the defendant); Cf. Villano vs. United States, 310 F. 2d 680, 683 (10th Cir. 1962) (papers seized in desk neither owned by nor in exclusive possession of the defendant). In a very recent case (decided by the U.S. Supreme Court on December 12, 1966), it was held that under the constitutional provision against unlawful searches and seizures, a person places himself or his property within a constitutionally protected area, be it his home or his office, his hotel room or his automobile: Where the argument falls is in its misapprehension of the fundamental nature and scope of Fourth Amendment protection. What the Fourth Amendment protects is the security a man relies upon when he places himself or his property within a constitutionally protected area, be it his home or his office, his hotel room or his automobile. There he is protected from unwarranted governmental intrusion. And when he puts some thing in his filing cabinet, in his desk drawer, or in his pocket, he has the right to know it will be secure from an unreasonable search or an unreasonable seizure. So it was that the Fourth Amendment could not tolerate the warrantless search of the hotel room in Jeffers, the purloining of the petitioner's private papers in Gouled, or the surreptitious electronic surveilance in Silverman. Countless other cases which have come to this Court over the years have involved a myriad of differing factual contexts in which the protections of the Fourth Amendment have been appropriately invoked. No doubt, the future will bring countless others. By nothing we say here do we either foresee or foreclose factual situations to which the Fourth Amendment may be applicable. (Hoffa vs. U.S., 87 S. Ct. 408 (December 12, 1966). See also U.S. vs. Jeffers, 342 U.S. 48, 72 S. Ct. 93 (November 13, 1951). (Emphasis supplied). Control of premises searched gives "standing." Independent of ownership or other personal interest in the records and documents seized, the petitioners have standing to move for return and suppression by virtue of their proprietary or leasehold interest in many of the premises searched. These proprietary and leasehold interests have been sufficiently set forth in their motion for reconsideration and need not be recounted here, except to emphasize that the petitioners paid rent, directly or indirectly, for practically all the premises searched (Room 91, 84 Carmen Apts; Room 304,

Army & Navy Club; Premises 2008, Dewey Boulevard; 1436 Colorado Street); maintained personal offices within the corporate offices (IBMC, USTC); had made improvements or furnished such offices; or had paid for the filing cabinets in which the papers were stored (Room 204, Army & Navy Club); and individually, or through their respective spouses, owned the controlling stock of the corporations involved. The petitioners' proprietary interest in most, if not all, of the premises searched therefore independently gives them standing to move for the return and suppression of the books, papers and affects seized therefrom. In Jones vs. United States, supra, the U.S. Supreme Court delineated the nature and extent of the interest in the searched premises necessary to maintain a motion to suppress. After reviewing what it considered to be the unduly technical standard of the then prevailing circuit court decisions, the Supreme Court said (362 U.S. 266): We do not lightly depart from this course of decisions by the lower courts. We are persuaded, however, that it is unnecessarily and ill-advised to import into the law surrounding the constitutional right to be free from unreasonable searches and seizures subtle distinctions, developed and refined by the common law in evolving the body of private property law which, more than almost any other branch of law, has been shaped by distinctions whose validity is largely historical. Even in the area from which they derive, due consideration has led to the discarding of those distinctions in the homeland of the common law. See Occupiers' Liability Act, 1957, 5 and 6 Eliz. 2, c. 31, carrying out Law Reform Committee, Third Report, Cmd. 9305. Distinctions such as those between "lessee", "licensee," "invitee," "guest," often only of gossamer strength, ought not be determinative in fashioning procedures ultimately referable to constitutional safeguards. See also Chapman vs. United States, 354 U.S. 610, 616-17 (1961). It has never been held that a person with requisite interest in the premises searched must own the property seized in order to have standing in a motion to return and suppress. In Alioto vs. United States, 216 F. Supp. 48 (1963), a Bookkeeper for several corporations from whose apartment the corporate records were seized successfully moved for their return. In United States vs. Antonelli, Fireworks Co., 53 F. Supp. 870, 873 (W D. N. Y. 1943), the corporation's president successfully moved for the return and suppression is to him of both personal and corporate documents seized from his home during the course of an illegal search: The lawful possession by Antonelli of documents and property, "either his own or the corporation's was entitled to protection against unreasonable search and seizure. Under the circumstances in the case at bar, the search and seizure were unreasonable and unlawful. The motion for the return of seized article and the suppression of the evidence so obtained should be granted. (Emphasis supplied).

Time was when only a person who had property in interest in either the place searched or the articles seize had the necessary standing to invoke the protection of the exclusionary rule. But in MacDonald vs. Unite States, 335 U.S. 461 (1948), Justice Robert Jackson joined by Justice Felix Frankfurter, advanced the view that "even a guest may expect the shelter of the rooftree he is under against criminal intrusion." This view finally became the official view of the U.S. Supreme Court and was articulated in United States vs. Jeffers, 432 U.S 48 (1951). Nine years later, in 1960, in Jones vs. Unite States, 362 U.S. 257, 267, the U.S. Supreme Court went a step further. Jones was a mere guest in the apartment unlawfully searched but the Court nonetheless declared that the exclusionary rule protected him as well. The concept of "person aggrieved by an unlawful search and seizure" was enlarged to include "anyone legitimately on premise where the search occurs." Shortly after the U.S. Supreme Court's Jones decision the U.S. Court of Appeals for the Fifth Circuit held that the defendant organizer, sole stockholder and president of a corporation had standing in a mail fraud prosecution against him to demand the return and suppression of corporate property. Henzel vs. United States, 296 F 2d 650, 652 (5th Cir. 1961), supra. The court conclude that the defendant had standing on two independent grounds: First he had a sufficient interest in the property seized, and second he had an adequate interest in the premises searched (just like in the case at bar). A postal inspector had unlawfully searched the corporation' premises and had seized most of the corporation's book and records. Looking to Jones, the court observed: Jones clearly tells us, therefore, what is not required qualify one as a "person aggrieved by an unlawful search and seizure." It tells us that appellant should not have been precluded from objecting to the Postal Inspector's search and seizure of the corporation's books and records merely because the appellant did not show ownership or possession of the books and records or a substantial possessory interest in the invade premises . . . (Henzel vs. United States, 296 F. 2d at 651). . Henzel was soon followed by Villano vs. United States, 310 F. 2d 680, 683, (10th Cir. 1962). In Villano, police officers seized two notebooks from a desk in the defendant's place of employment; the defendant did not claim ownership of either; he asserted that several employees (including himself) used the notebooks. The Court held that the employee had a protected interest and that there also was an invasion of privacy. Both Henzel and Villano considered also the fact that the search and seizure were "directed at" the moving defendant. Henzel vs. United States, 296 F. 2d at 682; Villano vs. United States, 310 F. 2d at 683. In a case in which an attorney closed his law office, placed his files in storage and went to Puerto Rico, the Court of Appeals for the Eighth Circuit recognized his standing to move to quash as unreasonable search and seizure under the

Fourth Amendment of the U.S. Constitution a grand jury subpoena duces tecum directed to the custodian of his files. The Government contended that the petitioner had no standing because the books and papers were physically in the possession of the custodian, and because the subpoena was directed against the custodian. The court rejected the contention, holding that Schwimmer legally had such possession, control and unrelinquished personal rights in the books and papers as not to enable the question of unreasonable search and seizure to be escaped through the mere procedural device of compelling a third-party naked possessor to produce and deliver them. Schwimmer vs. United States, 232 F. 2d 855, 861 (8th Cir. 1956). Aggrieved person doctrine where the search warrant s primarily directed against said person gives "standing." The latest United States decision squarely in point is United States vs. Birrell, 242 F. Supp. 191 (1965, U.S.D.C. S.D.N.Y.). The defendant had stored with an attorney certain files and papers, which attorney, by the name of Dunn, was not, at the time of the seizing of the records, Birrell's attorney. * Dunn, in turn, had stored most of the records at his home in the country and on a farm which, according to Dunn's affidavit, was under his (Dunn's) "control and management." The papers turned out to be private, personal and business papers together with corporate books and records of certain unnamed corporations in which Birrell did not even claim ownership. (All of these type records were seized in the case at bar). Nevertheless, the search in Birrell was held invalid by the court which held that even though Birrell did not own the premises where the records were stored, he had "standing" to move for the return of all the papers and properties seized. The court, relying on Jones vs. U.S., supra; U.S. vs. Antonelli Fireworks Co., 53 F. Supp. 870, Aff'd 155 F. 2d 631: Henzel vs. U.S., supra; and Schwimmer vs. U.S., supra, pointed out that It is overwhelmingly established that the searches here in question were directed solely and exclusively against Birrell. The only person suggested in the papers as having violated the law was Birrell. The first search warrant described the records as having been used "in committing a violation of Title 18, United States Code, Section 1341, by the use of the mails by one Lowell M. Birrell, . . ." The second search warrant was captioned: "United States of America vs. Lowell M. Birrell. (p. 198) Possession (actual or constructive), no less than ownership, gives standing to move to suppress. Such was the rule even before Jones. (p. 199) If, as thus indicated Birrell had at least constructive possession of the records stored with Dunn, it matters not whether he had any interest in the premises searched. See also Jeffers v. United States, 88 U.S. Appl. D.C. 58, 187 F. 2d 498 (1950), affirmed 432 U.S. 48, 72 S. Ct. 93, 96 L. Ed. 459 (1951).

The ruling in the Birrell case was reaffirmed on motion for reargument; the United States did not appeal from this decision. The factual situation in Birrell is strikingly similar to the case of the present petitioners; as in Birrell, many personal and corporate papers were seized from premises not petitioners' family residences; as in Birrell, the searches were "PRIMARILY DIRECTED SOLETY AND EXCLUSIVELY" against the petitioners. Still both types of documents were suppressed in Birrell because of the illegal search. In the case at bar, the petitioners connection with the premises raided is much closer than in Birrell. Thus, the petitioners have full standing to move for the quashing of all the warrants regardless whether these were directed against residences in the narrow sense of the word, as long as the documents were personal papers of the petitioners or (to the extent that they were corporate papers) were held by them in a personal capacity or under their personal control. Prescinding a from the foregoing, this Court, at all events, should order the return to the petitioners all personal and private papers and effects seized, no matter where these were seized, whether from their residences or corporate offices or any other place or places. The uncontradicted sworn statements of the petitioners in their, various pleadings submitted to this Court indisputably show that amongst the things seized from the corporate offices and other places were personal and private papers and effects belonging to the petitioners. If there should be any categorization of the documents, papers and things which where the objects of the unlawful searches and seizures, I submit that the grouping should be: (a) personal or private papers of the petitioners were they were unlawfully seized, be it their family residences offices, warehouses and/or premises owned and/or possessed (actually or constructively) by them as shown in all the search and in the sworn applications filed in securing the void search warrants and (b) purely corporate papers belonging to corporations. Under such categorization or grouping, the determination of which unlawfully seized papers, documents and things are personal/private of the petitioners or purely corporate papers will have to be left to the lower courts which issued the void search warrants in ultimately effecting the suppression and/or return of the said documents. And as unequivocally indicated by the authorities above cited, the petitioners likewise have clear legal standing to move for the suppression of purely corporate papers as "President and/or General Manager" of the corporations involved as specifically mentioned in the void search warrants. Finally, I must articulate my persuasion that although the cases cited in my disquisition were criminal prosecutions, the great clauses of the constitutional proscription on illegal searches and seizures do not withhold the mantle of their protection from cases not criminal in origin or nature.

G.R. No. 82585 November 14, 1988 MAXIMO V. SOLIVEN, ANTONIO V. ROCES, FREDERICK K. AGCAOLI, and GODOFREDO L. MANZANAS, petitioners, vs. THE HON. RAMON P. MAKASIAR, Presiding Judge of the Regional Trial Court of Manila, Branch 35, UNDERSECRETARY SILVESTRE BELLO III, of the Department of Justice, LUIS C. VICTOR, THE CITY FISCAL OF MANILA and PRESIDENT CORAZON C. AQUINO, respondents. G.R. No. 82827 November 14, 1988 LUIS D. BELTRAN, petitioner, vs. THE HON. RAMON P. MAKASIAR, Presiding Judge of Branch 35 of the Regional Trial Court, at Manila, THE HON. LUIS VICTOR, CITY FISCAL OF MANILA, PEOPLE OF THE PHILIPPINES, SUPERINTENDENT OF THE WESTERN POLICE DISTRICT, and THE MEMBERS OF THE PROCESS SERVING UNIT AT THE REGIONAL TRIAL COURT OF MANILA, respondents. G.R. No. 83979 November 14, 1988. LUIS D. BELTRAN, petitioner, vs. EXECUTIVE SECRETARY CATALINO MACARAIG, SECRETARY OF JUSTICE SEDFREY ORDOEZ, UNDERSECRETARY OF JUSTICE SILVESTRE BELLO III, THE CITY FISCAL OF MANILA JESUS F. GUERRERO, and JUDGE RAMON P. MAKASIAR, Presiding Judge of Branch 35 of the Regional Trial Court, at Manila, respondents. Angara, Abello, Concepcion, Regala and Cruz for petitioners in G.R. No. 82585. Perfecto V. Fernandez, Jose P. Fernandez and Cristobal P. Fernandez for petitioner in G.R. Nos. 82827 and 83979. RESOLUTION

PER CURIAM: In these consolidated cases, three principal issues were raised: (1) whether or not petitioners were denied due process when informations for libel were filed against them although the finding of the existence of a prima facie case was still under review by the Secretary of Justice and, subsequently, by the President; (2) whether or not the constitutional rights of Beltran were violated when respondent RTC judge issued a warrant for his arrest without personally examining the complainant and the witnesses, if any, to determine probable cause; and (3) whether or not the President of the Philippines, under the Constitution, may

initiate criminal proceedings against the petitioners through the filing of a complaint-affidavit. Subsequent events have rendered the first issue moot and academic. On March 30, 1988, the Secretary of Justice denied petitioners' motion for reconsideration and upheld the resolution of the Undersecretary of Justice sustaining the City Fiscal's finding of a prima facie case against petitioners. A second motion for reconsideration filed by petitioner Beltran was denied by the Secretary of Justice on April 7, 1988. On appeal, the President, through the Executive Secretary, affirmed the resolution of the Secretary of Justice on May 2, 1988. The motion for reconsideration was denied by the Executive Secretary on May 16, 1988. With these developments, petitioners' contention that they have been denied the administrative remedies available under the law has lost factual support. It may also be added that with respect to petitioner Beltran, the allegation of denial of due process of law in the preliminary investigation is negated by the fact that instead of submitting his counter- affidavits, he filed a "Motion to Declare Proceedings Closed," in effect waiving his right to refute the complaint by filing counter-affidavits. Due process of law does not require that the respondent in a criminal case actually file his counter-affidavits before the preliminary investigation is deemed completed. All that is required is that the respondent be given the opportunity to submit counter-affidavits if he is so minded. The second issue, raised by petitioner Beltran, calls for an interpretation of the constitutional provision on the issuance of warrants of arrest. The pertinent provision reads: Art. III, Sec. 2. The right of the people to be secure in their persons, houses, papers and effects against unreasonable searches and seizures of whatever nature and for any purpose shall be inviolable, and no search warrant or warrant of arrest shall issue except upon probable cause to be determined personally by the judge after examination nder oath or affirmation of the complainant and the witnesses he may produce, and particularly describing the place to be searched and the persons or things to be seized. The addition of the word "personally" after the word "determined" and the deletion of the grant of authority by the 1973 Constitution to issue warrants to "other responsible officers as may be authorized by law," has apparently convinced petitioner Beltran that the Constitution now requires the judge to personally examine the complainant and his witnesses in his determination of probable cause for the issuance of warrants of arrest. This is not an accurate interpretation. What the Constitution underscores is the exclusive and personal responsibility of the issuing judge to satisfy himself of the existence of probable cause. In satisfying himself of the existence of probable cause for the issuance of a

warrant of arrest, the judge is not required to personally examine the complainant and his witnesses. Following established doctrine and procedure, he shall: (1) personally evaluate the report and the supporting documents submitted by the fiscal regarding the existence of probable cause and, on the basis thereof, issue a warrant of arrest; or (2) if on the basis thereof he finds no probable cause, he may disregard the fiscal's report and require the submission of supporting affidavits of witnesses to aid him in arriving at a conclusion as to the existence of probable cause. Sound policy dictates this procedure, otherwise judges would be unduly laden with the preliminary examination and investigation of criminal complaints instead of concentrating on hearing and deciding cases filed before their courts. On June 30, 1987, the Supreme Court unanimously adopted Circular No. 12, setting down guidelines for the issuance of warrants of arrest. The procedure therein provided is reiterated and clarified in this resolution. It has not been shown that respondent judge has deviated from the prescribed procedure. Thus, with regard to the issuance of the warrants of arrest, a finding of grave abuse of discretion amounting to lack or excess of jurisdiction cannot be sustained. Anent the third issue, petitioner Beltran argues that "the reasons which necessitate presidential immunity from suit impose a correlative disability to file suit." He contends that if criminal proceedings ensue by virtue of the President's filing of her complaint-affidavit, she may subsequently have to be a witness for the prosecution, bringing her under the trial court's jurisdiction. This, continues Beltran, would in an indirect way defeat her privilege of immunity from suit, as by testifying on the witness stand, she would be exposing herself to possible contempt of court or perjury. The rationale for the grant to the President of the privilege of immunity from suit is to assure the exercise of Presidential duties and functions free from any hindrance or distraction, considering that being the Chief Executive of the Government is a job that, aside from requiring all of the office holder's time, also demands undivided attention. But this privilege of immunity from suit, pertains to the President by virtue of the office and may be invoked only by the holder of the office; not by any other person in the President's behalf. Thus, an accused in a criminal case in which the President is complainant cannot raise the presidential privilege as a defense to prevent the case from proceeding against such accused. Moreover, there is nothing in our laws that would prevent the President from waiving the privilege. Thus, if so minded the President may shed the protection afforded by the privilege and submit to the court's jurisdiction. The choice of

whether to exercise the privilege or to waive it is solely the President's prerogative. It is a decision that cannot be assumed and imposed by any other person. As regards the contention of petitioner Beltran that he could not be held liable for libel because of the privileged character or the publication, the Court reiterates that it is not a trier of facts and that such a defense is best left to the trial court to appreciate after receiving the evidence of the parties. As to petitioner Beltran's claim that to allow the libel case to proceed would produce a "chilling effect" on press freedom, the Court finds no basis at this stage to rule on the point. The petitions fail to establish that public respondents, through their separate acts, gravely abused their discretion as to amount to lack of jurisdiction. Hence, the writs of certiorari and prohibition prayed for cannot issue. WHEREFORE, finding no grave abuse of discretion amounting to excess or lack of jurisdiction on the part of the public respondents, the Court Resolved to DISMISS the petitions in G. R. Nos. 82585, 82827 and 83979. The Order to maintain the status quo contained in the Resolution of the Court en banc dated April 7, 1988 and reiterated in the Resolution dated April 26, 1988 is LIFTED. Fernan, C.J., Narvasa, Melencio-Herrera, Cruz, Paras, Feliciano, Gancayco, Padilla, Bidin, Sarmiento, Cortes, Grio-Aquino Medialdea and Regalado, JJ., concur.

Separate Opinions

GUTIERREZ, JR., J., concurring: I concur with the majority opinion insofar as it involves the three principal issues mentioned in its opening statement. However, as to the more important issue on whether or not the prosecution of the libel case would produce a "chilling effect" on press freedom, I beg to reserve my vote. I believe this is the more important issue in these petitions and it should be resolved now rather that later. Consistent with our decision in Salonga v. Cruz Pano (134 SCRA 438 [1985]), the Court should not hesitate to quash a criminal prosecution in the interest of more enlightened and substantial justice where it is not alone the criminal liability

of an accused in a seemingly minor libel case which is involved but broader considerations of governmental power versus a preferred freedom. We have in these four petitions the unusual situation where the highest official of the Republic and one who enjoys unprecedented public support asks for the prosecution of a newspaper columnist, the publisher and chairman of the editorial board, the managing editor and the business manager in a not too indubitable a case for alleged libel. I am fully in accord with an all out prosecution if the effect will be limited to punishing a newspaperman who, instead of observing accuracy and fairness, engages in unwarranted personal attacks, irresponsible twisting of facts, of malicious distortions of half-truths which tend to cause dishonor, discredit, or contempt of the complainant. However, this case is not a simple prosecution for libel. We have as complainant a powerful and popular President who heads the investigation and prosecution service and appoints members of appellate courts but who feels so terribly maligned that she has taken the unorthodox step of going to court inspite of the invocations of freedom of the press which would inevitably follow. I believe that this Court should have acted on this issue now instead of leaving the matter to fiscals and defense lawyers to argue before a trial judge. There is always bound to be harassment inherent in any criminal prosecution. Where the harassment goes beyond the usual difficulties encountered by any accused and results in an unwillingness of media to freely criticize government or to question government handling of sensitive issues and public affairs, this Court and not a lower tribunal should draw the demarcation line. As early as March 8, 1918, the decision in United States v. Bustos (37 Phil. 731) stated that "(c)omplete liberty to comment on the conduct of public men is a scalpel in the case of free speech. The sharp incision of its probe relieves the abscesses of officialdom. Men in public life may suffer under a hostile and unjust accusation; the wound can be assuaged with the balm of a clear conscience." The Court pointed out that while defamation is not authorized, criticism is to be expected and should be borne for the common good. In People v. Perfecto (43 Phil. 887 [1922]), the Court stated: xxx xxx xxx ... No longer is there a Minister of the Crown own or a person in authority of such exalted position that the citizen must speak of him only with bated breath. "In the eye of our Constitution and laws, every man is a sovereign, a ruler and a freeman, and has equal rights with every other man." (at p. 900)

In fact, the Court observed that high official position, instead of affording immunity from slanderous and libelous charges, would actually invite attacks by those who desire to create sensation. It would seem that what would ordinarily be slander if directed at the typical person should be examined from various perspectives if directed at a high government official. Again, the Supreme Court should draw this fine line instead of leaving it to lower tribunals. This Court has stressed as authoritative doctrine in Elizalde v. Gutierrez (76 SCRA 448 [1977]) that a prosecution for libel lacks justification if the offending words find sanctuary within the shelter of the free press guaranty. In other words, a prosecution for libel should not be allowed to continue, where after discounting the possibility that the words may not be really that libelous, there is likely to be a chilling effect, a patently inhibiting factor on the willingness of newspapermen, especially editors and publishers to courageously perform their critical role in society. If, instead of merely reading more carefully what a columnist writes in his daily column, the editors tell their people to lay off certain issues or certain officials, the effect on a free press would be highly injurious. Because many questions regarding press freedom are left unanswered by our resolution, I must call attention to our decisions which caution that "no inroads on press freedom should be allowed in the guise of punitive action visited on what otherwise should be characterized as libel." (Lopez v. Court of Appeals, 34 SCRA 117 [1970]; See also the citations in Elizalde v. Gutierrez, supra). The United States Supreme Court is even more emphatic, to wit: In deciding the question now, we are compelled by neither precedent nor policy to give any more weight to the epithet "libel" than we have to other "mere labels" of state law. N. A. A. C. P. v. Button, 371 US 415, 429, 9L ed 2d 405, 415, 83 S Ct 328. Like insurrection, contempt, advocacy of unlawful acts, breach of the peace, obscenity, solicitation of legal business, and the other various other formulae for the repression of expression that have been challenged in this Court, libel can claim no talismanic immunity from constitutional limitations. It must be measured by standards that satisfy the First Amendment. xxx xxx xxx Those who won our independence believed ... that public discussion is a political duty; and that this should be a fundamental principle of the American government. They recognized the risk to which all human institutions are subject. But they knew that order cannot be secured merely through fear of punishment for its infraction; that it is hazardous to discourage thought, hope and imagination; that fear breeds repression; that repression breeds hate; that hate menaces stable government; that the path of safety lies in the opportunity to discuss freely supposed grievances and proposed remedies; and that the fitting remedy for evil counsel is good ones. Believing in the power of reason as applied

through public discussion, they eschewed silence coerced by lawthe argument of force in its worst form. ... Thus we consider this case against the background of a profound national commitment to the principle that debate on public issues should be uninhibited, robust, and wide open, and that it may well include vehement, caustic, and sometimes unpleasantly sharp attacks on government and public officials. ... (at pp. 700-701) Shunting aside the individual liability of Mr. Luis Beltran, is there a prima facie showing that Messrs. Maximo Soliven, Antonio V. Roces, Frederick K. Agcaoili, and Godofredo L. Manzanas knowingly participated in a wilful purveying of falsehood? Considering the free speech aspects of these petitions, should not a differentiated approach to their particular liabilities be taken instead of lumping up everybody with the offending columnist? I realize that the law includes publishers and editors but perhaps the "chilling effect" issue applies with singular effectivity to publishers and editors vis-a-vis newspaper columnists. There is no question that, ordinarily, libel is not protected by the free speech clause but we have to understand that some provocative words, which if taken literally may appear to shame or disparage a public figure, may really be intended to provoke debate on public issues when uttered or written by a media personality. Will not a criminal prosecution in the type of case now before us dampen the vigor and limit the variety of public debate? There are many other questions arising from this unusual case which have not been considered. I, of course, concur with the Court's opinion because it has decided to limit the issues to narrowly drawn ones. I see no reason to disagree with the way the Court has resolved them. The first issue on prematurity is moot. The second issue discusses a procedure now embodied in the recently amended Rules of Court on how a Judge should proceed before he issues a warrant of arrest. Anent the third issue, considerations of public policy dictate that an incumbent President should not be sued. At the same time, the President cannot stand by helplessly bereft of legal remedies if somebody vilifies or maligns him or her. The Court has decided to defer the "chilling effect" issue for a later day. To this, I take exception. I know that most of our fiscals and judges are courageous individuals who would not allow any considerations of possible consequences to their careers to stand in the way of public duty. But why should we subject them to this problem? And why should we allow the possibility of the trial court treating and deciding the case as one for ordinary libel without bothering to fully explore the more important areas of concern, the extremely difficult issues involving government power and freedom of expression. However, since we have decided to defer the "chilling effect" issue for a later day, I limit myself to reiterating the dissenting words of Mr. Justice Jackson in the American case of Beaurnhais v. Illinois (343 U. S. 250) when he said:

If one can claim to announce the judgment of legal history on any subject, it is that criminal libel laws are consistent with the concept of ordered liberty only when applied with safeguards evolved to prevent their invasion of freedom of expression. In the trial of the libel case against the petitioners, the safeguards in the name of freedom of expression should be faithfully applied. Separate Opinions GUTIERREZ, JR., J., concurring: I concur with the majority opinion insofar as it involves the three principal issues mentioned in its opening statement. However, as to the more important issue on whether or not the prosecution of the libel case would produce a "chilling effect" on press freedom, I beg to reserve my vote. I believe this is the more important issue in these petitions and it should be resolved now rather that later. Consistent with our decision in Salonga v. Cruz Pano (134 SCRA 438 [1985]), the Court should not hesitate to quash a criminal prosecution in the interest of more enlightened and substantial justice where it is not alone the criminal liability of an accused in a seemingly minor libel case which is involved but broader considerations of governmental power versus a preferred freedom. We have in these four petitions the unusual situation where the highest official of the Republic and one who enjoys unprecedented public support asks for the prosecution of a newspaper columnist, the publisher and chairman of the editorial board, the managing editor and the business manager in a not too indubitable a case for alleged libel. I am fully in accord with an all out prosecution if the effect will be limited to punishing a newspaperman who, instead of observing accuracy and fairness, engages in unwarranted personal attacks, irresponsible twisting of facts, of malicious distortions of half-truths which tend to cause dishonor, discredit, or contempt of the complainant. However, this case is not a simple prosecution for libel. We have as complainant a powerful and popular President who heads the investigation and prosecution service and appoints members of appellate courts but who feels so terribly maligned that she has taken the unorthodox step of going to court inspite of the invocations of freedom of the press which would inevitably follow. I believe that this Court should have acted on this issue now instead of leaving the matter to fiscals and defense lawyers to argue before a trial judge. There is always bound to be harassment inherent in any criminal prosecution. Where the harassment goes beyond the usual difficulties encountered by any

accused and results in an unwillingness of media to freely criticize government or to question government handling of sensitive issues and public affairs, this Court and not a lower tribunal should draw the demarcation line. As early as March 8, 1918, the decision in United States v. Bustos (37 Phil. 731) stated that "(c)omplete liberty to comment on the conduct of public men is a scalpel in the case of free speech. The sharp incision of its probe relieves the abscesses of officialdom. Men in public life may suffer under a hostile and unjust accusation; the wound can be assuaged with the balm of a clear conscience." The Court pointed out that while defamation is not authorized, criticism is to be expected and should be borne for the common good. In People v. Perfecto (43 Phil. 887 [1922]), the Court stated: xxx xxx xxx ... No longer is there a Minister of the Crown own or a person in authority of such exalted position that the citizen must speak of him only with bated breath. "In the eye of our Constitution and laws, every man is a sovereign, a ruler and a freeman, and has equal rights with every other man." (at p. 900) In fact, the Court observed that high official position, instead of affording immunity from slanderous and libelous charges, would actually invite attacks by those who desire to create sensation. It would seem that what would ordinarily be slander if directed at the typical person should be examined from various perspectives if directed at a high government official. Again, the Supreme Court should draw this fine line instead of leaving it to lower tribunals. This Court has stressed as authoritative doctrine in Elizalde v. Gutierrez (76 SCRA 448 [1977]) that a prosecution for libel lacks justification if the offending words find sanctuary within the shelter of the free press guaranty. In other words, a prosecution for libel should not be allowed to continue, where after discounting the possibility that the words may not be really that libelous, there is likely to be a chilling effect, a patently inhibiting factor on the willingness of newspapermen, especially editors and publishers to courageously perform their critical role in society. If, instead of merely reading more carefully what a columnist writes in his daily column, the editors tell their people to lay off certain issues or certain officials, the effect on a free press would be highly injurious. Because many questions regarding press freedom are left unanswered by our resolution, I must call attention to our decisions which caution that "no inroads on press freedom should be allowed in the guise of punitive action visited on what otherwise should be characterized as libel." (Lopez v. Court of Appeals, 34 SCRA 117 [1970]; See also the citations in Elizalde v. Gutierrez, supra).<re||an1w>

The United States Supreme Court is even more emphatic, to wit: In deciding the question now, we are compelled by neither precedent nor policy to give any more weight to the epithet "libel" than we have to other "mere labels" of state law. N. A. A. C. P. v. Button, 371 US 415, 429, 9L ed 2d 405, 415, 83 S Ct 328. Like insurrection, contempt, advocacy of unlawful acts, breach of the peace, obscenity, solicitation of legal business, and the other various other formulae for the repression of expression that have been challenged in this Court, libel can claim no talismanic immunity from constitutional limitations. It must be measured by standards that satisfy the First Amendment. xxx xxx xxx Those who won our independence believed ... that public discussion is a political duty; and that this should be a fundamental principle of the American government. They recognized the risk to which all human institutions are subject. But they knew that order cannot be secured merely through fear of punishment for its infraction; that it is hazardous to discourage thought, hope and imagination; that fear breeds repression; that repression breeds hate; that hate menaces stable government; that the path of safety lies in the opportunity to discuss freely supposed grievances and proposed remedies; and that the fitting remedy for evil counsel is good ones. Believing in the power of reason as applied through public discussion, they eschewed silence coerced by lawthe argument of force in its worst form. ... Thus we consider this case against the background of a profound national commitment to the principle that debate on public issues should be uninhibited, robust, and wide open, and that it may well include vehement, caustic, and sometimes unpleasantly sharp attacks on government and public officials. ... (at pp. 700-701) Shunting aside the individual liability of Mr. Luis Beltran, is there a prima facie showing that Messrs. Maximo Soliven, Antonio V. Roces, Frederick K. Agcaoili, and Godofredo L. Manzanas knowingly participated in a wilful purveying of falsehood? Considering the free speech aspects of these petitions, should not a differentiated approach to their particular liabilities be taken instead of lumping up everybody with the offending columnist? I realize that the law includes publishers and editors but perhaps the "chilling effect" issue applies with singular effectivity to publishers and editors vis-a-vis newspaper columnists. There is no question that, ordinarily, libel is not protected by the free speech clause but we have to understand that some provocative words, which if taken literally may appear to shame or disparage a public figure, may really be intended to provoke debate on public issues when uttered or written by a media personality. Will not a criminal prosecution in the type of case now before us dampen the vigor and limit the variety of public debate? There are many other questions arising from this unusual case which have not been considered.

I, of course, concur with the Court's opinion because it has decided to limit the issues to narrowly drawn ones. I see no reason to disagree with the way the Court has resolved them. The first issue on prematurity is moot. The second issue discusses a procedure now embodied in the recently amended Rules of Court on how a Judge should proceed before he issues a warrant of arrest. Anent the third issue, considerations of public policy dictate that an incumbent President should not be sued. At the same time, the President cannot stand by helplessly bereft of legal remedies if somebody vilifies or maligns him or her. The Court has decided to defer the "chilling effect" issue for a later day. To this, I take exception. I know that most of our fiscals and judges are courageous individuals who would not allow any considerations of possible consequences to their careers to stand in the way of public duty. But why should we subject them to this problem? And why should we allow the possibility of the trial court treating and deciding the case as one for ordinary libel without bothering to fully explore the more important areas of concern, the extremely difficult issues involving government power and freedom of expression. However, since we have decided to defer the "chilling effect" issue for a later day, I limit myself to reiterating the dissenting words of Mr. Justice Jackson in the American case of Beaurnhais v. Illinois (343 U. S. 250) when he said: If one can claim to announce the judgment of legal history on any subject, it is that criminal libel laws are consistent with the concept of ordered liberty only when applied with safeguards evolved to prevent their invasion of freedom of expression. In the trial of the libel case against the petitioners, the safeguards in the name of freedom of expression should be faithfully applied.

G.R. No. 81756 October 21, 1991 NICOMEDES SILVA @ " Comedes", MARLON SILVA, @ "Tama" and ANTONIETA SILVA, petitioners, vs. THE HONORABLE PRESIDING JUDGE, REGIONAL TRIAL COURT OF NEGROS ORIENTAL, BRANCH XXXIII, DUMAGUETE CITY, respondent. Marcelo G. Flores for petitioners.

FERNAN, C.J.:p In this special civil action for certiorari, petitioners seek the nullification of Search Warrant No. 1 issued by respondent Judge as well as the return of the money in the amount of P1,231.00 seized from petitioner Antonieta Silva. The antecedent facts are as follows: On June 13, 1986, M/Sgt. Ranulfo Villamor, Jr., as chief of the PC Narcom Detachment in Dumaguete City, Negros Oriental, filed an "Application for Search Warrant" with the Regional Trial Court, Branch XXXIII, Dumaguete City against petitioners Nicomedes Silva and Marlon Silva. 1 This application was accompanied by a "Deposition of Witness" executed by Pfc. Arthur M. Alcoran and Pat. Leon T. Quindo, also dated June 13, 1986. 2 On the same day. Judge Nickarter A. Ontal, then Presiding Judge of the Regional Trial Court, Branch XXXIII, Dumaguete City, pursuant to the said "Application for Search Warrant" and "Deposition of Witness", issued Search Warrant No. 1, directing the aforesaid police officers to search the room of Marlon Silva in the residence of Nicomedes Silva for violation of Republic Act No. 6425, otherwise known as the Dangerous Drugs Act of 1972. as amended. Pertinent portions of Search Warrant No. 1 read as follows: It appearing to the satisfaction of the undersigned after examining oath (sic) MSGT. Ranulfo T. Villamor, Jr. and his witnesses (sic) Pfc. Arthur M. Alcoran and Pat. Leon T. Quindo that there is probable cause to believe that possession and control of Marijuana dried leaves, cigarettes, joint has been committed or is about to be committed and that there are good and sufficient reasons to believe that marijuana dried leaves, cigarettes, joint has in possession and/or control at Tama's Room (Rgt. side lst Floor) located at Nono-Limbaga Drive, Tanjay, Neg. Or. which is/are: X (Subject of the offense stated above (Stolen or embezzled or other proceeds of fruits of the offense;

X (Used or intended to be used as means of committing an offense. You are hereby commanded to make an immediate search at any time of the day (night) of the room of Tama Silva residence of his father Comedes Silva to open (sic) aparadors, lockers, cabinets, cartoons, containers, forthwith seize and take possession of the following property Marijuana dried leaves, cigarettes, joint and bring the said property to the undersigned to be dealt with as the law directs. 3 In the course of the search, the serving officers also seized money belonging to Antonieta Silva in the amount of P1,231.40. On June 16, 1986, Antonieta Silva filed a motion for the return of the said amount on the grounds that the search warrant only authorized the serving officers to seize marijuana dried leaves, cigarettes and joint, and that said officers failed or refused to make a return of the said search warrant in gross violation of Section 11, Rule 126 of the Rules of Court. 4 Acting on said motion, Judge Ontal issued an Order dated July 1, 1986, stating that the court "holds in abeyance the disposition of the said amount of P1,231.40 pending the filing of appropriate charges in connection with the search warrant." 5 On July 28, 1987, petitioners filed a motion to quash Search Warrant No. 1 on the grounds that (1) it was issued on the sole basis of a mimeographed "Application for Search Warrant" and "Deposition of Witness", which were accomplished by merely filling in the blanks and (2) the judge failed to personally examine the complainant and witnesses by searching questions and answers in violation of Section 3, Rule 126 of the Rules of Court. 6 On August 11, 1987, respondent trial court, through Judge Eugenio M. Cruz, who, by then, had replaced retired Judge Ontal, issued an Order denying the motion for lack of merit, finding the requisites necessary for the issuance of a valid search warrant duly complied with. 7 A motion for reconsideration dated September 1, 1987 filed by petitioners was likewise denied by Judge Cruz in an order dated October 19, 1987. Hence, this special civil action for certiorari. Petitioners allege that the issuance of Search Warrant No. 1 was tainted with illegality and that respondent Judge should be viewed to have acted without or in excess of jurisdiction, or committed grave abuse of discretion amounting to lack of jurisdiction when he issued the Order dated August 11, 1987, denying their motion to quash Search Warrant No, 1. We rule for petitioners.

Section 2, Article III (Bill of Rights) of the 1987 Constitution guarantees the right to personal liberty and security of homes against unreasonable searches and seizures. This section provides: Sec. 2. The right of the people to be secure in their persons, houses, papers, and effects against unreasonable searches and seizures of whatever nature and for any purpose shall be inviolable, and no search warrant or warrant of arrest shall issue except upon probable cause to be determined personally by the judge after examination under oath or affirmation of the complainant and the witnesses he may produce, and particularly describing the place to be searched and the persons or things to be seized. The purpose of the constitutional provision against unlawful searches and seizures is to prevent violations of private security in person and property, and unlawful invasion of the sanctity of the home, by officers of the law acting under legislative or judicial sanction, and to give remedy against such usurpations when attempted. 8 Thus, Sections 3 and 4, Rule 126 of the Rules of Court provide for the requisites for the issuance of a search warrant, to wit: SEC. 3. Requisite for issuing search warrant. A search warrant shall not issue but upon probable cause in connection with one specific offense to be determined personally by the judge after examination under oath or affirmation of the complainant and the witnesses he may produce, and particularly describing the place to be searched and the things to be seized. SEC. 4. Examination of complainant; record. The judge must, before issuing the warrant, personally examine in the form of searching questions and answers, in writing and under oath the complainant and any witnesses he may produce on facts personally known to them and attach to the record their sworn statements together with any affidavits submitted. Based on the aforecited constitutional and statutory provisions, the judge must, before issuing a search warrant, determine whether there is probable cause by examining the complainant and witnesses through searching questions and answers. In the case of Prudente vs. Dayrit, G.R. No. 82870, December 14, 1989, 180 SCRA 69, 767 this Court defined "probable cause" as follows: The "probable cause" for a valid search warrant, has been defined "as such facts and circumstances which would lead a reasonably discreet and prudent man to believe that an offense has been committed, and that objects sought in connection with the offense are in the place sought to be searched". This probable cause must be shown to be within the personal

knowledge of the complainant or the witnesses he may produce and not based on mere hearsay. In the case at bar, we have carefully examined the questioned search warrant as well as the "Application for Search Warrant" and "Deposition of Witness", and found that Judge Ontal failed to comply with the legal requirement that he must examine the applicant and his witnesses in the form of searching questions and answers in order to determine the existence of probable cause. The joint "Deposition of Witness" executed by Pfc. Alcoran and Pat. Quindo, which was submitted together with the "Application for Search Warrant" contained, for the most part suggestive questions answerable by merely placing "yes" or "no" in the blanks provided thereon. In fact there were only four (4) questions asked, to wit: Q Do you personally know M/Sgt. Ranulfo Villamor, Jr. the applicant for a search warrant? A Yes, sir. Q Do you have personal knowledge that the said premises subject of the offense stated above, and other proceeds of fruit of the offense, used or obtain (sic) or intended to be used as means of committing an offense? A Yes, sir. Q Do you know personally who is/are the person who has/have the property in his/their possession and control? A Yes, sir. Q How did you know all this (sic) things? A Through discreet surveillance. 9 The above deposition did not only contain leading questions but it was also very broad. The questions propounded to the witnesses were in fact, not probing but were merely routinary. The deposition was already mimeogragphed and all that the witnesses had to do was fill in their answers on the blanks provided. In the case of Nolasco vs. Pao, G.R. No. 69803, October 8, 1985, 139 SCRA 152, 163, this Court held: The "probable cause" required to justify the issuance of a search warrant comprehends such facts and circumstances as will induce a cautious man to rely upon them and act in pursuant thereof. Of the 8 questions asked, the 1st, 2nd and 4th pertain to identity. The 3rd and 5th are leading not searching questions. The 6th, 7th and 8th refer to the description of the personalities to be seized,

which is identical to that in the Search Warrant and suffers from the same lack of particularity. The examination conducted was general in nature and merely repetitious of the deposition of said witness. Mere generalization will not suffice and does not satisfy the requirements or probable cause upon which a warrant may issue. Likewise, in the Prudente case cited earlier, this Court declared the search warrant issued as invalid due to the failure of the judge to examine the witness in the form of searching questions and answers. Pertinent portion of the decision reads: Moreover, a perusal of the deposition of P/Lt. Florencio Angeles shows that it was too brief and short. Respondent Judge did not examine him "in the form of searching questions and answers". On the contrary, the questions asked were leading as they called for a simple "yes" or "no" answer. As held in Quintero vs. NBI, "the questions propounded by respondent Executive Judge to the applicant's witness' are not sufficiently searching to establish probable cause. Asking of leading questions to the deponent in an application for search warrant, and conducting of examination in a general manner, would not satisfy the requirements for issuance of a valid search warrant. 10 Thus, in issuing a search warrant, the judge must strictly comply with the constitutional and statutory requirement that he must determine the existence of probable cause by personally examining the applicant and his witnesses in the form of searching questions and answers. His failure to comply with this requirement constitutes grave abuse of discretion. As declared in Marcelo vs. De Guzman, G.R. No. L-29077, June 29, 1982, 114 SCRA 657, "the capricious disregard by the judge in not complying with the requirements before issuance of search warrants constitutes abuse of discretion". The officers implementing the search warrant clearly abused their authority when they seized the money of Antonieta Silva. This is highly irregular considering that Antonieta Silva was not even named as one of the respondents, that the warrant did not indicate the seizure of money but only of marijuana leaves, cigarettes and joints, and that the search warrant was issued for the seizure of personal property (a) subject of the offense and (b) used or intended to be used as means of committing an offense and NOT for personal property stolen or embezzled or other proceeds of fruits of the offense. Thus, the then presiding Judge Ontal likewise abused his discretion when he rejected the motion of petitioner Antonieta Silva seeking the return of her seized money. WHEREFORE, the petition is granted. Search Warrant No. 1 is hereby declared null and void. Respondent Judge of the Regional Trial Court of Negros Oriental, Branch XXXIII is directed to order the return to petitioner Antonieta Silva of the amount of P1,231.40 which had earlier been seized from her by virtue of the illegal search warrant. This decision is immediately executory. No costs.

SO ORDERED.

G.R. No. L-22196

June 30, 1967

ESTEBAN MORANO, CHAN SAU WAH and FU YAN FUN, petitionersappellants, vs. HON. MARTINIANO VIVO in his capacity as Acting Commissioner of Immigration, respondent-appellant. Engracio Fabre Law Office for petitioners-appellants. Office of the Solicitor General Arturo A. Alafriz and Solicitor A. M. Amores for respondent-appellant. SANCHEZ, J.: Chan Sau Wah, a Chinese citizen born in Fukien, China on January 6, 1932, arrived in the Philippines on November 23, 1961 to visit her cousin, Samuel Lee Malaps. She left in mainland China two of her children by a first marriage: Fu Tse Haw and Fu Yan Kai With her was Fu Yan Fun, her minor son also by the first marriage, born in Hongkong on September 11, 1957. Chan Sau Wah and her minor son Fu Yan Fun were permitted only into the Philippines under a temporary visitor's visa for two (2) months and after they posted a cash bond of P4,000.00. On January 24, 1962, Chan Sau Wah married Esteban Morano, a native-born Filipino citizen. Born to this union on September 16, 1962 was Esteban Morano, Jr. To prolong their stay in the Philippines, Chan Sau Wah and Fu Yan Fun obtained several extensions. The last extension expired on September 10, 1962.1wph1.t In a letter dated August 31, 1962, the Commissioner of Immigration ordered Chan Sau Wah and her son, Fu Yan Fun, to leave the country on or before September 10, 1962 with a warning that upon failure so to do, he will issue a warrant for their arrest and will cause the confiscation of their bond. Instead of leaving the country, on September 10, 1962, Chan Sau Wah (with her husband Esteban Morano) and Fu Yan Fun petitioned the Court of First Instance of Manila for mandamus to compel the Commissioner of Immigration to cancel petitioners' Alien Certificates of Registration; prohibition to stop the Commissioner from issuing a warrant for their arrest, and preliminary injunction to restrain the Commissioner from confiscating their cash bond and from issuing warrants of arrest pending resolution of this case.1 The trial court, on November 3, 1962, issued the writ of preliminary injunction prayed for, upon a P2,000-bond. After trial and the stipulations of facts filed by the parties, the Court of First Instance rendered judgment, viz: IN VIEW OF ALL THE FOREGOING, judgment is hereby rendered as follows:

(a) Granting this petition for Mandamus and Prohibition with respect to petitioner CHAN SAU WAH, who is hereby declared a citizen of the Philippines; ordering the respondent to cancel her Alien Certificate of Registration and other immigration papers, upon the payment of proper dues; and declaring the preliminary injunction with respect to her permanent, prohibiting the respondent, his representatives or subordinates from arresting and/or deporting said petitioner; (b) Dismissing this petition with respect to petitioner FU YAN FUN, and dissolving the writ of preliminary injunction issued herein, restraining the respondent, his representatives or subordinates from arresting and/or deporting said petitioner; (c) Authorizing respondent Commissioner to forfeit the bond filed by herein petitioners CHAN SAU WAH and FU YAN FUN in the amount of P4,000.00; and (d) Denying, for lack of merit, the prayer to declare Sec. 37 (a) of the Philippine Immigration Act of 1940 unconstitutional; Without pronouncement, as to costs. Petitioners and respondent Commissioner both appealed. We will deal with the claims of both appellants in their proper sequence. 1. The Solicitor General's brief assails the trial court's declaration that Chan Sau Wah is a citizen of the Philippines. The court a quo took the position that "Chan Sau Wah became, by virtue of, and upon, her marriage to Esteban Morano, a natural-born Filipino, a Filipino citizen.2 Placed to the fore is paragraph 1, Section 15 of Commonwealth Act 473 [Revised Naturalization Act], which reads: Sec. 15. Effect of the naturalization on wife children. Any woman who is now or may hereafter be married to a citizen of the Philippines, and who might herself be lawfully naturalized shall be deemed a citizen of the Philippines. To apply this provision, two requisites must concur: (a) valid marriage of an alien woman to a citizen of the Philippines and (b) the alien woman herself might be lawfully naturalized. We may concede that the first requisite has been properly met. The validity of the marriage is presumed. But can the same be said of the second requisite? This question by all means is not new. In a series of cases, this Court has declared that the marriage of an alien woman to a Filipino citizen does not ipso facto make her a Filipino citizen.

She must satisfactorily show that she has all the qualifications and none of the disqualifications required by the Naturalization Law.3 Ly Giok Ha alias Wy Giok Ha et al. vs. Emilio Galang, L-21332, March 18, 1966,* clearly writes down the philosophy behind the rule in the following expressive language, viz: Reflection will reveal why this must be so. The qualifications prescribed under section 2 of the Naturalization Act, and the disqualifications enumerated in its section 4, are not mutually exclusive; and if all that were to be required is that the wife of a Filipino be not disqualified under section 4, the result might well be that citizenship would be conferred upon persons in violation of the policy of the statute. For example, section 4 disqualifies only "(c) Polygamists or believers in the practice of polygamy; and (b) Persons convicted of crimes involving moral turpitude," so that a blackmailer, or a maintainer of gambling or bawdy houses, not previously convicted by a competent court, would not be thereby disqualified; still it is certain that the law did not intend such a person to, be admitted as a citizen in view of the requirement of section 2 that an applicant for citizenship "must be of good moral character." Similarly, the citizen's wife might be a convinced believer in racial supremacy, in government by certain selected classes, in the right to vote exclusively by certain "herrenvolk," and thus disbelieve in the principles underlying the Philippine Constitution; yet she would not be disqualified under section 4, as long as she is not "opposed to organized government," nor affiliated to groups "upholding or teaching doctrines opposing all organized governments," nor "defending or teaching the necessity or propriety of violence, personal assault or assassination for the success or predominance of their ideas." Et sic de caeteris. Upon the principle of selective citizenship, we cannot afford to depart from the wise precept affirmed and reaffirmed in the cases heretofore noted. In the additional stipulation of facts of July 3, 1963, petitioners admit that Chan Sau Wah is not possessed of all the qualifications required by the Naturalization Law. Because of all these we are left under no doubt that petitioner Chan Sau Wah did not become a Filipino citizen. 2. Squarely put in issue by petitioners is the constitutionality of Section 37 (a) of the Immigration Act of 1940, which reads: Sec. 37. (a) The following aliens shall be arrested upon the warrant of the Commissioner of Immigration or of any other officer designated by him for the

purpose and deported upon the warrant of the Commissioner of Immigration after a determination by the Board of Commissioners of the existence of the ground for deportation as charged against the alien: xxx xxx xxx

(7) Any alien who remains in the Philippines in violation of any limitation or condition under which he was admitted as a nonimmigrant. Petitioners argue that the legal precept just quoted trenches upon the constitutional mandate in Section 1 (3), Article III [Bill of Rights] of the Constitution, to wit: (3) The right of the people to be secure in their persons, houses, papers, and effects against unreasonable searches and seizures shall not be violated, and no warrants shall issue but upon probable cause, to be determined by the judge after examination under oath or affirmation of the complainant and the witnesses he may produce, and particularly describing the place to be searched, and the persons or things to be seized. They say that the Constitution limits to judges the authority to issue warrants of arrest and that the legislative delegation of such power to the Commissioner of Immigration is thus violative of the Bill of Rights. Section 1 (3), Article III of the Constitution, we perceive, does not require judicial intervention in the execution of a final order of deportation issued in accordance with law. The constitutional limitation contemplates an order of arrest in the exercise of judicial power4 as a step preliminary or incidental to prosecution or proceedings for a given offense or administrative action, not as a measure indispensable to carry out a valid decision by a competent official, such as a legal order of deportation, issued by the Commissioner of Immigration, in pursuance of a valid legislation. The following from American Jurisprudence,5 is illuminating: It is thoroughly established that Congress has power to order the deportation of aliens whose presence in the country it deems hurtful. Owing to the nature of the proceeding, the deportation of an alien who is found in this country in violation of law is not a deprivation of liberty without due process of law. This is so, although the inquiry devolves upon executive officers, and their findings of fact, after a fair though summary hearing, are made conclusive. xxx xxx xxx

The determination of the propriety of deportation is not a prosecution for, or a conviction of, crime; nor is the deportation a punishment, even though the facts

underlying the decision may constitute a crime under local law. The proceeding is in effect simply a refusal by the government to harbor persons whom it does not want. The coincidence of local penal law with the policy of Congress is purely accidental, and, though supported by the same facts, a criminal prosecution and a proceeding for deportation are separate and independent. In consequence, the constitutional guarantee set forth in Section 1 (3), Article III of the Constitution aforesaid, requiring that the issue of probable cause be determined by a judge, does not extend to deportation proceedings.6 The view we here express finds support in the discussions during the constitutional convention. The convention recognized, as sanctioned by due process, possibilities and cases of deprivation of liberty, other than by order of a competent court.7 Indeed, the power to deport or expel aliens is an attribute of sovereignty. Such power is planted on the "accepted maxim of international law, that every sovereign nation has the power, as inherent in sovereignty, and essential to self-preservation, to forbid the entrance of foreigners within its dominions."8 So it is, that this Court once aptly remarked that there can be no controversy on the fact that where aliens are admitted as temporary visitors, "the law is to the effect that temporary visitors who do not depart upon the expiration of the period of stay granted them are subject to deportation by the Commissioner of Immigration, for having violated the limitation or condition under which they were admitted as non-immigrants (Immigration Law, Sec. 37 (a), subsection (7); C.A. 613, as amended)."9 And, in a case directly in point, where the power of the Commissioner to issue warrants of arrest was challenged as unconstitutional, because "such power is only vested in a judge by Section 1, paragraph 3, Article III of our Constitution," this Court declared This argument overlooks the fact that the stay of appellant Ng Hua To as temporary visitor is subject to certain contractual stipulations as contained in the cash bond put up by him, among them, that in case of breach the Commissioner may require the recommitment of the person in whose favor the bond has been filed. The Commissioner did nothing but to enforce such condition. Such a step is necessary to enable the Commissioner to prepare the ground for his deportation under section 37 (a) of Commonwealth Act 613. A contrary interpretation would render such power nugatory to the detriment of the State.10 It is in this context that we rule that Section 37 (a) of the Immigration Act of 1940 is not constitutionally proscribed. 3. A sequel to the questions just discussed is the second error set forth in the government's brief. The Solicitor General balks at the lower court's ruling that

petitioner Chan Sau Wah is entitled to permanent residence in the Philippines without first complying with the requirements of Sections 9 and 13 of the Immigration Act of 1940, as amended by Republic Act 503. We first go to the law, viz: SEC. 9 [last paragraph] An alien who is admitted as a nonimmigrant cannot remain in the Philippines permanently. To obtain permanent admission, a nonimmigrant alien must depart voluntarily to some foreign country and procure from the appropriate Philippine consul the proper visa and thereafter undergo examination by the officers of the Bureau of Immigration at a Philippine port of entry for determination of his admissibility in accordance with the requirements of this Act. SEC. 13. Under the conditions set forth in this Act there may be admitted into the Philippines immigrants, termed "quota immigrants" not in excess of fifty (50) of any one nationality or without nationality for any one calendar year, except that the following immigrants, termed "nonquota immigrants," maybe admitted without regard to such numerical limitations. The corresponding Philippine Consular representative abroad shall investigate and certify the eligibility of a quota immigrant previous to his admission into the Philippines. Qualified and desirable aliens who are in the Philippines under temporary stay may be admitted within the quota, subject to the provisions of the last paragraph of section 9 of this Act. (a) The wife or the husband or the unmarried child under twenty-one years of age of a Philippine citizen, if accompanying or following to join such citizen; (b) A child of alien parents born during the temporary visit abroad of the mother, the mother having been previously lawfully admitted into the Philippine for permanent residence, if the child is accompanying or coming to join a parent and applies for admission within five years from the date of its birth; Concededly, Chan Sau Wah entered the Philippines on a tourist-temporary visitor's visa. She is a non-immigrant. Under Section 13 just quoted, she may therefore be admitted if she were a qualified and desirable alien and subject to the provisions of the last paragraph of Section 9. Therefore, first, she must depart voluntarily to some foreign country; second, she must procure from the appropriate consul the proper visa; and third, she must thereafter undergo examination by the officials of the Bureau of Immigration at the port of entry for determination of her admissibility in accordance with the requirements of the immigration Act. This Court in a number of cases has ruled, and consistently too, that an alien

admitted as a temporary visitor cannot change his or her status without first departing from the country and complying with the requirements of Section 9 of the Immigration Act. 11 The gravamen of petitioners' argument is that Chan Sau Wah has, since her entry, married in Manila a native-born Filipino, Esteban Morano. It will not particularly help analysis for petitioners to appeal to family solidarity in an effort to thwart her deportation. Chan Sau Wah, seemingly is not one who has a high regard for such solidarity. Proof: She left two of her children by the first marriage, both minors, in the care of neighbors in Fukien, China. Then, the wording of the statute heretofore adverted to is a forbidding obstacle which will prevent this Court from writing into the law an additional provision that marriage of a temporary alien visitor to a Filipino would ipso facto make her a permanent resident in his country. This is a field closed to judicial action. No breadth of discretion is allowed us. We cannot insulate her from the State's power of deportation. Really, it would be an easy matter for an alien woman to enter the Philippines as a temporary visitor, go through a mock marriage, but actually live with another man as husband and wife, and thereby skirt the provisions of our immigration law. Also, a woman of undesirable character may enter this country, ply a pernicious trade, marry a Filipino, and again throw overboard Sections 9 and 13 of the Act. Such a flanking movement, we are confident, is impermissible. Recently we confirmed the rule that an alien wife of a Filipino may not stay permanently without first departing from the Philippines. Reason: Discourage entry under false pretenses. 12 The ruling of the trial court on this score should be reversed. 4. It is petitioners' turn to point as error the dismissal of the petition for mandamus and prohibition with respect to petitioner Fu Yan Fun. Petitioners' line of thought is this: Fu Yan Fun follows the citizenship of his mother. They cite Section 15, paragraph 3, Commonwealth Act 473, which says that: A foreign-born minor child, if dwelling in the Philippines at the time of the naturalization of the parent, shall automatically become a Philippine citizen. . . . Petitioners' position is based on the assumption that Chan Sau Wah, the mother, is a Filipino citizen. We have held that she is not. At best, Fu Yan Fun is a stepson of Esteban Morano, husband of Chan Sau Wah. A step-son is not a foreignborn child of the step-father. The word child, we are certain, means legitimate child, not a step-child. We are not wanting in precedents. Thus, when the

Constitution provides that "[t]hose whose fathers are citizens of the Philippines" are citizens thereof, 13 the fundamental charter intends "those" to apply to legitimate children. 14 In another case, the term "minor children" or "minor child" in Section 15 of the Revised Naturalization Law refers only to legitimate children of Filipino citizens. This Court, thru Mr. Chief Justice Roberto Concepcion, there said: 15 It is claimed that the phrases "minor children" and "minor child," used in these provisions, include adopted children. The argument is predicated upon the theory that an adopted child is, for all intents and purposes, a legitimate child. Whenever, the word "children" or "child" is used in statutes, it is generally understood, however, to refer to legitimate children, unless the context of the law and its spirit indicate clearly the contrary. Thus, for instance, when the Constitution provides that "those whose fathers are citizens of the Philippines," and "those whose mothers are citizens of the Philippines" who shall elect Philippine citizenship upon reaching the age of majority, are citizens of the Philippines (Article IV, Section 1, subdivisions [3] and [4]), our fundamental law clearly refers to legitimate children (Chiongbian vs. De Leon, 46 Off. Gaz., 36523654; Serra v. Republic, L-4223, May 12, 1952). At any rate, Fu Yan Fun entered the Philippines as a temporary visitor. The status of a temporary visitor cannot be converted into, that of a permanent resident, as we have heretofore held, without first complying with Section 9 of the Immigration Law. 5. Petitioners finally aver that the lower court erred in authorizing respondent Commissioner to forfeit the bond filed by petitioners Chan Sau Wah and Fu Yan Fun in the amount of P4,000.00. Here is petitioners' posture. They enjoyed their stay in the Philippines upon a bond. Now they come to court and say that as the prescribed form of this bond was not expressly approved by the Secretary of Justice in accordance with Section 3 of Commonwealth Act 613, which reads SEC. 3. . . . He [Commissioner of Immigration] shall issue, subject to the approval of the Department Head, such rules and regulations and prescribes such forms of bond, reports, and other papers, and shall issue from time to time such instruction, not inconsistent with law, as he shall deem best calculated to carry out the provisions of the immigration laws. . . . that bond is void. Reasons there are which prevent us from giving our imprimatur to this argument. The provision requiring official approval of a bond is merely directory. "Irregularity or entire failure in this respect does not affect the validity of the bond. 16 The

reason for the rule, is found in 9 C.J., p. 26 (footnote), which reads: (a) Reason for rule. "Statutes requiring bonds to be approved by certain officials are not for the purpose of protecting the obligors in the bond, but are aimed to protect the public, to insure their solvency, and to create evidence of an unimpeachable character of the fact of their execution. When they are executed for a legal purpose, before a proper tribunal, and are in fact accepted and approved by the officer or body, whose duty it was to approve them, it could serve no useful purpose of the law to hold them invalid, to release all the obligors thereon, and to defeat every purpose of its execution, simply because the fact of approval was not indorsed precisely as had been directed by the Legislature." American Book Co. vs. Wells, 83 SW 622, 627, 26 Ky L-1159. (emphasis supplied) And another. This bond was accepted by the government. It had been there. The form of the bond here used is of long continued usage. If the government did not question the form of the bond at all, then we must assume that it counted with the Secretary's approval. For the presumption is that official duty has been legally performed. Surely enough, equitable considerations will stop petitioners from pleading invalidity of the bond. They offered that bond to enable them to enter and stay in this country. They enjoyed benefits therefrom. They cannot, "in law, and good conscience, be allowed to reap the fruits" of that bond, and then jettison the same. They are "precluded from attacking the validity" of such bond. 17 Actually, to petitioners the bond was good while they sought entry into the Philippines; they offered it as security for the undertaking; that they "will actually depart from the Philippines" when their term of stay expires. Now that the bond is being confiscated because they overstayed, they make an about-face and say that such bond is null and void. They shall not profit from this inconsistent position. Their bond should be confiscated. Conformably to the foregoing, the judgment under review is hereby modified as follows: (1) The portion thereof which reads: (a) Granting their petition for Mandamus and Prohibition with respect to petitioner CHAN SAU WAH, who is hereby declared a citizen of the Philippines; ordering the respondent to cancel her Alien Certificate of Registration and other immigration papers, upon the payment of proper dues; and declaring preliminary injunction with respect to her permanent, prohibiting the respondent, his representatives or subordinates from arresting and/or deporting said petitioner; is hereby reversed: and, in consequence

The petition for mandamus and prohibition with respect to petitioner Chan Sau Wah is hereby denied; and the judgment declaring her a citizen of the Philippines, directing respondent to cancel her Alien Certificate of Registration and other immigration papers, and declaring the preliminary injunction with respect to her permanent, are all hereby set aside; and (2) In all other respects, the decision appealed from is hereby affirmed. No costs. So ordered. Concepcion, C.J., Reyes, J.B.L., Makalintal, Bengzon J.P., Zaldivar and Castro, J.J., concur. Separate Opinions DIZON, J., concurring: I concur (in the result) with the majority opinion penned by Mr. Justice Conrado Sanchez, for the reason that, as stated therein, "In the additional stipulation of facts of July 3, 1963, petitioners admit that Chan Sau Wah is not possessed of all the qualifications required by the Naturalization Law."

G.R. No. 82544 June 28, 1988 IN THE MATTER OF THE PETITION FOR HABEAS CORPUS OF: ANDREW HARVEY, JOHN SHERMAN and ADRIAAN VAN DEL ELSHOUT, petitioners, vs. HONORABLE COMMISSIONER MIRIAM DEFENSOR SANTIAGO, COMMISSION ON IMMIGRATION AND DEPORTATION, respondent.

MELENCIO-HERRERA, J.: A petition for Habeas Corpus. Petitioners Andrew Harvey and John Sherman, 52 and 72 years, respectively, are both American nationals residing at Pagsanjan, Laguna, while Adriaan Van Elshout, 58 years old, is a Dutch citizen also residing at Pagsanjan, Laguna. The case stems from the apprehension of petitioners on 27 February 1988 from their respective residences by agents of the Commission on Immigration and Deportation (CID) by virtue of Mission Orders issued by respondent Commissioner Miriam Defensor Santiago of the CID. Petitioners are presently detained at the CID Detention Center. Petitioners were among the twenty-two (22) suspected alien pedophiles who were apprehended after three months of close surveillance by CID agents in Pagsanjan, Laguna. Two (2) days after apprehension, or on 29 February 1988, seventeen (17) of the twenty-two (22) arrested aliens opted for self-deportation and have left the country. One was released for lack of evidence; another was charged not for being a pedophile but for working without a valid working visa. Thus, of the original twenty two (22), only the three petitioners have chosen to face deportation. Seized during petitioners apprehension were rolls of photo negatives and photos of the suspected child prostitutes shown in salacious poses as well as boys and girls engaged in the sex act. There were also posters and other literature advertising the child prostitutes. The "Operation Report," on Andrew Harvey and Richard Sherman dated 29 February 1988 stated: xxx xxx xxx ANDREW MARK HARVEY was found together with two young boys. RICHARD SHERMAN was found with two naked boys inside his room.

In respect of Van Den Elshout the "After Mission Report," dated 27 February 1988 read in part: Noted: There were two (2) children ages 14 & 16 which subject readily accepted having been in his care and live-in for quite sometime. On 4 March 1988, deportation proceedings were instituted against petitioners for being undesirable aliens under Section 69 of the Revised Administrative Code (Deportation Case No. 88-13). The "Charge Sheet" read inter alia: Wherefore, this Office charges the respondents for deportation, as undesirable aliens, in that: they, being pedophiles, are inimical to public morals, public health and public safety as provided in Section 69 of the Revised Administrative Code. On 7 March 1988, Warrants of Arrest were issued by respondent against petitioners for violation of Sections 37, 45 and 46 of the Immigration Act and Section 69 of the Revised Administrative Code On the same date, the Board of Special Inquiry III commenced trial against petitioners. On 14 March 1988, petitioners filed an Urgent Petition for Release Under Bond alleging that their health was being seriously affected by their continuous detention. Upon recommendation of the Board of Commissioners for their provisional release, respondent ordered the CID doctor to examine petitioners, who certified that petitioners were healthy. On 22 March 1988, petitioners filed a Petition for Bail which, however, respondent denied considering the certification by the CID physician that petitioners were healthy. To avoid congestion, respondent ordered petitioners' transfer to the CID detention cell at Fort Bonifacio, but the transfer was deferred pending trial due to the difficulty of transporting them to and from the CID where trial was on-going. On 4 April 1988 petitioner Andrew Harvey filed a Manifestation/Motion stating that he had "finally agreed to a self-deportation" and praying that he be "provisionally released for at least 15 days and placed under the custody of Atty. Asinas before he voluntarily departs the country." On 7 April 1988, the Board of Special Inquiry III allowed provisional release of five (5) days only under certain conditions. However, it appears that on the same date that the aforesaid Manifestation/ Motion was filed, Harvey and his co-petitioners had already filed the present petition. On 4 April 1988, as heretofore stated, petitioners availed of this Petition for a Writ of Habeas Corpus. A Return of the Writ was filed by the Solicitor General and the Court heard the case on oral argument on 20 April 1988. A Traverse to the Writ

was presented by petitioners to which a Reply was filed by the Solicitor General. Petitioners question the validity of their detention on the following grounds: 1) There is no provision in the Philippine Immigration Act of 1940 nor under Section 69 of the Revised Administrative Code, which legally clothes the Commissioner with any authority to arrest and detain petitioners pending determination of the existence of a probable cause leading to an administrative investigation. 2) Respondent violated Section 2, Article III of the 1987 Constitution prohibiting unreasonable searches and seizures since the CID agents were not clothed with valid Warrants of arrest, search and seizure as required by the said provision. 3) Mere confidential information made to the CID agents and their suspicion of the activities of petitioners that they are pedophiles, coupled with their association with other suspected pedophiles, are not valid legal grounds for their arrest and detention unless they are caught in the act. They further allege that being a pedophile is not punishable by any Philippine Law nor is it a crime to be a pedophile. We reject petitioners' contentions and uphold respondent's official acts ably defended by the Solicitor General. There can be no question that the right against unreasonable searches and seizures guaranteed by Article III, Section 2 of the 1987 Constitution, is available to all persons, including aliens, whether accused of crime or not (Moncado vs. People's Court, 80 Phil. 1 [1948]. One of the constitutional requirements of a valid search warrant or warrant of arrest is that it must be based upon probable cause. Probable cause has been defined as referring to "such facts and circumstances antecedent to the issuance of the warrant that in themselves are sufficient to induce a cautious man to rely on them and act in pursuance thereof." (People vs. Syjuco 64 Phil. 667 [1937]; Alverez vs. CFI, 64 Phil. 33 [1937]). The 1985 Rules on Criminal Procedure also provide that an arrest wit a warrant may be effected by a peace officer or even a private person (1) when such person has committed, actually committing, or is attempting to commit an offense in his presence; and (2) when an offense has, in fact, been committed and he has personal knowledge of facts indicating that the person to be arrested has committed it (Rule 113, Section 5). In this case, the arrest of petitioners was based on probable cause determined after close surveillance for three (3) months during which period their activities were monitored. The existence of probable cause justified the arrest and the seizure of the photo negatives, photographs and posters without warrant (See

Papa vs. Mago, L-27360, February 28, 1968,22 SCRA 857; People vs. Court of First Instance of Rizal, L-41686, November 17, 1980, 101 SCRA 86, cited in CRUZ, Constitutional Law, 1987 ed., p. 143). Those articles were seized as an incident to a lawful arrest and, are therefore, admissible in evidence (Section 12, Rule 126,1985 Rules on criminal Procedure). But even assuming arguendo that the arrest of petitioners was not valid at its inception, the records show that formal deportation charges have been filed against them, as undesirable aliens, on 4 March 1988. Warrants of arrest were issued against them on 7 March 1988 "for violation of Section 37, 45 and 46 of the Immigration Act and Section 69 of the Administrative Code." A hearing is presently being conducted by a Board of Special Inquiry. The restraint against their persons, therefore, has become legal. The Writ has served its purpose. The process of the law is being followed (Cruz vs. Montoya, L-39823, February 25, 1975, 62 SCRA 543). "were a person's detention was later made by virtue of a judicial order in relation to criminal cases subsequently filed against the detainee, his petition for habeas corpus becomes moot and academic" (Beltran vs. Garcia, L-49014, April 30, 1979, 89 SCRA 717). "It is a fundamental rule that a writ of habeas corpus will not be granted when the confinement is or has become legal, although such confinement was illegal at the beginning" (Matsura vs. Director of Prisons, 77 Phil. 1050 [1947]). That petitioners were not "caught in the act" does not make their arrest illegal. Petitioners were found with young boys in their respective rooms, the ones with John Sherman being naked. Under those circumstances the CID agents had reasonable grounds to believe that petitioners had committed "pedophilia" defined as "psychosexual perversion involving children" (Kraft-Ebbing Psychopatia Sexualis p. 555; Paraphilia (or unusual sexual activity) in which children are the preferred sexual object" (Webster's Third New International Dictionary, 1971 ed., p. 1665) [Solicitor General's Return of the Writ, on p. 101. While not a crime under the Revised Penal Code, it is behavior offensive to public morals and violative of the declared policy of the State to promote and protect the physical, moral, spiritual, and social well-being of our youth (Article II, Section 13, 1987 Constitution). At any rate, the filing by petitioners of a petition to be released on bail should be considered as a waiver of any irregularity attending their arrest and estops them from questioning its validity (Callanta v. Villanueva, L-24646 & L-24674, June 20, 1977, 77 SCRA 377; Bagcal vs. Villaraza, L-61770, January 31, 1983, 120 SCRA 525). The deportation charges instituted by respondent Commissioner are in accordance with Section 37(a) of the Philippine Immigration Act of 1940, in relation to Section 69 of the Revised Administrative Code. Section 37(a) provides in part:

(a) The following aliens shall be arrested upon the warrant of the Commissioner of Immigration and Deportation or any other officer designated by him for the purpose and deported upon the warrant of the Commissioner of Immigration and Deportation after a determination by the Board of Commissioners of the existence of the ground for deportation as charged against the alien; xxx xxx xxx The foregoing provision should be construed in its entirety in view of the summary and indivisible nature of a deportation proceeding, otherwise, the very purpose of deportation proceeding would be defeated. Section 37(a) is not constitutionally proscribed (Morano vs. Vivo, L-22196, June 30, 1967, 20 SCRA 562). The specific constraints in both the 1935 1 and 1987 2 Constitutions, which are substantially Identical, contemplate prosecutions essentially criminal in nature. Deportation proceedings, on the other hand, are administrative in character. An order of deportation is never construed as a punishment. It is preventive, not a penal process. It need not be conducted strictly in accordance with ordinary Court proceedings. It is of course well-settled that deportation proceedings do not constitute a criminal action. The order of deportation is not a punishment, (Maliler vs. Eby, 264 U.S., 32), it being merely the return to his country of an alien who has broken the conditions upon which he could continue to reside within our borders (U.S. vs. De los Santos, 33 Phil., 397). The deportation proceedings are administrative in character, (Kessler vs. Stracker 307 U.S., 22) summary in nature, and need not be conducted strictly in accordance with the ordinary court proceedings (Murdock vs. Clark, 53 F. [2d], 155). It is essential, however, that the warrant of arrest shall give the alien sufficient information about the charges against him, relating the facts relied upon. (U.S. vs. Uhl 211 F., 628.) It is also essential that he be given a fair hearing with the assistance of counsel, if he so desires, before unprejudiced investigators (Strench vs. Pedaris, 55 F. [2d], 597; Ex parte Jew You On, 16 F. [2d], 153). However, all the strict rules of evidence governing judicial controversies do not need to be observed; only such as are fundamental and essential like the right of cross-examination. (U.S. vs. Hughes, 104 F. [2d], 14; Murdock vs. Clark, 53 F. [2d], 155.) Hearsay evidence may even be admitted, provided the alien is given the opportunity to explain or rebut it (Morrell vs. Baker, 270 F., 577; Sercerchi vs. Ward, 27 F. Supp., 437). (Lao Tang Bun vs. Fabre 81 Phil. 682 [1948]). The ruling in Vivo vs. Montesa (G. R. No. 24576, July 29, 1968, 24 SCRA 155) that "the issuance of warrants of arrest by the Commissioner of Immigration, solely for purposes of investigation and before a final order of deportation is issued, conflicts with paragraph 3, Section I of Article III of the Constitution" (referring to the 1935 Constitution) 3 is not invocable herein. Respondent Commissioner's Warrant of Arrest issued on 7 March 1988 did not order

petitioners to appear and show cause why they should not be deported. They were issued specifically "for violation of Sections 37, 45 and 46 of the Immigration Act and Section 69 of the Revised Administrative Code." Before that, deportation proceedings had been commenced against them as undesirable aliens on 4 March 1988 and the arrest was a step preliminary to their possible deportation. Section 37 of the Immigration Law, which empowers the Commissioner of Immigration to issue warrants for the arrest of overstaying aliens is constitutional. The arrest is a stop preliminary to the deportation of the aliens who had violated the condition of their stay in this country. (Morano vs. Vivo, L-22196, June 30, 1967, 20 SCRA 562). To rule otherwise would be to render the authority given the Commissioner nugatory to the detriment of the State. The pertinent provision of Commonwealth Act No. 613, as amended, which gives authority to the Commissioner of Immigration to order the arrest of an alien temporary visitor preparatory to his deportation for failure to put up new bonds required for the stay, is not unconstitutional. xxx xxx xxx ... Such a step is necessary to enable the Commissioner to prepare the ground for his deportation under Section 37[al of Commonwealth Act 613. A contrary interpretation would render such power nugatory to the detriment of the State. (Ng Hua To vs. Galang, G. R. No. 10145, February 29, 1964, 10 SCRA 411). "The requirement of probable cause, to be determined by a Judge, does not extend to deportation proceedings." (Morano vs. Vivo, supra, citing Tiu Chun Hai vs. Commissioner, infra). There need be no "truncated" recourse to both judicial and administrative warrants in a single deportation proceedings. The foregoing does not deviate from the ruling in Qua Chee Gan vs. Deportation Board (G. R. No. 10280, September 30, 1963, 9 SCRA 27 [1963]) reiterated in Vivo vs. Montesa, supra, that "under the express terms of our Constitution (the 1935 Constitution), it is therefore even doubtful whether the arrest of an individual may be ordered by any authority other than a judge if the purpose is merely to determine the existence of a probable cause, leading to an administrative investigation." For, as heretofore stated, probable cause had already been shown to exist before the warrants of arrest were issued. What is essential is that there should be a specific charge against the alien intended to be arrested and deported, that a fair hearing be conducted (Section 37[c]) with the assistance of counsel, if desired, and that the charge be substantiated by competent evidence. Thus, Section 69 of the Revised

Administrative Code explicitly provides: Sec. 69. Deportation of subject of foreign power. A subject of a foreign power residing in the Philippines shall not be deported, expelled, or excluded from said Islands or repatriated to his own country by the President of the Philippines except upon prior investigation, conducted by said Executive or his authorized agent, of the ground upon which such action is contemplated. In such a case the person concerned shall be informed of the charge or charges against him and he shall be allowed not less than 3 days for the preparation of his defense. He shall also have the right to be heard by himself or counsel, to produce witnesses in his own behalf, and to cross-examine the opposing witnesses. The denial by respondent Commissioner of petitioners' release on bail, also challenged by them, was in order because in deportation proceedings, the right to bail is not a matter of right but a matter of discretion on the part of the Commissioner of Immigration and Deportation. Thus, Section 37(e) of the Philippine Immigration Act of 1940 provides that "any alien under arrest in a deportation proceeding may be released under bond or under such other conditions as may be imposed by the Commissioner of Immigration." The use of the word "may" in said provision indicates that the grant of bail is merely permissive and not mandatory on the part of the Commissioner. The exercise of the power is wholly discretionary (Ong Hee Sang vs. Commissioner of Immigration, L-9700, February 28,1962, 4 SCRA 442). "Neither the Constitution nor Section 69 of the Revised Administrative Code guarantees the right of aliens facing deportation to provisional liberty on bail." (Tiu Chun Hai et al vs. Deportation Board, 104 Phil. 949 [1958]). As deportation proceedings do not partake of the nature of a criminal action, the constitutional guarantee to bail may not be invoked by aliens in said proceedings (Ong Hee Sang vs. Commissioner of Immigration, supra). Every sovereign power has the inherent power to exclude aliens from its territory upon such grounds as it may deem proper for its self-preservation or public interest (Lao Tan Bun vs. Fabre 81 Phil. 682 [1948]). The power to deport aliens is an act of State, an act done by or under the authority of the sovereign power (In re McCulloch Dick, 38 Phil. 41 [1918]). It is a police measure against undesirable aliens whose continued presence in the country is found to be injurious to the public good and the domestic tranquility of the people (Forbes vs. Chuoco Tiaco et al., 16 Phil. 534 [1910]). Particularly so in this case where the State has expressly committed itself to defend the tight of children to assistance and special protection from all forms of neglect, abuse, cruelty, exploitation, and other conditions prejudicial to their development (Article XV, Section 3[2]). Respondent Commissioner of Immigration and Deportation, in instituting deportation proceedings against petitioners, acted in the interests of the State. WHEREFORE, the Petition is dismissed and the Writ of Habeas Corpus is hereby denied.

SO ORDERED.

G.R. No. L-45358

January 29, 1937

NARCISO ALVAREZ, petitioner, vs. THE COURT OF FIRST INSTANCE OF TAYABAS and THE ANTI-USURY BOARD, respondents. Godofredo Reyes for petitioner. Adolfo N. Feliciano for respondents Anti-Usury Board. No appearance for other respondent. IMPERIAL, J.: The petitioner asks that the warrant of June 3, 1936, issued by the Court of First Instance of Tayabas, ordering the search of his house and the seizure, at any time of the day or night, of certain accounting books, documents and papers belonging to him in his residence situated in Infanta, Province of Tayabas, as well as the order of a later date, authorizing the agents of the Anti-Usury Board to retain the articles seized, be declared illegal and set aside, and prays that all the articles in question be returned to him. On the date above-mentioned, the chief of the secret service of the Anti-Usury Board, of the Department of Justice, presented to Judge Eduardo Gutierrez David then presiding over the Court of First Instance of Tayabas, an affidavit alleging that according to reliable information, the petitioner kept in his house in Infanta, Tayabas, books, documents, receipts, lists, chits and other papers used by him in connection with his activities as a money-lender charging usurious rates of interest in violation of the law. In his oath at the and of the affidavit, the chief of the secret service stated that his answers to the questions were correct to the best of his knowledge and belief. He did not swear to the truth of his statements upon his own knowledge of the facts but upon the information received by him from a reliable person. Upon the affidavit in question the Judge, on said date, issued the warrant which is the subject matter of the petition, ordering the search of the petitioner's house at nay time of the day or night, the seizure of the books and documents above-mentioned and the immediate delivery thereof to him to be disposed of in accordance with the law. With said warrant, several agents of the Anti-Usury Board entered the petitioner's store and residence at seven o'clock on the night of June 4, 1936, and seized and took possession of the following articles: internal revenue licenses for the years 1933 to 1936, one ledger, two journals, two cashbooks, nine order books, four notebooks, four checks stubs, two memorandums, three bankbooks, two contracts, four stubs, forty-eight stubs of purchases of copra, two inventories, two bundles of bills of lading, one bundle of credit receipts, one bundle of stubs of purchases of copra, two packages of correspondence, one receipt book belonging to Luis Fernandez, fourteen bundles of invoices and other papers many documents and loan contracts with security and promissory notes, 504 chits, promissory notes and stubs of used checks of the Hongkong & Shanghai Banking Corporation. The search for and a seizure of said articles were made with the opposition of the petitioner who stated his protest below the inventories on the ground that the agents seized even the originals of the documents. As the articles had not been brought immediately to the judge who issued the search warrant, the petitioner, through his attorney, filed a motion on June 8, 1936, praying that the agent Emilio L. Siongco, or any other agent, be ordered immediately to deposit all the seized articles in the office of the clerk of court and that said agent be declared guilty of

contempt for having disobeyed the order of the court. On said date the court issued an order directing Emilio L. Siongco to deposit all the articles seized within twenty-four hours from the receipt of notice thereof and giving him a period of five (5) days within which to show cause why he should not be punished for contempt of court. On June 10th, Attorney Arsenio Rodriguez, representing the Anti-Usury Board, filed a motion praying that the order of the 8th of said month be set aside and that the Anti-Usury Board be authorized to retain the articles seized for a period of thirty (30) days for the necessary investigation. The attorney for the petitioner, on June 20th, filed another motion alleging that, notwithstanding the order of the 8th of said month, the officials of the Anti-Usury Board had failed to deposit the articles seized by them and praying that a search warrant be issued, that the sheriff be ordered to take all the articles into his custody and deposit of the Anti-Usury Board be punished for contempt of court. Said attorney, on June 24th, filed an ex parte petition alleging that while agent Emilio L. Siongco had deposited some documents and papers in the office of the clerk of court, he had so far failed to file an inventory duly verified by oath of all the documents seized by him, to return the search warrant together with the affidavit it presented in support thereof, or to present the report of the proceedings taken by him; and prayed that said agent be directed to filed the documents in question immediately. On the 25th of said month the court issued an order requiring agent Emilio L. Siongco forthwith to file the search warrant and the affidavit in the court, together with the proceedings taken by him, and to present an inventory duly verified by oath of all the articles seized. On July 2d of said year, the attorney for the petitioner filed another petition alleging that the search warrant issue was illegal and that it had nit yet been returned to date together with the proceedings taken in connection therewith, and praying that said warrant be cancelled, that an order be issued directing the return of all the articles seized to the petitioner, that the agent who seized them be declared guilty of contempt of court, and that charges be filed against him for abuse of authority. On September 10, 1936, the court issued an order holding: that the search warrant was obtained and issued in accordance with the law, that it had been duly complied with and, consequently, should not be cancelled, and that agent Emilio L. Siongco did not commit any contempt of court and must, therefore, be exonerated, and ordering the chief of the Anti-Usury Board in Manila to show case, if any, within the unextendible period of two (2) days from the date of notice of said order, why all the articles seized appearing in the inventory, Exhibit 1, should not be returned to the petitioner. The assistant chief of the Anti-Usury Board of the Department of Justice filed a motion praying, for the reasons stated therein, that the articles seized be ordered retained for the purpose of conducting an investigation of the violation of the Anti-Usury Law committed by the petitioner. In view of the opposition of the attorney for the petitioner, the court, on September 25th, issued an order requiring the Anti-Usury Board to specify the time needed by it to examine the documents and papers seized and which of them should be retained, granting it a period of five (5) days for said purpose. On the 30th of said month the assistant chief of the Anti-Usury Board filed a motion praying that he be granted ten (10) days to comply with the order of September 25th and that the clerk of court be ordered to return to him all the documents and papers together with the inventory thereof. The court, in an order of October 2d of said year, granted him the additional period of ten(10) days and ordered the clerk of court to send him a copy of the inventory. On October 10th, said official again filed another motion alleging that he

needed sixty (60) days to examine the documents and papers seized, which are designated on pages 1 to 4 of the inventory by Nos. 5, 1016, 23, 25, 26, 27, 30, 31, 34, 36, 37, 38, 39, 40, 41, 42, 43 and 45, and praying that he be granted said period of sixty (60) days. In an order of October 16th, the court granted him the period of sixty (60) days to investigate said nineteen (19) documents. The petitioner alleges, and it is not denied by the respondents, that these nineteen (19)documents continue in the possession of the court, the rest having been returned to said petitioner. I. A search warrant is an order in writing, issued in the name of the People of the Philippine Islands, signed by a judge or a justice of the peace, and directed to a peace officer, commanding him to search for personal property and bring it before the court (section 95, General Orders. No. 58, as amended by section 6 of Act No. 2886). Of all the rights of a citizen, few are of greater importance or more essential to his peace and happiness than the right of personal security, and that involves the exemption of his private affairs, books, and papers from the inspection and scrutiny of others (In re Pacific Railways Commission, 32 Fed., 241; Interstate Commerce Commission vs Brimson, 38 Law. ed., 1047; Broyd vs. U. S., 29 Law. ed., 746; Caroll vs. U. S., 69 Law. ed., 543, 549). While the power to search and seize is necessary to the public welfare, still it must be exercised and the law enforced without transgressing the constitutional rights or citizen, for the enforcement of no statue is of sufficient importance to justify indifference to the basis principles of government (People vs. Elias, 147 N. E., 472). II. As the protection of the citizen and the maintenance of his constitutional right is one of the highest duties and privileges of the court, these constitutional guaranties should be given a liberal construction or a strict construction in favor of the individual, to prevent stealthy encroachment upon, or gradual depreciation on, the rights secured by them(State vs. Custer County, 198 Pac., 362; State vs. McDaniel, 231 Pac., 965; 237 Pac., 373). Since the proceeding is a drastic one, it is the general rule that statutes authorizing searches and seizure or search warrants must be strictly construed (Rose vs. St. Clair, 28 Fed., [2d], 189; Leonard vs. U. S., 6 Fed. [2d], 353; Perry vs. U. S. 14 Fed. [2d],88; Cofer vs. State, 118 So., 613). III. The petitioner claims that the search warrant issued by the court is illegal because it has been based upon the affidavit of agent Mariano G. Almeda in whose oath he declared that he had no personal knowledge of the facts which were to serve as a basis for the issuance of the warrant but that he had knowledge thereof through mere information secured from a person whom he considered reliable. To the question "What are your reason for applying for this search warrant", appearing in the affidavit, the agent answered: "It has been reported to me by a person whom I consider to be reliable that there are being kept in said premises, books, documents, receipts, lists, chits, and other papers used by him in connection with his activities as a money-lender, charging a usurious rate of interest, in violation of the law" and in attesting the truth of his statements contained in the affidavit, the said agent states that he found them to be correct and true to the best of his knowledge and belief.

Section 1, paragraph 3, of Article III of the Constitution, relative to the bill of rights, provides that "The right of the people to be secure in their persons, houses, papers, and effects against unreasonable searches and seizures shall not be violated, and no warrants shall issue but upon probable cause, to be determined by the judge after examination under oath or affirmation of the complainant and the witnesses he may produce, and particularly describing the place top be searched, and the persons or things to be seized." Section 97 of General Orders, No. 58 provides that "A search warrant shall not issue except for probable cause and upon application supported by oath particularly describing the place to be searched and the person or thing to be seized." It will be noted that both provisions require that there be not only probable cause before the issuance of a search warrant but that the search warrant must be based upon an application supported by oath of the applicant ands the witnesses he may produce. In its broadest sense, an oath includes any form of attestation by which a party signifies that he is bound in conscience to perform an act faithfully and truthfully; and it is sometimes defined asan outward pledge given by the person taking it that his attestation or promise is made under an immediate sense of his responsibility to God (Bouvier's Law Dictionary; State vs. Jackson, 137 N. W., 1034; In re Sage, 24 Oh. Cir. Ct. [N. S.], 7; Pumphery vs. State, 122 N. W., 19; Priest vs. State, 6 N. W., 468; State vs. Jones, 154 Pac., 378; Atwood vs. State, 111 So., 865). The oath required must refer to the truth of the facts within the personal knowledge of the petitioner or his witnesses, because the purpose thereof is to convince the committing magistrate, not the individual making the affidavit and seeking the issuance of the warrant, of the existence of probable cause (U. S. vs. Tureaud, 20 Fed., 621; U. S. vs. Michalski, 265 Fed., 8349; U. S. vs. Pitotto, 267 Fed., 603; U. S. vs. Lai Chew, 298 Fed., 652). The true test of sufficiency of an affidavit to warrant issuance of a search warrant is whether it has been drawn in such a manner that perjury could be charged thereon and affiant be held liable for damages caused (State vs. Roosevelt Country 20th Jud. Dis. Ct., 244 Pac., 280; State vs. Quartier, 236 Pac., 746). It will likewise be noted that section 1, paragraph 3, of Article III of the Constitution prohibits unreasonable searches and seizure. Unreasonable searches and seizures are a menace against which the constitutional guarantee afford full protection. The term "unreasonable search and seizure" is not defined in the Constitution or in General Orders No. 58, and it is said to have no fixed, absolute or unchangeable meaning, although the term has been defined in general language. All illegal searches and seizure are unreasonable while lawful ones are reasonable. What constitutes a reasonable or unreasonable search or seizure in any particular case is purely a judicial question, determinable from a consideration of the circumstances involved, including the purpose of the search, the presence or absence or probable cause, the manner in which the search and seizure was made, the place or thing searched, and the character of the articles procured (Go-Bart Importing Co. vs. U. S. 75 Law. ed., 374; Peru vs. U. S., 4 Fed., [2d], 881;U. S. vs. Vatune, 292 Fed., 497; Angelo vs. U. S. 70 Law, ed., 145; Lambert vs. U. S. 282 Fed., 413; U. S. vs. Bateman, 278 Fed., 231; Mason vs. Rollins, 16 Fed. Cas. [No. 9252], 2 Biss., 99). In view of the foregoing and under the above-cited authorities, it appears that the affidavit, which served as the exclusive basis of the search warrant, is insufficient

and fatally defective by reason of the manner in which the oath was made, and therefore, it is hereby held that the search warrant in question and the subsequent seizure of the books, documents and other papers are illegal and do not in any way warrant the deprivation to which the petitioner was subjected. IV. Another ground alleged by the petitioner in asking that the search warrant be declared illegal and cancelled is that it was not supported by other affidavits aside from that made by the applicant. In other words, it is contended that the search warrant cannot be issued unless it be supported by affidavits made by the applicant and the witnesses to be presented necessity by him. Section 1, paragraph 3, of Article III of the Constitution provides that no warrants shall issue but upon probable cause, to be determined by the judge after examination under oath or affirmation of the complainant and the witnesses he may produce. Section 98 of General Orders, No. 58 provides that the judge or justice must, before issuing the warrant, examine under oath the complainant and any witnesses he may produce and take their depositions in writing. It is the practice in this jurisdiction to attach the affidavit of at least the applicant or complainant to the application. It is admitted that the judge who issued the search warrant in this case, relied exclusively upon the affidavit made by agent Mariano G. Almeda and that he did not require nor take the deposition of any other witness. Neither the Constitution nor General Orders. No. 58 provides that it is of imperative necessity to take the deposition of the witnesses to be presented by the applicant or complainant in addition to the affidavit of the latter. The purpose of both in requiring the presentation of depositions is nothing more than to satisfy the committing magistrate of the existence of probable cause. Therefore, if the affidavit of the applicant or complainant is sufficient, the judge may dispense with that of other witnesses. Inasmuch as the affidavit of the agent in this case was insufficient because his knowledge of the facts was not personal but merely hearsay, it is the duty of the judge to require the affidavit of one or more witnesses for the purpose of determining the existence of probable cause to warrant the issuance of the search warrant. When the affidavit of the applicant of the complaint contains sufficient facts within his personal and direct knowledge, it is sufficient if the judge is satisfied that there exist probable cause; when the applicant's knowledge of the facts is mere hearsay, the affidavit of one or more witnesses having a personal knowledge of the fact is necessary. We conclude, therefore, that the warrant issued is likewise illegal because it was based only on the affidavit of the agent who had no personal knowledge of the facts. V. The petitioner alleged as another ground for the declaration of the illegality of the search warrant and the cancellation thereof, the fact that it authorized its execution at night. Section 101 of General Orders, No. 58 authorizes that the search be made at night when it is positively asserted in the affidavits that the property is on the person or in the place ordered to be searched. As we have declared the affidavits insufficient and the warrant issued exclusively upon it illegal, our conclusion is that the contention is equally well founded and that the search could not legally be made at night. VI. One of the grounds alleged by the petitioner in support of his contention that the warrant was issued illegally is the lack of an adequate description of the books and documents to be seized. Section 1, paragraphs 3, of Article III of the Constitution, and

section 97 of General Orders, No. 58 provide that the affidavit to be presented, which shall serve as the basis for determining whether probable cause exist and whether the warrant should be issued, must contain a particular description of the place to be searched and the person or thing to be seized. These provisions are mandatory and must be strictly complied with (Munch vs. U. S., 24 Fed. [2d], 518; U. S. vs. Boyd, 1 Fed. [2d], 1019; U. S. vs. Carlson, 292 Fed., 463; U. S. vs. Borkowski, 268 Fed., 408; In re Tri-State Coal & Coke Co., 253 Fed., 605; People vs. Mayen, 188 Cal., 237; People vs. Kahn, 256 Ill. App., 4125); but where, by the nature of the goods to be seized, their description must be rather generally, it is not required that a technical description be given, as this would mean that no warrant could issue (People vs. Rubio, 57 Phil., 284; People vs. Kahn, supra). The only description of the articles given in the affidavit presented to the judge was as follows: "that there are being kept in said premises books, documents, receipts, lists, chits and other papers used by him in connection with his activities as moneylender, charging a usurious rate of interest, in violation of the law." Taking into consideration the nature of the article so described, it is clear that no other more adequate and detailed description could have been given, particularly because it is difficult to give a particular description of the contents thereof. The description so made substantially complies with the legal provisions because the officer of the law who executed the warrant was thereby placed in a position enabling him to identify the articles, which he did. VII. The last ground alleged by the petitioner, in support of his claim that the search warrant was obtained illegally, is that the articles were seized in order that the Anti-Usury Board might provide itself with evidence to be used by it in the criminal case or cases which might be filed against him for violation of the Anti-usury Law. At the hearing of the incidents of the case raised before the court it clearly appeared that the books and documents had really been seized to enable the Anti-Usury Board to conduct an investigation and later use all or some of the articles in question as evidence against the petitioner in the criminal cases that may be filed against him. The seizure of books and documents by means of a search warrant, for the purpose of using them as evidence in a criminal case against the person in whose possession they were found, is unconstitutional because it makes the warrant unreasonable, and it is equivalent to a violation of the constitutional provision prohibiting the compulsion of an accused to testify against himself (Uy Kheytin vs. Villareal, 42 Phil,, 886; Brady vs. U. S., 266 U. S., 620; Temperani vs. U. S., 299 Fed., 365; U. S. vs. Madden, 297 Fed., 679; Boyd vs. U. S.,116 U. S., 116; Caroll vs. U. S., 267 U. S., 132). Therefore, it appearing that at least nineteen of the documents in question were seized for the purpose of using them as evidence against the petitioner in the criminal proceeding or proceedings for violation against him, we hold that the search warrant issued is illegal and that the documents should be returned to him. The Anti-Usury Board insinuates in its answer that the petitioner cannot now question the validity of the search warrant or the proceedings had subsequent to the issuance thereof, because he has waived his constitutional rights in proposing a compromise whereby he agreed to pay a fine of P200 for the purpose of evading the criminal proceeding or proceedings. We are of the opinion that there was no such waiver, first, because the

petitioner has emphatically denied the offer of compromise and, second, because if there was a compromise it reffered but to the institution of criminal proceedings fro violation of the Anti-Usury Law. The waiver would have been a good defense for the respondents had the petitioner voluntarily consented to the search and seizure of the articles in question, but such was not the case because the petitioner protested from the beginning and stated his protest in writing in the insufficient inventory furnished him by the agents. Said board alleges as another defense that the remedy sought by the petitioner does not lie because he can appeal from the orders which prejudiced him and are the subject matter of his petition. Section 222 of the Code of Civil Procedure in fact provides that mandamus will not issue when there is another plain, speedy and adequate remedy in the ordinary course of law. We are of the opinion, however, that an appeal from said orders would have to lapse before he recovers possession of the documents and before the rights, of which he has been unlawfully deprived, are restored to him (Fajardo vs. Llorente, 6 Phil., 426; Manotoc vs. McMicking and Trinidad, 10 Phil., 119; Cruz Herrera de Lukban vs. McMicking, 14 Phil., 641; Lamb vs. Phipps, 22 Phil., 456). Summarizing the foregoing conclusions, we hold: 1. That the provisions of the Constitution and General Orders, No. 58, relative to search and seizure, should be given a liberal construction in favor of the individual in order to maintain the constitutional guaranties whole and in their full force; 2. That since the provisions in question are drastic in their form and fundamentally restrict the enjoyment of the ownership, possession and use of the personal property of the individual, they should be strictly construed; 3. That the search and seizure made are illegal for the following reasons: (a) Because the warrant was based solely upon the affidavit of the petitioner who had no personal knowledge of the facts of probable cause, and (b) because the warrant was issued for the sole purpose of seizing evidence which would later be used in the criminal proceedings that might be instituted against the petitioner, for violation of the Anti-Usury Law; 4. That as the warrant had been issued unreasonably, and as it does not appear positively in the affidavit that the articles were in the possession of the petitioner and in the place indicated, neither could the search and seizure be made at night; 5. That although it is not mandatory to present affidavits of witnesses to corroborate the applicant or a complainant in cases where the latter has personal knowledge of the facts, when the applicant's or complainant's knowledge of the facts is merely hearsay, it is the duty of the judge to require affidavits of other witnesses so that he may determine whether probable cause exists; 6. That a detailed description of the person and place to be searched and the articles to be

seized is necessary, but whereby, by the nature of the articles to be seized, their description must be rather general, but is not required that a technical description be given, as this would mean that no warrant could issue; 7. That the petitioner did not waive his constitutional rights because the offer of compromise or settlement attributed to him, does not mean, if so made, that he voluntarily tolerated the search and seizure; and 8. That an appeal from the orders questioned by the petitioner, if taken by him, would not be an effective, speedy or adequate remedy in the ordinary course of law, and, consequently, the petition for mandamus filed by him, lies. For the foregoing considerations, the search warrant and the seizure of June 3, 1936, and the orders of the respondent court authorizing the relation of the books and documents, are declared illegal and are set aside, and it is ordered that the judge presiding over the Court of First Instance of Tayabas direct the immediate return to the petitioner of the nineteen (19) documents designated on pages 1 to 4 of the inventory by Nos. 5, 10, 16, 23, 25,26, 27, 30, 31, 34, 36, 37, 38, 39, 40, 41, 42, 43 and 45, without special pronouncement as to costs. So ordered. Avancea, C.J., Villa-Real, Diaz and Concepcion, JJ., concur.

Separate Opinions ABAD SANTOS, J., concurring: My views on the fundamental questions involved in this case are fully set forth in my dissenting opinion filed in People vs. Rubio (57 Phil., 384, 395). I am gratified to see that, in the main, those views have now prevailed. I therefore concur in the decision of the court herein. LAUREL, J., concurring: I subscribe to the views expressed in the foregoing carefully prepared opinion, with the reservation now to be stated. To my mind, the search warrant in this case does not satisfy the constitutional requirement regarding the particularity of the description of "the place to be searched and the persons or things to be seized" (par. 3, sec. 1, Art. III, Constitution of the Philippines). Reference to "books, documents, receipts, lists, chits and other papers used by him in connection with his activities as money-lender, charging usurious rates of interest in violation of the law" in the search warrant is so general, loose and vague as to confer unlimited discretion upon the officer serving the warrant to choose and determine for himself just what are the "books, documents, receipts, lists, chits and other papers" used by the petitioner in connection with his alleged activities as money-lender. The

evident purpose and intent of the constitutional requirement is to limit the things to be seized to those, and only those, particularly described in the search warrant, to the end that unreasonable searches and seizures may not be made, that abuses may not be committed (Uy Kheytin vs. Villareal, 42 Phil., 886).

G.R. No. 109633 July 20, 1994 THE PEOPLE OF THE PHILIPPINES, plaintiff-appellee, vs. NORMANDO DEL ROSARIO Y LOPEZ, accused-appellant. The Solicitor General for plaintiff-appellee. Topacio and Topacio for accused-appellants.

MELO, J.: Normando del Rosario was charged before Branch 17 of the Regional Trial Court of the Fourth Judicial Region stationed in Cavite City with Illegal Possession of Firearm and Ammunitions in Criminal Case No. 236-91 and Illegal Sale of Regulated Drugs in Criminal Case No. 237-91, under two informations reading, respectively, as follows: Criminal Case No. 236-91 That on or about September 4, 1991, in the City of Cavite, Republic of the Philippines and within the jurisdiction of this Honorable Court, the above-named accused, without legal authority, did, then and there, willfully, unlawfully, feloniously and knowingly have in his possession and control a homemade (paltik)caliber .22 revolver with three (3) live ammunition. Contrary to law. Criminal Case No. 237-91 That on or about September 4, 1991, in the City of Cavite, Republic of the Philippines and within the jurisdiction of this Honorable Court, the above-named accused, without legal authority, did, then and there, willfully, unlawfully, feloniously and knowingly sell to a poseur buyer an aluminum foil containing Methamphetamine Hydrochloride also known as "Shabu", a regulated drug. Contrary to law. (pp. 20-21, Rollo.) Upon arraignment, accused-appellant pleaded not guilty to both charges, and after joint trial of the two cases, the court a quo rendered a decision, the dispositive portion of which reads: WHEREFORE, in view of the foregoing, the Court finds the accused Normando

del Rosario y Lopez guilty beyond reasonable doubt in the above-entitled cases and he is hereby sentenced to undergo imprisonment: in Crim. Case No. 236-91 for Violation of P.D. 1866 of Seventeen (17) years, Four (4) months and One (1) day of reclusion temporal, as minimum to Twenty (20) years of reclusion temporal, as maximum and in Crim. Case No. 237-91 for a violation of Section 15, Article III of Republic Act 6425, as amended of life imprisonment and to pay a fine of P30,000.00, without subsidiary imprisonment in case of insolvency and to pay the costs in both cases. The shabu, the One Hundred Peso bill and other paraphernalia are hereby ordered confiscated in favor of the government. (pp. 28-29, Rollo.) From said decision, the instant appeal has been interposed. The prosecution's version of the case, as set forth in appellee's brief, is as follows: Upon application of SPO3 Raymundo Untiveros of the Philippine National Police (PNP) of Cavite City, Regional Trial Court Judge Arturo de Guia issued in the morning of September 4, 1991 a search warrant (Exh. T, p. 50, Rec. Crim. Case No. 237-91) authorizing the search and seizure of an "undetermined quantity of Methamphetamine Hydrochloride commonly known as shabu and its paraphernalias" in the premises of appellant's house located at 828 R. Basa St., San Roque, Cavite City. However, the search warrant was not implemented immediately due to the lack of police personnel to form the raiding team (pp. 4, 7, tsn., Feb. 4, 1992). At about 9 o'clock in the evening of that day, a raiding team was finally organized. SPO3 Untiveros headed the raiding team with PO3 Rogelio Francisco, SPO1 Eduardo Novero, SPO3 Reynaldo de la Cruz, PO1 Carlito Barbuco, PO3 Onrubio and SPO2 Villegas as members (pp. 5, 10, tsn., Feb. 4, 1992; p. 7, tsn., Dec. 11, 1991). In the final briefing of the raiding team at the police station, it was agreed upon that PO1 Venerando Luna will buy shabu from appellant and after his return from appellant's house, the raiding team will implement the search warrant (p. 10, tsn., Feb. 4, 1992; pp. 17-18, tsn., Dec. 11, 1991). A marked money consisting of a P100 bill bearing serial no. PQ 329406 (Exh. P, p. 51, Rec.) was given by the Station Commander to PO1 Luna and entered in the police logbook (p. 12, Feb. 4, 1992). PO1 Luna with a companion proceeded to appellant's house to implement the search warrant. Barangay Capt. Maigue, Norma del Rosario and appellant witnessed the search at appellant's house (p. 10, tsn., Dec. 11, 1991). SPO3 de la Cruz and PO3 Francisco found a black canister containing shabu, an aluminum foil, a paltik .22 caliber (Exh. O) atop the TV set, three used

ammunitions in a cup and three wallets (Exhs. Q, R, S), one containing the marked money (Exh. P; pp. 11-12, tsn., Dec. 11, 1992). SPO1 Novero found inside a show box aluminum foils, napkins and a burner (p. 9, tsn., March 11, 1992). SPO3 de la Cruz turned over the wallet containing the marked money to PO3 Onrubio (p. 8, 32, tsn., Jan. 7, 1992). The seized items were photographed thereat by Fred Agana and then turned over to PO3 Onrubio (pp. 8, 32, tsn., Jan. 7, 1992). SPO3 Untiveros issued receipts (Exhs. V, V-1, pp. 53-54, Rec.) for the seized items with Barangay Capt. Maigue and appellant's sister Norma as signing witnesses. He also made a return (Exh. U, p. 52, Rec.) of the seized items to the court (pp. 11-155, tsn., Feb. 18, 1992.). At police station, the seized items were taped and initialed by SPO3 de la Cruz (p. 33, tsn., Jan. 7, 1992). The next day, SPO4 Pilapil, through PO1 Barbuco, forwarded to NBI Forensic Chemist Mary Ann Aranas for laboratory analysis the aluminum foil (Exhs. A, J, pp. 37, 46, Rec.) containing suspected shabu bought by PO1 Luna from appellant in the buy-bust operation as well as the aluminum foils (Exhs. G, K, pp. 43, 47, Rec.) containing suspected marijuana which were confiscated by virtue of the search warrant. The findings of NBI Forensic Chemist Aranas disclosed that all the specimen submitted to her for laboratory analysis by SPO1 Pilapil, thru PO1 Barbuco, gave positive results for Methamphetamine Hydrochloride (pp. 2-9, tsn., Dec. 3, 1991; Exh. B, C, H, I, pp. 38, 39, 44, 45, Rec.). (pp. 102-105, Rollo.) Carefully evaluating the evidence on record, we believe that the prosecution has failed to prove the guilt of accused-appellant. Much is to be desired in the manner the police authorities effected the arrest of accused-appellant and the same observation may be made with regard to the way the prosecution conducted its case. Foremost among the inadequacies of the prosecution is its failure to call to the witness stand PO1 Venerando Luna, the alleged poseur-buyer. There is, thus, a total absence of evidence to establish the purported sale of shabu by accusedappellant to Venerando Luna, the supposed poseur-buyer. The omission to present the poseur-buyer casts serious doubts that an illegal sale of a dangerous drug actually took place. The trial court gave much weight to the testimonies of the police members of the buy-bust operation. However, the prosecution did not present as witness the supposed poseur-buyer. Such omission casts serious doubt on appellant's guilt because without the testimony of the poseur-buyer, there is no convincing evidence to show that appellant sold marijuana. The testimonies of the rest of the buy-bust operation are hearsay in view of the fact that the poseur-buyer, was never presented at the trial. There was even no testimony that when the

accused-appellant handed the stuff to the poseur-buyer that the latter in turn handed the marked money. The failure of the prosecution to present the alleged buyer of the marijuana was a fatal flaw in the case against the accused. (People vs. Fulgarillas, 212 SCRA 76, 80 [1992]) The testimony of prosecution witness PO3 Rogelio Francisco that Veneracion Luna, the alleged Poseur-buyer, bought shabu from accused-appellant was derived solely from what Luna supposedly told him (pp. 19-20, tsn., December 11, 1991) and, therefore, is patently hearsay evidence, without any evidentiary weight whatsoever. Likewise, the statements of prosecution witnesses Policemen Reynaldo de la Cruz, Raymundo Untiveros, and Eduardo Novera, Jr. as to the alleged sale of shabu are hearsay, without weight, as all of them were not present during the alleged sale. According to the version of the prosecution, during the alleged buy-bust operation, accused-appellant handed over to Veneracion Luna, the alleged poseur-buyer, a quantity of shabu, and Luna in turn paid accused-appellant a marked P100 bill and then returned to the police station and informed the raiding team that he had already bought the shabu from accused-appellant. Thereupon, the raiding team proceeded to the house of accused-appellant to implement the search warrant. The version of the prosecution is highly incredible. The record is devoid of any reason why the police officers did not make any attempt to arrest accused-appellant at the time he allegedly sold the shabu to Veneracion Luna who was accompanied by another police officer. That was the opportune moment to arrest accused-appellant. The version foisted by the prosecution upon this Court is contrary to human experience in the ordinary course of human conduct. The usual procedure in a buy-bust operation is for the police officers to arrest the pusher of drugs at the very moment he hands over the dangerous drug to the poseur-buyer. That is the very reason why such a police operation is called a "buy-bust" operation. The police poseur-buyer "buys" dangerous drugs from the pusher and "busts" (arrests) him the moment the pusher hands over the drug to the police officer. We thus entertain serious doubts that the shabu contained in a small canister was actually seized or confiscated at the residence of accused-appellant. In consequence, the manner the police officers conducted the subsequent and much-delayed search is highly irregular. Upon bargaining into the residence of accused-appellant, the police officers found him lying down and they immediately arrested and detained him in the living room while they searched the other parts of the house. Although they fetched two persons to witness the search, the witnesses were called in only after the policemen had already entered accusedappellant's residence (pp. 22-23, tsn, December 11, 1991), and, therefore, the policemen had more than ample time to plant the shabu. Corollary to the constitutional precept that, in all criminal prosecutions, the accused shall be presumed innocent until the contrary is proved (Sec. 14(2), Article III,

Constitution of the Republic of the Philippines) is the rule that in order to convict an accused the circumstances of the case must exclude all and each and every hypothesis consistent with his innocence (People vs. Tanchoco; 76 Phil. 463 [1946]; People vs. Constante, 12 SCRA 653 [1964]; People vs. Jara, 144 SCRA 516 [1986]). The facts of the case do not rule out the hypothesis that accusedappellant is innocent. At any rate, accused-appellant cannot be convicted of possession of the shabu contained in a canister and allegedly seized at his house, for the charge against him was for selling shabu with the information alleging that the "accused, without legal authority did . . . sell to a poseur buyer an aluminum foil containing Methamphetamine Hydrochloride . . ." Sale is totally different from possession. Article 1458 of the Civil Code defines sale as a contract whereby "one of the contracting parties obligates himself to transfer the ownership of and to deliver a determine thing, and the other to pay therefor a price certain in money or its equivalent", while "possession is the holding of a thing or the enjoyment of a right" as defined by Article 523 of the Civil Code. Accused-appellant cannot be convicted of a crime which is not charged in the information for to do so would deny him the due process of law (People vs. Despavellador, 1 SCRA 205 [1961]; People vs. Mori, 55 SCRA 382 [1974]). Neither can accused-appellant be convicted of illegal possession of firearm and ammunition. The search warrant implemented by the raiding party authorized only the search and seizure of ". . . the described quantity of Methamphetamine Hydrochloride commonly known as shabu and its paraphernalia" (Exh. O, p. 50, original record). Thus, the raiding party was authorized to seize only shabu and paraphernalia for the use thereof and no other. A search warrant is not a sweeping authority empowering a raiding party to undertake a finishing expedition to seize and confiscate any and all kinds of evidence or articles relating to a crime. The Constitution itself (Section 2, Article III) and the Rules of Court (Section 3, Rule 126) specifically mandate that the search warrant must particularly describe the things to be seized. Thus, the search warrant was no authority for the police officers to seize the firearm which was not mentioned, much less described with particularity, in the search warrant. Neither may it be maintained that the gun was seized in the course of an arrest, for as earlier observed, accused-appellant's arrest was far from regular and legal. Said firearm, having been illegally seized, the same is not admissible in evidence (Stonehill vs. Diokno, 20 SCRA 383 [1967]). The Constitution expressly ordains the exclusion in evidence of illegally seized articles. Any evidence obtained in violation of this or the preceding section shall be inadmissible for any purpose in any proceeding. (Section 3[2], Article III, Constitution of the Republic of the Philippines). With the exclusion in evidence of the illegally seized firearm, there is, therefore, a

total absence of evidence to support the charge of illegal possession of firearm, against accused-appellant. The same may be said of the charge of illegal possession of ammunition. WHEREFORE, the decision appealed from is hereby REVERSED and accusedappellant is hereby ACQUITTED in Criminal Case No. 236-91 and Criminal Case No. 237-91. The immediate release of accused-appellant is hereby ordered unless there exists a pending valid cause against him. The shabu, the marked P100 bill, firearm, and ammunition are hereby ordered confiscated in favor of the government. SO ORDERED.

G.R. No. 81567 October 3, 1991 IN THE MATTER OF THE PETITION FOR HABEAS CORPUS OF ROBERTO UMIL, ROLANDO DURAL and RENATO VILLANUEVA, MANOLITA O. UMIL and NICANOR P. DURAL, FELICITAS V. SESE, petitioners, vs. FIDEL V. RAMOS, MAJ. GEN. RENATO DE VILLA, BRIG. GEN. RAMON MONTANO, BRIG. GEN. ALEXANDER AGUIRRE, respondents. G.R. Nos. 84581-82 October 3, 1991 AMELIA ROQUE and WILFREDO BUENAOBRA, petitioners, vs. GEN. RENATO DE VILLA and GEN, RAMON MONTANO, respondents. G.R. Nos. 84583-84 October 3, 1991 IN THE MATTER OF THE PETITION FOR HABEAS CORPUS OF ATTY. DOMINGO T. ANONUEVO and RAMON CASIPLE: DOMINGO T. ANONUEVO and RAMON CASIPLE, petitioners, vs. HON. FIDEL V. RAMOS, GEN. RENATO S. DE VILLA, COL. EVARISTO CARIO, LT. COL. REX D. PIAD, T/SGT. CONRADO DE TORRES, S/SGT. ARNOLD DURIAN, and Commanding Officer, PC-INP Detention Center, Camp Crame, Quezon City, respondents. G.R. No. 83162 October 3, 1991 IN THE MATTER OF THE APPLICATION FOR HABEAS CORPUS OF VICKY A. OCAYA AND DANNY RIVERA: VIRGILIO A. OCAYA, petitioners, vs. BRIG. GEN. ALEXANDER AGUIRRE, COL. HERCULES CATALUNA, COL. NESTOR MARIANO, respondents. G.R. No. 85727 October 3, 1991 IN THE MATTER OF APPLICATION FOR HABEAS CORPUS OF DEOGRACIAS ESPIRITU, petitioner, vs. BRIG. GEN.ALFREDO S. LIM, COL. RICARDO REYES, respondents. G.R. No. 86332 October 3, 1991 IN THE MATTER OF THE PETITION FOR HABEAS CORPUS OF NARCISO B. NAZARENO: ALFREDO NAZARENO,petitioner, vs. THE STATION COMMANDER OF THE MUNTINGLUPA POLICE STATION, Muntinglupa, Metro Manila, P/SGT. JACINTO MEDINA, P/SGT. ELADIO TAGLE, P/SGT. LEVI SOLEDAD, and P/SGT. MALTRO AROJADO, respondents. Efren H. Mercado for petitioners in G.R. No. 81567 and G. R. No. 83162.

Ricardo C. Valmonte for petitioners in G.R. Nos. 84581-82 Josefina G. Campbell-Castillo for petitioners in G.R. Nos. 84583-84. Potenciano A. Flores, Jr. for petitioner in G.R. No. 85727. The Solicitor General for the respondents. RESOLUTION

PER CURIAM:p Before the Court are separate motions filed by the petitioners in the aboveentitled petitions, seeking reconsideration of the Court's decision promulgated on 9 July 1990 (the decision, for brevity) which dismissed the petitions, with the following dispositive part: WHEREFORE, the petitions are hereby DISMISSED, except that in G.R. No. 85727 (Espiritu vs. Lim), the bail bond for petitioner's provisional liberty is hereby ordered reduced from P60,000.00 to P10,000.00. No costs. The Court avails of this opportunity to clarify its ruling a begins with the statement that the decision did not rule as many misunderstood it to do that mere suspicion that one is Communist Party or New People's Army member is a valid ground for his arrest without warrant. Moreover, the decision merely applied long existing laws to the factual situations obtaining in the several petitions. Among these laws are th outlawing the Communist Party of the Philippines (CPP) similar organizations and penalizing membership therein be dealt with shortly). It is elementary, in this connection, if these laws no longer reflect the thinking or sentiment of the people, it is Congress as the elected representative of the people not the Court that should repeal, change or modify them. In their separate motions for reconsideration, petitioners, in sum, maintain: 1. That the assailed decision, in upholding the validity of the questioned arrests made without warrant, and in relying on the provisions of the Rules of Court, particularly Section 5 of Rule 113 (Arrest), disregards the fact that such arrests violated the constitutional rights of the persons arrested; 2. That the doctrine laid down in Garcia vs. Enrile 1 and Ilagan vs. Enrile 2 should be abandoned; 3. That the decision erred in considering the admissions made by the persons arrested as to their membership in the Communist Party of the Philippines/New

People's Army, and their ownership of the unlicensed firearms, ammunitions and subversive documents found in their possession at the time of arrest, inasmuch as those confessions do not comply with the requirements on admissibility of extrajudicial admissions; 4. That the assailed decision is based on a misappreciation of facts; 5. That G.R. No. 81567 (the Umil case) should not be deemed moot and academic. We find no merit in the motions for reconsideration. It can not be overlooked that these are petitions for the issuance of the writ of habeas corpus, filed by petitioners under the Rules of Court. 3 The writ of habeas corpus exists as a speedy and effective remedy to relieve persons from unlawful restraint. 4 Therefore, the function of the special proceedings of habeas corpus is to inquire into the legality of one's detention, 5 so that if detention is illegal, the detainee may be ordered forthwit released. In the petitions at bar, to ascertain whether the detention petitioners was illegal or not, the Court before rendering decision dated 9 July 1990, looked into whether their questioned arrests without warrant were made in accordance with law. For, if the arrests were made in accordance with law, would follow that the detention resulting from such arrests also in accordance with law. There can be no dispute that, as a general rule, no peace officer or person has the power or authority to arrest anyo without a warrant of arrest, except in those cases express authorized by law. 6 The law expressly allowing arrests witho warrant is found in Section 5, Rule 113 of the Rules of Court which states the grounds upon which a valid arrest, without warrant, can be conducted. In the present cases, the focus is understandably on Section 5, paragraphs (a) and (b) of the said Rule 113, which read: Sec. 5. Arrest without warrant; when lawful. A peace officer or a private person may, without a warrant, arrest a person: (a) When, in his presence, the person to he arrested has committed, is actually committing, or is attempting to commit an offense; (b) When an offense has in fact just been committed, and he has personal knowledge of facts indicating that the person to be arrest has committed it; and . . . (Emphasis supplied). The Court's decision of 9 July 1990 rules that the arrest Rolando Dural (G.R. No.

81567) without warrant is justified it can be said that, within the contemplation of Section 5 Rule 113, he (Dural) was committing an offense, when arrested because Dural was arrested for being a member of the New People's Army, an outlawed organization, where membership penalized, 7 and for subversion which, like rebellion is, under the doctrine of Garcia vs. Enrile, 8 a continuing offense, thus: The crimes of insurrection or rebellion, subversion, conspiracy or proposal to commit such crimes, and other crimes and offenses committed in the furtherance (sic) on the occasion thereof, or incident thereto, or in connection therewith under Presidential Proclamation No. 2045, are all in the nature of continuing offenses which set them apart from the common offenses, aside from their essentially involving a massive conspiracy of nationwide magnitude. . . . Given the ideological content of membership in the CPP/NPA which includes armed struggle for the overthrow of organized government, Dural did not cease to be, or became less of a subversive, FOR PURPOSES OF ARREST, simply because he was, at the time of arrest, confined in the St. Agnes Hospital. Dural was identified as one of several persons who the day before his arrest, without warrant, at the St. Agnes Hospital, had shot two (2) CAPCOM policemen in their patrol car. That Dural had shot the two (2) policemen in Caloocan City as part of his mission as a "sparrow" (NPA member) did not end there and then. Dural, given another opportunity, would have shot or would shoot other policemen anywhere as agents or representatives of organized government. It is in this sense that subversion like rebellion (or insurrection) is perceived here as a continuing offense. Unlike other so-called "common" offenses, i.e. adultery, murder, arson, etc., which generally end upon their commission, subversion and rebellion are anchored on an ideological base which compels the repetition of the same acts of lawlessness and violence until the overriding objective of overthrowing organized government is attained. Nor can it be said that Dural's arrest was grounded on mere suspicion by the arresting officers of his membership in the CPP/NPA. His arrest was based on "probable cause," as supported by actual facts that will be shown hereafter. Viewed from another but related perspective, it may also be said, under the facts of the Umil case, that the arrest of Dural falls under Section 5, paragraph (b), Rule 113 of the Rules of Court, which requires two (2) conditions for a valid arrestt without warrant: first, that the person to be arrested has just committed an offense, and second, that the arresting peace officer or private person has personal knowledge of facts indicating that the person to be arrested is the one who committed the offense. Section 5(b), Rule 113, it will be noted, refers to arrests without warrant, based on "personal knowledge of facts" acquired by the arresting officer or private person. It has been ruled that "personal knowledge of facts," in arrests without warrant

must be based upon probable cause, which means an actual belief or reasonable grounds of suspicion 9 The grounds of suspicion are reasonable when, in the absence of actual belief of the arresting officers, the suspicion that the person to be arrested is probably guilty of committing the offense, is based on actual facts, i.e., supported by circumstances sufficiently strong in themselves to create the probable cause of guilt of the person to be arrested. 10 A reasonable suspicion therefore must be founded on probable cause, coupled with good faith on the part of the peace officers making the arrest. 11 These requisites were complied with in the Umil case and in the other cases at bar. In G.R. No. 81567 (Umil case), military agents, on 1 February 1988, were dispatched to the St. Agnes Hospital, Roosevelt Avenue, Quezon City, to verify a confidential information which was received by their office, about a "sparrow man" (NPA member) who had been admitted to the said hospital with a gunshot wound; that the information further disclosed that the wounded man in the said hospital was among the five (5) male "sparrows" who murdered two (2) Capcom mobile patrols the day before, or on 31 January 1988 at about 12:00 o'clock noon, before a road hump along Macanining St., Bagong Barrio, Caloocan City; that based on the same information, the wounded man's name was listed by the hospital management as "Ronnie Javellon," twenty-two (22) years old of Block 10, Lot 4, South City Homes, Bian, Laguna. 12 Said confidential information received by the arresting officers, to the effect that an NPA member ("sparrow unit") was being treated for a gunshot wound in the named hospital, is deemed reasonable and with cause as it was based on actual facts and supported by circumstances sufficient to engender a belief that an NPA member was truly in the said hospital. The actual facts supported by circumstances are: first the day before, or on 31 January 1988, two (2) CAPCOM soldiers were actually killed in Bagong Bario, Caloocan City by five (5) "sparrows" including Dural; second a wounded person listed in the hospital records as "Ronnie Javellon" was actually then being treated in St. Agnes Hospital for a gunshot wound; third as the records of this case disclosed later, "Ronnie Javellon" and his address entered in the hospital records were fictitious and the wounded man was in reality Rolando Dural. In fine, the confidential information received by the arresting officers merited their immediate attention and action and, in fact, it was found to be true. Even the petitioners in their motion for reconsideration, 13 believe that the confidential information of the arresting officers to the effect that Dural was then being treated in St. Agnes Hospital was actually received from the attending doctor and hospital management in compliance with the directives of the law, 14 and, therefore, came from reliable sources.

As to the condition that "probable cause" must also be coupled with acts done in good faith by the officers who make the arrest, the Court notes that the peace officers wno arrested Dural are deemed to have conducted the same in good faith, considering that law enforcers are presumed to regularly perform their official duties. The records show that the arresting officers did not appear to have been ill-motivated in arresting Dural. 15 It is therefore clear that the arrest, without warrant, of Dural was made in compliance with the requirements of paragraphs (a) and (b) of Section 5, Rule 113. Parenthetically, it should be mentioned here that a few day after Dural's arrest, without warrant, an information charging double murder with assault against agents of persons in authority was filed against Dural in the Regional Trial Court of Caloocan City (Criminal Case No. C-30112). He was thus promptly placed under judicial custody (as distinguished fro custody of the arresting officers). On 31 August 1988, he wa convicted of the crime charged and sentenced to reclusion perpetua. The judgment of conviction is now on appeal before this Court in G.R. No. 84921. As to Amelia Roque and Wilfredo Buenaobra (G.R. Nos. 84581-82), Domingo Anonuevo and Ramon Casiple (G.R. Nos. 84583-84) and Vicky Ocaya (G.R. No. 83162), their arrests, without warrant, are also justified. They were searched pursuant to search warrants issued by a court of law and were found wit unlicensed firearms, explosives and/or ammunition in their persons. They were, therefore, caught in flagrante delicto which justified their outright arrests without warrant, under Sec 5(a), Rule 113, Rules of Court. Parenthetically, it should be mentioned here that a few davs after their arrests without warrant, informations were filed in court against said petitioners, thereby placing them within judicial custody and disposition. Furthermore, Buenaobra mooted his own petition fo habeas corpus by announcing to this Court during the hearing of these petitions that he had chosen to remain in detention in the custody of the authorities. More specifically, the antecedent facts in the "in flagrante" cases are: 1. On 27 June 1988, the military agents received information imparted by a former NPA about the operations of the CPP and NPA in Metro Manila and that a certain house occupied by one Renato Constantine, located in the Villaluz Compound, Molave St., Marikina Heights, Marikina, Metro Manila was being used as their safehouse; that in view of this information, the said house was placed under military surveillance and on 12 August 1988, pursuant to a search warrant duly issued by court, a search of the house was conducted; that when Renato Constantine was then confronted he could not produce any permit to possess the firearms, ammunitions, radio and other communications equipment, and he admitted that he was a ranking member of the CPP. 16 2. In the case of Wilfredo Buenaobra, he arrived at the house of Renato Constantino in the evening of 12 August 1988, and admitted that he was an NPA

courier and he had with him letters to Renato Constantine and other members of the rebel group. 3. On the other hand, the arrest of Amelia Roque was a consequence of the arrest of Buenaobra who had in his possession papers leading to the whereabouts of Roque; 17 that, at the time of her arrest, the military agents found subversive documents and live ammunitions, and she admitted then that the documents belonged to her. 18 4. As regards Domingo Anonuevo and Ramon Casiple they without warrant on 13 August 1988, when they arrived at the Renato Constantine in the evening of said date; that when the them, subversive documents, and loaded guns were found possession but failing to show a permit to possess them. 19 were arrested said house of agents frisked in the latter's

5. With regard to Vicky Ocaya, she was arrested, without warrant when she arrived (on 12 May 1988) at the premises ofthe house of one Benito Tiamzon who was believed to be the head of the CPP/NPA, and whose house was subject of a search warrant duly issued by the court. At the time of her arrest without warrant the agents of the PC-Intelligence and Investigation found ammunitions and subversive documents in the car of Ocaya. 20 It is to be noted in the above cases (Roque, Buenaobra, Anonuevo, Casiple and Ocaya) that the reason which compelled the military agents to make the arrests without warrant was the information given to the military authorities that two (2) safehouses (one occupied by Renato Constantine and the other by Benito Tiamzon) were being used by the CPP/NPA for their operations, with information as to their exact location and the names of Renato Constantine and Benito Tiamzon as residents or occupants thereof. And at the time of the actual arrests, the following circumstances surrounded said arrests (of Roque, Buenaobra, Anonuevo and Casiple), which confirmed the belief of the military agents that the information they had received was true and the persons to be arrested were probably guilty of the commission of certain crimes: first: search warrant was duly issued to effect the search of the Constantine safehouse; second: found in the safehouse was a person named Renato Constantine, who admitted that he was a ranking member of the CPP, and found in his possession were unlicensed firearms and communications equipment; third: at the time of their arrests, in their possession were unlicensed firearms, ammunitions and/or subversive documents, and they admitted ownership thereof as well as their membership in the CPP/NPA. And then, shortly after their arrests, they were positively identified by their former comrades in the organization as CPP/NPA members. In view of these circumstances, the corresponding informations were filed in court against said arrested persons. The records also show that, as in the case of Dural, the arrests without warrant made by the military agents in the Constantino safehouse and later in the Amelia

Roque house, do not appear to have been ill-motivated or irregularly performed. With all these facts and circumstances existing before, during and after the arrest of the afore-named persons (Dural, Buenaobra, Roque, Anonuevo, Casiple and Ocaya), no prudent an can say that it would have been better for the military agents not to have acted at all and made any arrest. That would have been an unpardonable neglect of official duty and a cause for disciplinary action against the peace officers involved. For, one of the duties of law enforcers is to arrest lawbreakers in order to place them in the hands of executive and judicial authorities upon whom devolves the duty to investigate the acts constituting the alleged violation of law and to prosecute and secure the punishment therefor. 21 An arrest is therefore in the nature of an administrative measure. The power to arrest without warrant is without limitation as long as the requirements of Section 5, Rule 113 are met. This rule is founded on an overwhelming public interest in peace and order in our communities. In ascertaining whether the arrest without warrant is conducted in accordance with the conditions set forth in Section 5, Rule 113, this Court determines not whether the persons arrested are indeed guilty of committing the crime for which they were arrested. 22 Not evidence of guilt, but "probable cause" is the reason that can validly compel the peace officers, in the performance of their duties and in the interest of public order, to conduct an arrest without warrant. 23 The courts should not expect of law-enforcers more than what the law requires of them. Under the conditions set forth in Section 5, Rule 113, particularly paragraph (b) thereof, even if the arrested persons are later found to be innocent and acquitted, the arresting officers are not liable. 24 But if they do not strictly comply with the said conditions, the arresting officers can be held liable for the crime of arbitrary detention, 25 for damages under Article 32 of the Civil Code 26 and/or for other administrative sanctions. In G.R. No. 85727, Espiritu, on 23 November 1988, was arrested without warrant, on the basis of the attestation of certain witnesses: that about 5:00 o'clock in the afternoon of 22 November 1988, at the corner of Magsaysay Boulevard and Velencia St., Sta. Mesa, Manila, Espiritu spoke at a gathering of drivers and sympathizers, where he said, among other things: Bukas tuloy ang welga natin . . . hanggang sa magkagulona. 27 (Emphasis supplied) and that the police authorities were present during the press conference held at the National Press Club (NPC) on 22 November 1988 where Espiritu called for a nationwide strike (of jeepney and bus drivers) on 23 November 1988. 28 Espiritu was arrested without warrant, not for subversion or any "continuing offense," but

for uttering the above-quoted language which, in the perception of the arresting officers, was inciting to sedition. Many persons may differ as to the validity of such perception and regard the language as falling within free speech guaranteed by the Constitution. But, then, Espiritu had not lost the right to insist, during the pre-trial or trial on the merits, that he was just exercising his right to free speech regardless of the charged atmosphere in which it was uttered. But, the authority of the peace officers to make the arrest, without warrant, at the time the words were uttered, or soon thereafter, is still another thing. In the balancing of authority and freedom, which obviously becomes difficult at times, the Court has, in this case, tilted the scale in favor of authority but only for purposes of the arrest (not conviction). Let it be noted that the Court has ordered the bail for Espiritu's release to be reduced from P60,000.00 to P10,000.00. Let it also be noted that supervening events have made the Espiritu case moot and academic. For Espiritu had before arraignment asked the court a quo for reinvestigation, the peace officers did not appear. Because of this development, the defense asked the court a quo at the resumption of the hearings to dismiss the case. Case against Espiritu (Criminal Case No. 88-68385) has been provisionally dismissed and his bail bond cancelled. In G.R. No. 86332 (Nazareno), the records show that in the morning of 14 December 1988, Romulo Bunye II was killed by a group of men in Alabang, Muntinlupa, Metro Manila; that at about 5:00 o'clock in the morning of 28 December 1988, Ramil Regala, one of the suspects in the said killing, was arrested and he pointed to Narciso Nazareno as one of his companions during the killing of Bunye II; that at 7:20 of the same morning (28 December 1988), the police agents arrested Nazareno, without warrant, for investigation. 29 Although the killing of Bunye II occurred on 14 December 1988, while Nazareno's arrest without warrant was made only on 28 December 1988, or 14 days later, the arrest fans under Section 5(b) of Rule 113, since it was only on 28 December 1988 that the police authorities came to know that Nazareno was probably one of those guilty in the killing of Bunye II and the arrest had to be made promptly, even without warrant, (after the police were alerted) and despite the lapse of fourteen (14) days to prevent possible flight. As shown in the decision under consideration, this Court, in upholding the arrest without warrant of Nazareno noted several facts and events surrounding his arrest and detention, as follows: . . . on 3 January 1989 (or six (6) days after his arrest without warrant), an information charging Narciso Nazareno, Ramil Regala and two (2) others, with the killing of Romulo Bunye II was filed wit the Regional Trial Court of Makati, Metro Manila. The case is dock eted therein as Criminal Case No. 731.

On 7 January 1989, Narciso Nazareno filed a motion to post bail but the motion was denied by the trial court in an order dated 10 January 1989, even as the motion to post bail, earlier filed by his co-accused, Manuel Laureaga, was granted by the same trial court. On 13 January 1989, a petition for habeas corpus was filed with this Court on behalf of Narciso Nazareno and on 13 January 1989, the Court issued the writ of habeas corpus, retumable to the Presiding Judge of the Regional Trial Court of Bifian, Laguna, Branch 24, ordering said court to hear the case on 30 January 1989 and thereafter resolve the petition. At the conclusion of the hearing, or on 1 February 1989, the Presiding Judge of the Regional Trial Court of Bian, Laguna issued a resolution denying the petition for habeas corpus, it appearing that the said Narciso Nazareno is in the custody of the respondents by reason of an information filed against him with the Regional Trial Court of Makati, Metro Manila which liad taken cognizance of said case and had, in fact, denied the motion for bail filed by said Narciso Nazareno (presumably because of the strength of the evidence against him). This Court reiterates that shortly after the arrests of Espiritu and Nazareno, the corresponding informations against them were filed in court. The arrests of Espiritu and Nazareno were based on probable cause and supported by factual circumstances. They complied with conditions set forth in Section 5(b) of Rule 113. They were not arbitrary or whimsical arrests. Parenthetically, it should be here stated that Nazareno has since been convicted by the court a quo for murder and sentenced to reclusion perpetua. He has appealed the judgment of conviction to the Court of Appeals where it is pending as of this date ( CA-G.R. No. still undocketed). Petitioners contend that the decision of 9 July 1990 ignored the contitution requisiteds for admissibility of an extrajudicial admission. In the case of Buenaobra (G.R. Nos. 84581-82), he admitted 30 that he was an NPA courier. On the other hand, in the case of Amelia Roque, she admitted 31 that the unlicensed firearms, ammunition and subversive documents found in her possession during her arrest, belonged to her. The Court, it is true, took into account the admissions of the arrested persons of their membership in the CPP/NPA, as well as their ownership of the unlicensed firearms, ammunitions and documents in their possession. But again, these admissions, as revealed by the records, strengthen the Court's perception that truly the grounds upon which the arresting officers based their arrests without warrant, are supported by probable cause, i.e. that the persons arrested were probably guilty of the commission of certain offenses, in compliance with Section 5, Rule 113 of the Rules of Court. To note these admissions, on the other hand,

is not to rule that the persons arrested are already guilty of the offenses upon which their warrantless arrests were predicated. The task of determining the guilt or innocence of persons arrested without warrant is not proper in a petition for habeas corpus. It pertains to the trial of the case on the merits. As to the argument that the doctrines in Garcia vs. Enrile, and Ilagan vs. Enrile should be abandoned, this Court finds no compelling reason at this time to disturb the same, particularly ln the light of prevailing conditions where national security and liability are still directly challenged perhaps with greater vigor from the communist rebels. What is important is that everv arrest without warrant be tested as to its legality via habeas corpus proceeding. This Court. will promptly look into and all other appropriate courts are enjoined to do the same the legality of the arrest without warrant so that if the conditions under Sec. 5 of Rule 113, Rules of Court, as elucidated in this Resolution, are not met, then the detainee shall forthwith be ordered released; but if such conditions are met, then the detainee shall not be made to languish in his detention but must be promptly tried to the end that he may be either acquitted or convicted, with the least delay, as warranted by the evidence. A Final Word This Resolution ends as it began, reiterating that mere suspicion of being a Communist Party member or a subversive is absolutely not a ground for the arrest without warrant of the suspect. The Court predicated the validity of the questioned arrests without warrant in these petitions, not on mere unsubstantiated suspicion, but on compliance with the conditions set forth in Section 5, Rule 113, Rules of Court, a long existing law, and which, for stress, are probable cause and good faith of the arresting peace officers, and, further, on the basis of, as the records show, the actual facts and circumstances supporting the arrests. More than the allure of popularity or palatability to some groups, what is important is that the Court be right. ACCORDINGLY, the motions for reconsideration of the decision dated 9 July 1990, are DENIED. This denial is FINAL. SO ORDERED.

G.R. No. 93239 March 18, 1991 PEOPLE OF THE PHILIPPINES, plaintiff-appellee, vs. EDISON SUCRO, accused-appellant. The Solicitor General for plaintiff-appellee. Fidencio S. Raz for accused-appellant.

GUTIERREZ, JR., J.:p Edison Sucro was charged with and convicted of violation of Section 4, Article II of the Dangerous Drugs Act, under an Information which reads: That on or about the 21st day of March, 1989, in the evening, in the Poblacion, Municipality of Kalibo, Province of Aklan, Republic of the Philippines, and within the jurisdiction of this Honorable Court, the above-named accused, acting as a pusher or broker in the business of selling, administering, delivery, giving away to another and/or distributing prohibited drugs, did then and there wilfully, unlawfully and feloniously and without authority of law have in his possession and control nineteen (19) pieces of marijuana cigarette sticks and four (4) tea bags of dried marijuana leaves which were confiscated from him by the police authorities of Kalibo, Aklan, shortly after having sold one tea bag of dried marijuana leaves to a customer. (Rollo, p. 9) Upon arraignment, the accused-appellant, assisted by counsel, entered a plea of "not guilty" to the offense charged. Trial ensued and a judgment of conviction was rendered, the pertinent portion of which reads: WHEREFORE, judgment is rendered finding the accused Edison Sucro guilty of the sale of prohibited drug under Section 4, Article II of the Dangerous Drug Act, as amended, and sentencing him to suffer the penalty of life imprisonment, and pay a fine of P20,000, and costs. He shall be entitled to full credit in the service of his sentence with the period for which he has undergone preventive imprisonment to the date of promulgation of this judgment. All the items of marijuana confiscated in this case are declared forfeited in favor of the State. (Rollo, p. 41) From the foregoing judgment of conviction, accused-appellant interposes this appeal, assigning the following as errors allegedly committed by the court a quo, to wit: I

THE LOWER COURT ERRED IN ADMITTING AS EVIDENCE FOR THE PROSECUTION EXHIBITS "E"-"E-4", TEA BAGS OF ALLEGED MARIJUANA, TO BE THE CORPUS DELICTI; FURTHERMORE, THAT THE SAME WERE TAKEN WITHOUT THE REQUIRED WARRANT OF SEARCH AND ARREST SINCE THE ACCUSED WAS NOT IN THE ACT OF COMMITTING ANY OFFENSE AT THE TIME OF HIS ARREST. II THE LOWER COURT ERRED IN FINDING THE ACCUSED EDISON SUCRO GUILTY OF THE SALE OF PROHIBITED DRUGS UNDER SECTION 4, ARTICLE II, OF THE DANGEROUS DRUGS ACT AND SENTENCING HIM TO SUFFER A PENALTY OF LIFE IMPRISONMENT AND TO PAY A FINE OF P 20,000.00. (Appellant's Brief, p. 1) The antecedent facts of the case as summarized by the Solicitor General are as follows: On March 21, 1989, Pat. Roy Fulgencio, a member of the INP, Kalibo, Aklan, was instructed by P/Lt. Vicente Seraspi, Jr. (Station Commander of the INP Kalibo, Aklan) to monitor the activities of appellant Edison Sucro, because of information gathered by Seraspi that Sucro was selling marijuana. (p. 6, TSN, May 2,1989). As planned, at about 5:00 P.M. on said date, Pat. Fulgencio Positioned himself under the house of a certain Arlie Regalado at C. Quimpo Street. Adjacent to the house of Regalado, about 2 meters away, was a chapel. Thereafter, Pat. Fulgencio saw appellant enter the chapel, taking something which turned out later to be marijuana from the compartment of a cart found inside the chapel, and then return to the street where he handed the same to a buyer, Aldie Borromeo. After a while appellant went back to the chapel and again came out with marijuana which he gave to a group of persons. (pp. 6-8, 15-18, Ibid). It was at this instance that Pat. Fulgencio radioed P/Lt. Seraspi and reported the activity going on. P/Lt. Seraspi instructed Pat. Fulgencio to continue monitoring developments. At about 6:30 P.M., Pat. Fulgencio again called up Seraspi to report that a third buyer later Identified as Ronnie Macabante, was transacting with appellant. (pp. 18-19, Ibid) At that point, the team of P/Lt. Seraspi proceeded to the area and while the police officers were at the Youth Hostel at Maagma St., Pat. Fulgencio told P/Lt. Seraspi to intercept Macabante and appellant. P/Lt. Seraspi and his team caught up with Macabante at the crossing of Mabini and Maagma Sts. in front of the Aklan Medical Center. Upon seeing the police, Macabante threw something to the ground which turned out to be a tea bag of marijuana. (pp. 6-8, TSN, June 19, 1989) When confronted, Macabante readily admitted that he bought the same from appellant (Edison Sucro) in front of the chapel. (p. 6, TSN, May 24,

1989) The police team was able to overtake and arrest appellant at the corner of C. Quimpo and Veterans Sts. The police recovered 19 sticks and 4 teabags of marijuana from the cart inside the chapel and another teabag from Macabante, The teabags of marijuana were sent to the PC-INP Crime Laboratory Service, at Camp Delgado, Iloilo City for analysis. The specimens (Exhibits "G" to "G-18", Exhibits "E" to "E-4") were all found positive of marijuana. (pp. 47, TSN, Sept. 4, 1989)" (Appellee's Brief, pp. 3-6) As can be seen from the facts, the issue hinges mainly on whether or not the arrest without warrant of the accused is lawful and consequently, whether or not the evidence resulting from such arrest is admissible. We rule in the affirmative. The accused-appellant contends that his arrest was illegal, being a violation of his rights granted under Section 2, Article III of the 1987 Constitution. He stresses that there was sufficient time for the police officers to apply for a search and arrest warrants considering that Fulgencio informed his Station Commander of the activities of the accused two days before March 21, 1989, the date of his arrest. This contention is without merit. Section 5, Rule 113 of the Rules on Criminal Procedure provides for the instances where arrest without warrant is considered lawful. The rule states: Arrest without warrant, when lawful. A peace officer or private person may, without warrant, arrest a person: (a) When in his presence, the person to be arrested has committed, is actually committing, or is attempting to commit an offense; (b) When an offense has in fact just been committed, and he has personal knowledge of facts indicating that the person to be arrested has committed it; (Emphasis supplied) An offense is committed in the presence or within the view of an officer, within the meaning of the rule authorizing an arrest without a warrant, when the officer sees the offense, although at a distance, or hears the disturbances created thereby and proceeds at once to the scene thereof. (U.S. v. Fortaleza, 12 Phil. 472 [1909]; and U.S. v. Samonte, 16 Phil. 516 [1910]) The records show that Fulgencio went to Arlie Regalado's house at C. Quimpo Street to monitor the activities of the accused who was earlier reported to be selling marijuana at a chapel two (2) meters away from Regalado's house.

Fulgencio, within a distance of two meters saw Sucro conduct his nefarious activity. He saw Sucro talk to some persons, go inside the chapel, and return to them and exchange some things. These, Sucro did three times during the time that he was being monitored. Fulgencio would then relay the on-going transaction to P/Lt. Seraspi. Anent the second requirement, the fact that Macabante, when intercepted by the police, was caught throwing the marijuana stick and when confronted, readily admitted that he bought the same from accused-appellant clearly indicates that Sucro had just sold the marijuana stick to Macabante, and therefore, had just committed an illegal act of which the police officers had personal knowledge, being members of the team which monitored Sucro's nefarious activity. The court earlier indicated in the case of People v. Bati (G.R. No. 87429, August 27, 1990) that police officers have personal knowledge of the actual commission of the crime when it had earlier conducted surveillance activities of the accused. Thus, it stated: When Luciano and Caraan reached the place where the alleged transaction would take place and while positioned at a street comer, they saw appellant Regalado Bati and Warner Marquez by the side of the street about forty to fifty meters away from them (the public officers). They saw Marquez giving something to Bati, who, thereafter handed a wrapped object to Marquez who then inserted the object inside the front of his pants in front of his abdomen while Bati, on his part, placed the thing given to him inside his pocket. (p. 2) xxx xxx xxx . . . Both Patrolman Luciano and Caraan actually witnessed the same and their testimonies were based on their actual and personal knowledge of the events that took place leading to appellant's arrest. They may not have been within hearing distance, specially since conversation would expectedly be carried on in hushed tones, but they were certainly near enough to observe the movements of the appellant and the buyer. Moreover, these prosecution witnesses are all law enforcers and are, therefore, presumed to have regularly performed their duties in the absence of proof to the contrary (People v. Bati, supra citing People v. Agapito, G.R. No. 73786, October 12, 1987) The accused questions the failure of the police officers to secure a warrant considering that Fulgencio himself knew of Sucro's activities even prior to the former's joining the police force. Fulgencio reported Sucro's activities only three days before the incident. As the records reveal, Fulgencio and Sucro had known each other since their childhood years and that after Fulgencio joined the police force, he told the accused-appellant not to sell drugs in their locality. Hence, it is possible that

because of this friendship, Fulgencio hesitated to report his childhood friend and merely advised him not to engage in such activity. However, because of reliable information given by some informants that selling was going on everyday, he was constrained to report the matter to the Station Commander. On the other hand, the failure of the police officers to secure a warrant stems from the fact that their knowledge acquired from the surveillance was insufficient to fulfill the requirements for the issuance of a search warrant. What is paramount is that probable cause existed. Thus, it has been held in the case of People v. Lo Ho Wing, et al. (G.R. No. 88017, January 21, 1991): In the instant case, it was firmly established from the factual findings of the trial court that the authorities had reasonable ground to believe that appellant would attempt to bring in contraband and transport it within the country. The belief was based on intelligence reports gathered from surveillance activities on the suspected syndicate, of which appellant was touted to be a member. Aside from this, they were also certain as to the expected date and time of arrival of the accused from China. But such knowledge was clearly insufficient to enable them to fulfill the requirements for the issuance of a search warrant. Still and all, the important thing is that there was probable cause to conduct the warrantless search, which must still be present in such a case. As the Solicitor General has pointed out: There are several instances when a warrantless search and seizure can be effected without necessarily being preceded by an arrest provided the same is effected on the basis of probable cause (e.g. stop and search without warrant at checkpoints). Between warrantless searches and seizures at checkpoints and in the case at bar the latter is more reasonable considering that unlike in the former, it was effected on the basis of probable cause. Under the circumstances (monitoring of transactions) there existed probable cause for the arresting officers, to arrest appellant who was in fact selling marijuana and to seize the contraband. That searches and seizures must be supported by a valid warrant is not an absolute rule (Manipon, Jr. v. Sandiganbayan, 143 SCRA 267 [1986]). Among the exceptions granted by law is a search incidental to a lawful arrest under Sec. 12, Rule 126 of the Rules on Criminal Procedure, which provides that a person lawfully arrested may be searched for dangerous weapons or anything which may be used as proof of the commission of an offense, without a search warrant. (People v. Castiller, G.R. No. 87783, August 6, 1990) The accused-appellant claims that the arrest having been done without warrant, it follows that the evidence obtained therefrom is inadmissible. As earlier discussed, there is nothing unlawful about the arrest considering its

compliance with the requirements of a warrantless arrest. Ergo, the fruits obtained from such lawful arrest are admissible in evidence. Edison Sucro assails the trial court's reliance on the statement of Macabante whose reason for testifying could be merely to escape prosecution. We quote the trial court's finding as to the testimony of Macabante: The non-filing of a complaint against him for possession of marijuana may have been the reason of (sic) his willingness to testify in court against the accused. But this does not necessarily taint the evidence that proceeds from his lips. As explained by Lt. Seraspi, the best sources of information against drug pushers are usually their customers, especially if as in this case, there is no other direct evidence of the selling except the testimony of the buyer. We accept this observation as a realistic appraisal of a situation in which drug users are, and should be employed by law enforcement authorities to bolster the drive against pushers who are the real felons in our society. We have observed the demeanor of the witness in court, and found him to be straightforward, unhesitating, and spontaneous in his declarations, so that we are satisfied as to his intention and disposition to tell the truth (Rollo, p. 40) Time and again it has been held that the findings of the trial court are entitled to great weight and should not be disturbed on appeal unless it is shown that the trial court had overlooked certain facts of weight and importance, it being acknowledged. that the court below, having seen and heard the witnesses during the trial, is in a better position to evaluate their testimonies (People v. Umali, et al., G.R. No. 84450, February 4, 1991 citing People v. Alvarez, 163 SCRA 745 [1988]; People v. Dorado, 30 SCRA 53 [1969]; and People v. Espejo, 36 SCRA 400 [1970]). Furthermore, the testimony of Macabante was corroborated on material points by public officers Fulgencio and Seraspi. There is nothing in the record to suggest that the police officers were compelled by any motive than to accomplish their mission to capture a drug pusher in the execution of the crime, the presumption being that police officers perform their duties regularly in the absence of any evidence to the contrary (Rule 131, Sec. 3(m), Revised Rules on Evidence; People v. Castiller, supra citing People v. Natipravat, 145 SCRA 483 [1986]). The prosecution evidence was further bolstered by the findings of the Forensic Chemist that the items seized were all positive for marijuana. In contrast to the evidence presented by the prosecution, accused-appellant's defense is alibi which is unavailing considering that he was positively identified by Macabante to be the person from whom he bought marijuana.

Sucro alleges that he could not have committed the crime since he was with his uncle and cousin distributing handbills for his Auntie's candidacy. The fact, however, remains that it does not preclude the possibility that he was present in the vicinity as established by his admission that he moved a lot and even had the occasion to meet Macabante on the street. It is well-settled that mere denials cannot prevail against the positive identification of the appellant as the seller of the prohibited substances. (People v. Khan, 161 SCRA 406 [1988]; and People v. Paco, 170 SCRA 681 [1989]) Premises considered, this Court is convinced that appellant Edison Sucro had indeed committed the offense charged. The trial court's decision must be upheld. WHEREFORE, the decision appealed from is hereby AFFIRMED. SO ORDERED.

G.R. No. 95902 February 4, 1992 PEOPLE OF THE PHILIPPINES, plaintiff-appellee, vs. DON RODRIGUEZA, accused-appellant. The Solicitor General for plaintiff-appellee. Public Attorney's Office for accused-appellant.

REGALADO, J.: On appeal before us is the decision of the Regional Trial Court of Legaspi City, Branch 10, finding accused-appellant Don Rodrigueza guilty beyond reasonable doubt of violating Section 4, Article II of the Dangerous Drugs Act of 1972 (Republic Act No. 6425, as amended) and sentencing him to suffer the penalty of life imprisonment and to pay a fine of P20,000.00 and costs. 1 However, the Solicitor General, deviating from his conventional stance in the prosecution of criminal cases, recommends the acquittal of appellant for the reasons stated in his Manifestation for Acquittal (In Lieu of Appellee's Brief) filed with the Court. We have reviewed and analyzed the testimonial and documentary evidence in this case and we find said recommendation to be well taken. The information, dated July 10, 1987, charges Don Rodrigueza and his coaccused, Samuel Segovia and Antonio Lonceras, with allegedly having in their custody and possession 100 grams of marijuana leaves and for selling, in a buybust operation, said 100 grams of dried marijuana leaves for a consideration of P200.00. 2 During the arraignment, all the accused pleaded not guilty to the charge against them. At the trial, the prosecution and the defense presented several witnesses after which the court a quo rendered judgment acquitting Samuel Segovia and Antonio Lonceras but convicting and penalizing herein appellant as hereinbefore stated. The following facts are culled from the decision of the trial court and the evidence presented by the prosecution. At around 5:00 o'clock in the afternoon of July 1, 1987, CIC Ciriaco Taduran was in their headquarters at the Office of the Narcotics Regional Unit at Camp Bagong Ibalon, Legaspi City, together with S/Sgt. Elpidio Molinawe, CIC Leonardo B. Galutan and their commanding officer, Major Crisostomo M. Zeidem, when a confidential informer arrived and told them that there was an ongoing illegal traffic of prohibited drugs in Tagas, Daraga, Albay. Major Zeidem

formed a team to conduct a buy-bust operation, which team was given P200.00 in different denominations to buy marijuana. These bills were treated with ultraviolet powder at the Philippine Constabulary Crime Laboratory (PCCL). Sgt. Molinawe gave the money to Taduran who acted as the poseur buyer. He was told to look for a certain Don, the alleged seller of prohibited drugs. Taduran went to Tagas alone and, while along the road, he met Samuel Segovia. He asked Segovia where be could find Don and where he could buy marijuana. Segovia left for a while and when be returned, he was accompanied by a man who was later on introduced to him as Don, herein appellant. 3 After agreeing on the price of P200.00 for 100 grams of marijuana, Don halted a passing tricycle driven by Antonio Lonceras. He boarded it and left Taduran and Segovia. When he came back, Don gave Taduran "a certain object wrapped in a plastic" which was later identified as marijuana, and received payment therefor. Thereafter, Taduran returned to the headquarters and made a report regarding his said purchase of marijuana. 4 Based on that information, Major Zeidem ordered a team to conduct an operation to apprehend the suspects. In the evening of the same date, CIC Galutan and S/Sgt. Molinawe proceeded to Regidor Street, Daraga, Albay and arrested appellant, Antonio Lonceras and Samuel Segovia. The constables were not, however, armed with a warrant of arrest when they apprehended the three accused. The arrestees were brought to the headquarters for investigation. 5 Thereafter, agents of the Narcotics Command (NARCOM) conducted a raid in the house of Jovencio Rodrigueza, father of appellant. Taduran did not go with them. During the raid, they were able to confiscate dried marijuana leaves and a plastic syringe, among others. The search, however, was not authorized by any search warrant. 6 The next day, July 2, 1987, Jovencio Rodrigueza was released from detention but appellant was detained. An affidavit, allegedly taken from and executed by him, was sworn to by him before the assistant city prosecutor. Appellant had no counsel when his sworn statement was taken during that custodial investigation. The arrestees were also examined by personnel of the PCCL and were found positive for ultraviolet powder. 7 The three accused presented different versions of their alleged participations. Samuel Segovia testified that he was in their house in the evening of July 1, 1987 listening to the radio. Later, he ate his merienda and then went out to buy cigarettes from the store. While he was at the store, a jeep stopped behind him. Several armed men alighted therefrom and ordered him to get inside the jeep. He refused but he was forced to board the vehicle. He was even hit by the butt of a gun. 8

He was thereafter brought to Camp Bagong Ibalon where he was investigated and was repeatedly asked regarding the whereabouts of Rodrigueza. He was manhandled by the NARCOM agents and was detained while inside the camp. He was then made to hold a P10.00 bill treated with ultraviolet powder. When he was taken to the PCCL and examined he was found positive of the ultraviolet powder. He was also made to sign some papers but he did not know what they were all about. 9 Appellant, on the other hand, testified that on said date he was in the house of his aunt in San Roque, Legaspi City. He stayed there overnight and did not leave the place until the next day when his brother arrived and told him that their father was taken by some military men the preceding night. Appellant went to Camp Bagong Ibalon and arrived there at around 8:00 o'clock in the morning of July 2, 1987. When he arrived, he was asked if he knew anything about the marijuana incident, to which question he answered in the negative. Like Segovia, he was made to hold a P10.00 bill and was brought to the crime laboratory for examination. From that time on, he was not allowed to go home and was detained inside the camp. He was also tortured in order to make him admit his complicity in the alleged sale of marijuana. 10 In the assignment of errors in his brief, appellant contends that the trial court erred in (1) admitting in evidence the sworn statement of appellant which was obtained in violation of his constitutional rights; (2) convicting appellant of the crime charged despite the fact that the 100 grams of dried marijuana leaves allegedly bought from him were not properly identified; (3) convicting appellant of the crime charged despite the fact that the evidence for the prosecution is weak and not convincing; and (4) finding appellant guilty beyond reasonable doubt of selling or at least acting as broker in the sale of the 100 grams of marijuana to CIC Taduran late in the afternoon of July 1, 1987, despite the failure of the prosecution to prove his guilt beyond reasonable doubt. 11 We rule for the appellant and approve the recommendation for his acquittal. In disposing of this case, however, we feel that the issues raised by appellant should properly be discussed seriatim. 1. A buy-bust operation is a form of entrapment employed by peace officers to trap and catch a malefactor in flagrante delicto. 12 Applied to the case at bar, the term in flagrante delicto requires that the suspected drug dealer must be caught redhanded in the act of selling marijuana or any prohibited drug to a person acting or posing as a buyer. In the instant case, however, the procedure adopted by the NARCOM agents failed to meet this qualification. Based on the very evidence of the prosecution, after the alleged consummation of the sale of dried marijuana leaves, CIC Taduran immediately released appellant Rodrigueza instead of arresting and taking him into his custody. This act of CIC Taduran, assuming arguendo that the

supposed sale of marijuana did take place, is decidedly contrary to the natural course of things and inconsistent with the aforestated purpose of a buy-bust operation. It is rather absurd on his part to let appellant escape without having been subjected to the sanctions imposed by law. It is, in fact, a dereliction of duty by an agent of the law. 2. The admissibility of the sworn statement allegedly executed by appellant was squarely placed in issue and, as correctly pointed out by the defense, said sworn statement is inadmissible in evidence against appellant. We have once again to reiterate and emphasize that Article III of the 1987 Constitution provides: Sec. 12 (1). Any person under investigation for the commission of an offense shall have the right to be informed of his right to remain silent and to have a competent and independent counsel preferably of his own choice. If the person cannot afford the services of counsel, he must be provided with one. These rights cannot be waived except in writing and in the presence of counsel. xxx xxx xxx (3) Any confession or admission obtained in violation of this or section 17 hereof shall be inadmissible in evidence against him. An examination of said sworn statement shows that appellant was informed of his constitutional right to remain silent and to be assisted by counsel during custodial examination. He was also asked if he was waiving his right to be assisted by counsel and he answered in the affirmative. However, while the rights of a person under custodial investigation may be waived, such waiver must be made not only voluntarily, knowingly and intelligently but also in the presence and with the assistance of counsel. 13 In the present case, the waiver made by appellant being without the assistance of counsel, this omission alone is sufficient to invalidate said sworn statement. 14 3. Corollary to this, we take cognizance of the error of the trial court in admitting in evidence against appellant the articles allegedly confiscated during the raid conducted in the house of Jovencio Rodrigueza. As provided in the present Constitution, a search, to be valid, must generally be authorized by a search warrant duly issued by the proper government authority. 15 True, in some instances, this Court has allowed government authorities to conduct searches and seizures even without a search warrant. Thus, when the owner of the premises waives his right against such incursion; 16 when the search is incidental to a lawful arrest; 17 when it is made on vessels and aircraft for violation of customs laws; 18 when it is made on automobiles for the purpose of preventing violations of smuggling or immigration laws; 19 when it involves

prohibited articles in plain view; 20 or in cases of inspection of buildings and other premises for the enforcement of fire, sanitary and building regulations, 21 a search may be validly made even without a search warrant. In the case at bar, however, the raid conducted by the NARCOM agents in the house of Jovencio Rodrigueza was not authorized by any search warrant. It does not appear, either, that the situation falls under any of the aforementioned cases. Hence, appellant's right against unreasonable search and seizure was clearly violated. The NARCOM agents could not have justified their act by invoking the urgency and necessity of the situation because the testimonies of the prosecution witnesses reveal that the place had already been put under surveillance for quite some time. Had it been their intention to conduct the raid, then they should, because they easily could, have first secured a search warrant during that time. 4. The Court further notes the confusion and ambiguity in the identification of the confiscated marijuana leaves and other prohibited drug paraphernalia presented as evidence against appellant. CIC Taduran, who acted as the poseur buyer, testified that appellant sold him 100 grams of dried marijuana leaves wrapped in a plastic bag. Surprisingly, and no plausible explanation has been advanced therefor, what were submitted to and examined by the PCCL and thereafter utilized as evidence against the appellant were the following items: One (1) red and white colored plastic bag containing the following: Exh. "A"Thirty (30) grams of suspected dried marijuana fruiting tops contained inside a transparent plastic bag. Exh. "B" Fifty (50) grams of suspected dried marijuana leaves and seeds contained inside a white colored plastic labelled "Robertson". Exh. "C" Four (4) aluminum foils each containing suspected dried marijuana fruiting tops having a total weight of seven grams then further wrapped with a piece of aluminum foil. Exh. "D" Five (5) small transparent plastic bags each containing suspected dried marijuana fruiting tops having a total weight of seventeen grams. Exh. "E" One plastic syringe. 22 Evidently, these prohibited articles were among those confiscated during the socalled follow-up raid in the house of Jovencio Rodrigueza. The unanswered question then arises as to the identity of the marijuana leaves that became the basis of appellant's conviction. 23 In People vs. Rubio, 24 this Court had the

occasion to rule that the plastic bag and the dried marijuana leaves contained therein constitute the corpus delicti of the crime. As such, the existence thereof must be proved with certainty and conclusiveness. Failure to do so would be fatal to the cause of the prosecution. 5. It is accepted that, as a rule, minor inconsistencies in the testimony of a witness will not affect his credibility. It even enhances such credibility because it only shows that he has not been rehearsed. 25 However, when the inconsistencies pertain to material and crucial points, the same detract from his overall credibility. The exception, rather than the rule, applies in the case at bar. As correctly pointed out by the Solicitor General, the testimonies of the prosecution witnesses are tainted with serious flaws and material inconsistencies rendering the same incredible. 26 CIC Taduran, in his testimony, said that they had already been conducting surveillance of the place where the buy-bust operation was to take place. It turned out, however, that he did not even know the exact place and the identity of the person from whom he was to buy marijuana leaves. Thus: FISCAL TOLOSA Q What place in Tagas were you able to go (to)? WITNESS A I am not actually familiar in (sic) that place, in Tagas, although we occasionally passed there. Q Now, upon your arrival in Tagas, what did you do that afternoon? A I waited for the suspect because previously, we have already been conducted (sic) surveylance (sic) in the vicinity. Q Upon arrival in Tagas, were you able to see the suspect? A By the road, sir. Q Who was the first person did you see (sic) when you arrived at Tagas? A The first person whom I saw is Samuel Segovia. Q Were you able to talk with this Samuel Segovia? A According to him, we could get some. 27

The same findings go for the testimony of witness Galutan. In his direct examination, he declared that they arrested the three accused all at the same time on the fateful night of July 1, 1987. But, in his cross-examination and as corroborated by the Joint Affidavit of Arrest 28 submitted by him and Molinawe, it appeared that Lonceras and Segovia were arrested on different times and that appellant Don Rodrigueza was not among those who were arrested. Instead, it was Jovencio Rodrigueza, Don's father, who was picked up at a much later time. With said inconsistencies in sharp focus, we are constrained to give more credibility to the testimony of appellant Rodrigueza. While it is true that appellant's defense amounts to an alibi, and as such is the weakest defense in a criminal prosecution, there are, nonetheless, some evidentiary aspects pointing to the truth in his testimony. Firstly, the Joint Affidavit of Arrest corroborates his testimony that he was not among those who were arrested on the night of July 1, 1987. His co-accused Segovia also testified that appellant Rodrigueza was not with them when they were apprehended by the NARCOM agents. Secondly, the apparent motive of the NARCOM agents in prosecuting the accused was also revealed during the trial of the case. Rebuttal witnesses Gracita Bahillo, sister of appellant, and Hospicio Segovia, father of Samuel Segovia, testified that Sgt. Molinawe, who has since been reportedly dismissed from the service, asked for P10,000.00 from each of them in exchange for the liberty of the accused. 29 This allegation was never refuted by the prosecution. Hence, the rule laid down by this Court that the statements of prosecution witnesses are entitled to full faith and credit 30 has no application in the case at bar. Finally, the Court has repeatedly ruled that to sustain the conviction of the accused, the prosecution must rely on the strength of its own evidence and not on the weakness of the defense. 31 As clearly shown by the evidence, the prosecution has failed to establish its cause. It has not overcome the presumption of innocence accorded to appellant. This being the case, appellant should not be allowed to suffer for unwarranted and imaginary imputations against him. WHEREFORE, the judgment of conviction of the court below is hereby REVERSED and SET ASIDE and accused-appellant Don Rodrigueza is hereby ACQUITTED of the crime charged. It is hereby ordered that he be immediately released from custody unless he is otherwise detained for some other lawful cause. SO ORDERED.

G.R. No. 101837 February 11, 1992 ROLITO GO y TAMBUNTING, petitioner, vs. THE COURT OF APPEALS, THE HON. BENJAMIN V. PELAYO, Presiding Judge, Branch 168, Regional Trial Court, NCJR Pasig, M.M., and PEOPLE OF THE PHILIPPINES, respondents.

FELICIANO, J.: According to the findings of the San Juan Police in their Investigation Report, 1 on 2 July 1991, Eldon Maguan was driving his car along Wilson St., San Juan, Metro Manila, heading towards P. Guevarra St. Petitioner entered Wilson St., where it is a one-way street and started travelling in the opposite or "wrong" direction. At the corner of Wilson and J. Abad Santos Sts., petitioner's and Maguan's cars nearly bumped each other. Petitioner alighted from his car, walked over and shot Maguan inside his car. Petitioner then boarded his car and left the scene. A security guard at a nearby restaurant was able to take down petitioner's car plate number. The police arrived shortly thereafter at the scene of the shooting and there retrieved an empty shell and one round of live ammunition for a 9 mm caliber pistol. Verification at the Land Transportation Office showed that the car was registered to one Elsa Ang Go. The following day, the police returned to the scene of the shooting to find out where the suspect had come from; they were informed that petitioner had dined at Cravings Bake Shop shortly before the shooting. The police obtained a facsimile or impression of the credit card used by petitioner from the cashier of the bake shop. The security guard of the bake shop was shown a picture of petitioner and he positively identified him as the same person who had shot Maguan. Having established that the assailant was probably the petitioner, the police launched a manhunt for petitioner. On 8 July 1991, petitioner presented himself before the San Juan Police Station to verify news reports that he was being hunted by the police; he was accompanied by two (2) lawyers. The police forthwith detained him. An eyewitness to the shooting, who was at the police station at that time, positively identified petitioner as the gunman. That same day, the police promptly filed a complaint for frustrated homicide 2 against petitioner with the Office of the Provincial Prosecutor of Rizal. First Assistant Provincial Prosecutor Dennis Villa Ignacio ("Prosecutor") informed petitioner, in the presence of his lawyers, that he could avail himself of his right to preliminary investigation but that he must first sign a waiver of the provisions of Article 125 of the Revised Penal Code. Petitioner refused to execute any such waiver. On 9 July 1991, while the complaint was still with the Prosecutor, and before an

information could be filed in court, the victim, Eldon Maguan, died of his gunshot wound(s). Accordingly, on 11 July 1991, the Prosecutor, instead of filing an information for frustrated homicide, filed an information for murder 3 before the Regional Trial Court. No bail was recommended. At the bottom of the information, the Prosecutor certified that no preliminary investigation had been conducted because the accused did not execute and sign a waiver of the provisions of Article 125 of the Revised Penal Code. In the afternoon of the same day, 11 July 1991, counsel for petitioner filed with the Prosecutor an omnibus motion for immediate release and proper preliminary investigation, 4 alleging that the warrantless arrest of petitioner was unlawful and that no preliminary investigation had been conducted before the information was filed. Petitioner also prayed that he be released on recognizance or on bail. Provincial Prosecutor Mauro Castro, acting on the omnibus motion, wrote on the last page of the motion itself that he interposed no objection to petitioner being granted provisional liberty on a cash bond of P100,000.00. On 12 July 1991, petitioner filed an urgent ex-parte motion for special raffle 5 in order to expedite action on the Prosecutor's bail recommendation. The case was raffled to the sala of respondent Judge, who, on the same date, approved the cash bond 6 posted by petitioner and ordered his release. 7 Petitioner was in fact released that same day. On 16 July 1991, the Prosecutor filed with the Regional Trial Court a motion for leave to conduct preliminary investigation 8 and prayed that in the meantime all proceedings in the court be suspended. He stated that petitioner had filed before the Office of the Provincial Prosecutor of Rizal an omnibus motion for immediate release and preliminary investigation, which motion had been granted by Provincial Prosecutor Mauro Castro, who also agreed to recommend cash bail of P100,000.00. The Prosecutor attached to the motion for leave a copy of petitioner's omnibus motion of 11 July 1991. Also on 16 July 1991, the trial court issued an Order 9 granting leave to conduct preliminary investigation and cancelling the arraignment set for 15 August 1991 until after the prosecution shall have concluded its preliminary investigation. On 17 July 1991, however, respondent Judge motu proprio issued an Order, 10 embodying the following: (1) the 12 July 1991 Order which granted bail was recalled; petitioner was given 48 hours from receipt of the Order to surrender himself; (2) the 16 July 1991 Order which granted leave to the prosecutor to conduct preliminary investigation was recalled and cancelled; (3) petitioner's omnibus motion for immediate release and preliminary investigation dated 11 July 1991 was treated as a petition for bail and set for hearing on 23 July 1991.

On 19 July 1991, petitioner filed a petition for certiorari, prohibition and mandamus before the Supreme Court assailing the 17 July 1991 Order, contending that the information was null and void because no preliminary investigation had been previously conducted, in violation of his right to due process. Petitioner also moved for suspension of all proceedings in the case pending resolution by the Supreme Court of his petition; this motion was, however, denied by respondent Judge. On 23 July 1991, petitioner surrendered to the police. By a Resolution dated 24 July 1991, this Court remanded the petition for certiorari, prohibition and mandamus to the Court of Appeals. On 16 August 1991, respondent Judge issued an order in open court setting the arraignment of petitioner on 23 August 1991. On 19 August 1991, petitioner filed with the Court of Appeals a motion to restrain his arraignment. On 23 August 1991, respondent judge issued a Commitment Order directing the Provincial Warden of Rizal to admit petitioner into his custody at the Rizal Provincial Jail. On the same date, petitioner was arraigned. In view, however, of his refusal to enter a plea, the trial court entered for him a plea of not guilty. The Trial court then set the criminal case for continuous hearings on 19, 24 and 26 September; on 2, 3, 11 and 17 October; and on 7, 8, 14, 15, 21 and 22 November 1991. 11 On 27 August 1991, petitioner filed a petition for habeas corpus 12 in the Court of Appeals. He alleged that in view of public respondent's failure to join issues in the petition for certiorari earlier filed by him, after the lapse of more than a month, thus prolonging his detention, he was entitled to be released on habeas corpus. On 30 August 1991, the Court of Appeals issued the writ of habeas corpus. 13 The petition for certiorari, prohibition and mandamus, on the one hand, and the petition for habeas corpus, upon the other, were subsequently consolidated in the Court of Appeals. The Court of Appeals, on 2 September 1991, issued a resolution denying petitioner's motion to restrain his arraignment on the ground that that motion had become moot and academic. On 19 September 1991, trial of the criminal case commenced and the prosecution presented its first witness. On 23 September 1991, the Court of Appeals rendered a consolidated decision 14 dismissing the two (2) petitions, on the following grounds:

a. Petitioner's warrantless arrest was valid because the offense for which he was arrested and charged had been "freshly committed." His identity had been established through investigation. At the time he showed up at the police station, there had been an existing manhunt for him. During the confrontation at the San Juan Police Station, one witness positively identified petitioner as the culprit. b. Petitioner's act of posting bail constituted waiver of any irregularity attending his arrest. He waived his right to preliminary investigation by not invoking it properly and seasonably under the Rules. c. The trial court did not abuse its discretion when it issued the 17 July 1991 Order because the trial court had the inherent power to amend and control its processes so as to make them conformable to law and justice. d. Since there was a valid information for murder against petitioner and a valid commitment order (issued by the trial judge after petitioner surrendered to the authorities whereby petitioner was given to the custody of the Provincial Warden), the petition for habeas corpus could not be granted. On 3 October 1991, the prosecution presented three (3) more witnesses at the trial. Counsel for petitioner also filed a "Withdrawal of Appearance" 15 with the trial court, with petitioner's conformity. On 4 October 1991, the present Petition for Review on Certiorari was filed. On 14 October 1991, the Court issued a Resolution directing respondent Judge to hold in abeyance the hearing of the criminal case below until further orders from this Court. In this Petition for Review, two (2) principal issues need to be addressed: first, whether or not a lawful warrantless arrest had been effected by the San Juan Police in respect of petitioner Go; and second, whether petitioner had effectively waived his right to preliminary investigation. We consider these issues seriatim. In respect of the first issue, the Solicitor General argues that under the facts of the case, petitioner had been validly arrested without warrant. Since petitioner's identity as the gunman who had shot Eldon Maguan on 2 July 1991 had been sufficiently established by police work, petitioner was validly arrested six (6) days later at the San Juan Police Station. The Solicitor General invokes Nazareno v. Station Commander, etc., et al., 16 one of the seven (7) cases consolidated with In the Matter of the Petition for Habeas Corpus of Roberto Umil, etc., v. Ramos, et al. 17 where a majority of the Court upheld a warrantees arrest as valid although effected fourteen (14) days after the killing in connection with which Nazareno had been arrested. Accordingly, in the view of the Solicitor General, the provisions of Section 7, Rule 112 of the Rules of Court were applicable and because petitioner had declined to waive the provisions of Article 125 of the Revised Penal Code, the Prosecutor was legally justified in filing the information

for murder even without preliminary investigation. On the other hand, petitioner argues that he was not lawfully arrested without warrant because he went to the police station six (6) days after the shooting which he had allegedly perpetrated. Thus, petitioner argues, the crime had not been "just committed" at the time that he was arrested. Moreover, none of the police officers who arrested him had been an eyewitness to the shooting of Maguan and accordingly none had the "personal knowledge" required for the lawfulness of a warrantees arrest. Since there had been no lawful warrantless arrest. Section 7, Rule 112 of the Rules of Court which establishes the only exception to the right to preliminary investigation, could not apply in respect of petitioner. The reliance of both petitioner and the Solicitor General upon Umil v. Ramos is, in the circumstances of this case, misplaced. In Umil v. Ramos, by an eight-to-six vote, the Court sustained the legality of the warrantless arrests of petitioners made from one (1) to fourteen days after the actual commission of the offenses, upon the ground that such offenses constituted "continuing crimes." Those offenses were subversion, membership in an outlawed organization like the New People's Army, etc. In the instant case, the offense for which petitioner was arrested was murder, an offense which was obviously commenced and completed at one definite location in time and space. No one had pretended that the fatal shooting of Maguan was a "continuing crime." Secondly, we do not believe that the warrantees "arrest" or detention of petitioner in the instant case falls within the terms of Section 5 of Rule 113 of the 1985 Rules on Criminal Procedure which provides as follows: Sec. 5 Arrest without warrant; when lawful. A peace officer or a private person may, without warrant, arrest a person: (a) When, in his presence, the person to be arrested has committed, is actually committing, or is attempting to commit an offense; (b) When an offense has in fact just been committed, and he has personal knowledge of facts indicating that the person to be arrested has committed it; and (c) When the person to be arrested is a prisoner who has escaped from a penal establishment or place where he is serving final judgment or temporarily confined while his case is pending, or has escaped while being transferred from one confinement to another. In cases falling under paragraphs (a) and (b) hereof, the person arrested without a warrant shall be forthwith delivered to the nearest police station or jail, and he shall be proceed against in accordance with Rule 112, Section 7.

Petitioner's "arrest" took place six (6) days after the shooting of Maguan. The "arresting" officers obviously were not present, within the meaning of Section 5(a), at the time petitioner had allegedly shot Maguan. Neither could the "arrest" effected six (6) days after the shooting be reasonably regarded as effected "when [the shooting had] in fact just been committed" within the meaning of Section 5(b). Moreover, none of the "arresting" officers had any "personal knowledge" of facts indicating that petitioner was the gunman who had shot Maguan. The information upon which the police acted had been derived from statements made by alleged eyewitnesses to the shooting one stated that petitioner was the gunman; another was able to take down the alleged gunman's car's plate number which turned out to be registered in petitioner's wife's name. That information did not, however, constitute "personal knowledge." 18 It is thus clear to the Court that there was no lawful warrantless arrest of petitioner within the meaning of Section 5 of Rule 113. It is clear too that Section 7 of Rule 112, which provides: Sec. 7 When accused lawfully arrested without warrant. When a person is lawfully arrested without a warrant for an offense cognizable by the Regional Trial Court the complaint or information may be filed by the offended party, peace officer or fiscal without a preliminary investigation having been first conducted, on the basis of the affidavit of the offended party or arresting office or person However, before the filing of such complaint or information, the person arrested may ask for a preliminary investigation by a proper officer in accordance with this Rule, but he must sign a waiver of the provisions of Article 125 of the Revised Penal Code, as amended, with the assistance of a lawyer and in case of nonavailability of a lawyer, a responsible person of his choice. Notwithstanding such waiver, he may apply for bail as provided in the corresponding rule and the investigation must be terminated within fifteen (15) days from its inception. If the case has been filed in court without a preliminary investigation having been first conducted, the accused may within five (5) days from the time he learns of the filing of the information, ask for a preliminary investigation with the same right to adduce evidence in his favor in the manner prescribed in this Rule. (Emphasis supplied) is also not applicable. Indeed, petitioner was not arrested at all. When he walked into San Juan Police Station, accompanied by two (2) lawyers, he in fact placed himself at the disposal of the police authorities. He did not state that he was "surrendering" himself, in all probability to avoid the implication he was admitting that he had slain Eldon Maguan or that he was otherwise guilty of a crime. When the police filed a complaint for frustrated homicide with the Prosecutor, the latter should have immediately scheduled a preliminary investigation to determine whether there was probable cause for charging petitioner in court for the killing of Eldon Maguan. Instead, as noted earlier, the Prosecutor proceed under the

erroneous supposition that Section 7 of Rule 112 was applicable and required petitioner to waive the provisions of Article 125 of the Revised Penal Code as a condition for carrying out a preliminary investigation. This was substantive error, for petitioner was entitled to a preliminary investigation and that right should have been accorded him without any conditions. Moreover, since petitioner had not been arrested, with or without a warrant, he was also entitled to be released forthwith subject only to his appearing at the preliminary investigation. Turning to the second issue of whether or not petitioner had waived his right to preliminary investigation, we note that petitioner had from the very beginning demanded that a preliminary investigation be conducted. As earlier pointed out, on the same day that the information for murder was filed with the Regional Trial Court, petitioner filed with the Prosecutor an omnibus motion for immediate release and preliminary investigation. The Solicitor General contends that that omnibus motion should have been filed with the trial court and not with the Prosecutor, and that the petitioner should accordingly be held to have waived his right to preliminary investigation. We do not believe that waiver of petitioner's statutory right to preliminary investigation may be predicated on such a slim basis. The preliminary investigation was to be conducted by the Prosecutor, not by the Regional Trial Court. It is true that at the time of filing of petitioner's omnibus motion, the information for murder had already been filed with the Regional Trial Court: it is not clear from the record whether petitioner was aware of this fact at the time his omnibus motion was actually filed with the Prosecutor. In Crespo v. Mogul, 19 this Court held: The preliminary investigation conducted by the fiscal for the purpose of determining whether a prima facie case exists to warranting the prosecution of the accused is terminated upon the filing of the information in the proper court. In turn, as above stated, the filing of said information sets in motion the criminal action against the accused in Court. Should the fiscal find it proper to conduct a reinvestigation of the case, at such stage, the permission of the Court must be secured. After such reinvestigation the finding and recommendations of the fiscal should be submitted to the Court for appropriate action. While it is true that the fiscal has the quasi-judicial discretion to determine whether or not a criminal case should be filed in court or not, once the case had already been brought to Court whatever disposition the fiscal may feel should be proper in the case thereafter should be addressed for the consideration of the Court. The only qualification is that the action of the Court must not impair the substantial rights of the accused., or the right of the People to due process of law. xxx xxx xxx The rule therefore in this jurisdiction is that once a complaint or information is filed in Court any disposition of the case [such] as its dismissal or the conviction or acquittal of the accused rests in the sound discretion of the Court. Although the fiscal retains the direction and control of the prosecution of criminal cases

even while the case is already in Court he cannot impose his opinion on the trial court. The Court is the best and sole judge on what to do with the case before it. . . . 20 (Citations omitted; emphasis supplied) Nonetheless, since petitioner in his omnibus motion was asking for preliminary investigation and not for a re-investigation (Crespo v. Mogul involved a reinvestigation), and since the Prosecutor himself did file with the trial court, on the 5th day after filing the information for murder, a motion for leave to conduct preliminary investigation (attaching to his motion a copy of petitioner's omnibus motion), we conclude that petitioner's omnibus motion was in effect filed with the trial court. What was crystal clear was that petitioner did ask for a preliminary investigation on the very day that the information was filed without such preliminary investigation, and that the trial court was five (5) days later apprised of the desire of the petitioner for such preliminary investigation. Finally, the trial court did in fact grant the Prosecutor's prayer for leave to conduct preliminary investigation. Thus, even on the (mistaken) supposition apparently made by the Prosecutor that Section 7 of Rule 112 of the Revised Court was applicable, the 5day reglementary period in Section 7, Rule 112 must be held to have been substantially complied with. We believe and so hold that petitioner did not waive his right to a preliminary investigation. While that right is statutory rather than constitutional in its fundament, since it has in fact been established by statute, it is a component part of due process in criminal justice. 21 The right to have a preliminary investigation conducted before being bound over to trial for a criminal offense and hence formally at risk of incarceration or some other penalty, is not a mere formal or technical right; it is a substantive right. The accused in a criminal trial is inevitably exposed to prolonged anxiety, aggravation, humiliation, not to speak of expense; the right to an opportunity to avoid a process painful to any one save, perhaps, to hardened criminals, is a valuable right. To deny petitioner's claim to a preliminary investigation would be to deprive him the full measure of his right to due process. The question may be raised whether petitioner still retains his right to a preliminary investigation in the instant case considering that he was already arraigned on 23 August 1991. The rule is that the right to preliminary investigation is waived when the accused fails to invoke it before or at the time of entering a plea at arraignment. 22 In the instant case, petitioner Go had vigorously insisted on his right to preliminary investigation before his arraignment. At the time of his arraignment, petitioner was already before the Court of Appeals on certiorari, prohibition and mandamus precisely asking for a preliminary investigation before being forced to stand trial. Again, in the circumstances of this case, we do not believe that by posting bail petitioner had waived his right to preliminary investigation. In People v. Selfaison, 23 we did hold that appellants there had waived their right to preliminary investigation because immediately after their arrest, they filed bail and proceeded

to trial "without previously claiming that they did not have the benefit of a preliminary investigation." 24 In the instant case, petitioner Go asked for release on recognizance or on bail and for preliminary investigation in one omnibus motion. He had thus claimed his right to preliminary investigation before respondent Judge approved the cash bond posted by petitioner and ordered his release on 12 July 1991. Accordingly, we cannot reasonably imply waiver of preliminary investigation on the part of petitioner. In fact, when the Prosecutor filed a motion in court asking for leave to conduct preliminary investigation, he clearly if impliedly recognized that petitioner's claim to preliminary investigation was a legitimate one. We would clarify, however, that contrary to petitioner's contention the failure to accord preliminary investigation, while constituting a denial of the appropriate and full measure of the statutory process of criminal justice, did not impair the validity of the information for murder nor affect the jurisdiction of the trial court. 25 It must also be recalled that the Prosecutor had actually agreed that petitioner was entitled to bail. This was equivalent to an acknowledgment on the part of the Prosecutor that the evidence of guilt then in his hands was not strong. Accordingly, we consider that the 17 July 1991 order of respondent Judge recalling his own order granting bail and requiring petitioner to surrender himself within forty-eight (48) hours from notice, was plainly arbitrary considering that no evidence at all and certainly no new or additional evidence had been submitted to respondent Judge that could have justified the recall of his order issued just five (5) days before. It follows that petitioner was entitled to be released on bail as a matter of right. The final question which the Court must face is this: how does the fact that, in the instant case, trial on the merits has already commenced, the Prosecutor having already presented four (4) witnesses, impact upon, firstly, petitioner's right to a preliminary investigation and, secondly, petitioner's right to be released on bail? Does he continue to be entitled to have a preliminary investigation conducted in respect of the charge against him? Does petitioner remain entitled to be released on bail? Turning first to the matter of preliminary investigation, we consider that petitioner remains entitled to a preliminary investigation although trial on the merits has already began. Trial on the merits should be suspended or held in abeyance and a preliminary investigation forthwith accorded to petitioner. 26 It is true that the Prosecutor might, in view of the evidence that he may at this time have on hand, conclude that probable cause exists; upon the other hand, the Prosecutor conceivably could reach the conclusion that the evidence on hand does not warrant a finding of probable cause. In any event, the constitutional point is that petitioner was not accorded what he was entitled to by way of procedural due process. 27 Petitioner was forced to undergo arraignment and literally pushed to trial without preliminary investigation, with extraordinary haste, to the applause

from the audience that filled the courtroom. If he submitted to arraignment at trial, petitioner did so "kicking and screaming," in a manner of speaking . During the proceedings held before the trial court on 23 August 1991, the date set for arraignment of petitioner, and just before arraignment, counsel made very clear petitioner's vigorous protest and objection to the arraignment precisely because of the denial of preliminary investigation. 28 So energetic and determined were petitioner's counsel's protests and objections that an obviously angered court and prosecutor dared him to withdraw or walkout, promising to replace him with counsel de oficio. During the trial, before the prosecution called its first witness, petitioner through counsel once again reiterated his objection to going to trial without preliminary investigation: petitioner's counsel made of record his "continuing objection." 29 Petitioner had promptly gone to the appellate court on certiorari and prohibition to challenge the lawfulness of the procedure he was being forced to undergo and the lawfulness of his detention. 30 If he did not walk out on the trial, and if he cross-examined the prosecution's witnesses, it was because he was extremely loath to be represented by counsel de oficio selected by the trial judge, and to run the risk of being held to have waived also his right to use what is frequently the only test of truth in the judicial process. In respect of the matter of bail, we similarly believe and so hold that petitioner remains entitled to be released on bail as a matter of right. Should the evidence already of record concerning petitioner's guilt be, in the reasonable belief of the Prosecutor, strong, the Prosecutor may move in the trial court for cancellation of petitioner's bail. It would then be up to the trial court, after a careful and objective assessment of the evidence on record, to grant or deny the motion for cancellation of bail. To reach any other conclusions here, that is, to hold that petitioner's rights to a preliminary investigation and to bail were effectively obliterated by evidence subsequently admitted into the record would be to legitimize the deprivation of due process and to permit the Government to benefit from its own wrong or culpable omission and effectively to dilute important rights of accused persons well-nigh to the vanishing point. It may be that to require the State to accord petitioner his rights to a preliminary investigation and to bail at this point, could turn out ultimately to be largely a ceremonial exercise. But the Court is not compelled to speculate. And, in any case, it would not be idle ceremony; rather, it would be a celebration by the State of the rights and liberties of its own people and a re-affirmation of its obligation and determination to respect those rights and liberties. ACCORDINGLY, the Court resolved to GRANT the Petition for Review on Certiorari. The Order of the trial court dated 17 July 1991 is hereby SET ASIDE and NULLIFIED, and the Decision of the Court of Appeals dated 23 September 1991 hereby REVERSED. The Office of the Provincial Prosecutor is hereby ORDERED to conduct forthwith

a preliminary investigation of the charge of murder against petitioner Go, and to complete such preliminary investigation within a period of fifteen (15) days from commencement thereof. The trial on the merits of the criminal case in the Regional Trial Court shall be SUSPENDED to await the conclusion of the preliminary investigation. Meantime, petitioner is hereby ORDERED released forthwith upon posting of a cash bail bond of One Hundred Thousand Pesos (P100,000.00). This release shall be without prejudice to any lawful order that the trial court may issue, should the Office of the Provincial Prosecutor move for cancellation of bail at the conclusion of the preliminary investigation. No pronouncement as to costs. This Decision is immediately executory. SO ORDERED.

G.R. No. 89139 August 2, 1990 ROMEO POSADAS y ZAMORA, petitioner, vs. THE HONORABLE COURT OF APPEALS and THE PEOPLE OF THE PHILIPPINES, respondents. Rudy G. Agravate for petitioner.

GANCAYCO, J.: The validity of a warrantless search on the person of petitioner is put into issue in this case. On October 16, 1986 at about 10:00 o'clock in the morning Pat. Ursicio Ungab and Pat. Umbra Umpar, both members of the Integrated National Police (INP) of the Davao Metrodiscom assigned with the Intelligence Task Force, were conducting a surveillance along Magallanes Street, Davao City. While they were within the premises of the Rizal Memorial Colleges they spotted petitioner carrying a "buri" bag and they noticed him to be acting suspiciously. They approached the petitioner and identified themselves as members of the INP. Petitioner attempted to flee but his attempt to get away was thwarted by the two notwithstanding his resistance. They then checked the "buri" bag of the petitioner where they found one (1) caliber .38 Smith & Wesson revolver with Serial No. 770196 1 two (2) rounds of live ammunition for a .38 caliber gun 2 a smoke (tear gas) grenade, 3 and two (2) live ammunitions for a .22 caliber gun. 4 They brought the petitioner to the police station for further investigation. In the course of the same, the petitioner was asked to show the necessary license or authority to possess firearms and ammunitions found in his possession but he failed to do so. He was then taken to the Davao Metrodiscom office and the prohibited articles recovered from him were indorsed to M/Sgt. Didoy the officer then on duty. He was prosecuted for illegal possession of firearms and ammunitions in the Regional Trial Court of Davao City wherein after a plea of not guilty and trial on the merits a decision was rendered on October 8, 1987 finding petitioner guilty of the offense charged as follows: WHEREFORE, in view of all the foregoing, this Court , finds the accused guilty beyond reasonable doubt of the offense charged. It appearing that the accuse d was below eighteen (18) years old at the time of the commission of the offense (Art. 68, par. 2), he is hereby sentenced to an indeterminate penalty ranging from TEN (10) YEARS and ONE (1) DAY of prision mayor to TWELVE (12) Years, FIVE (5) months and Eleven (11) days of

Reclusion Temporal, and to pay the costs. The firearm, ammunitions and smoke grenade are forfeited in favor of the government and the Branch Clerk of Court is hereby directed to turn over said items to the Chief, Davao Metrodiscom, Davao City. 5 Not satisfied therewith the petitioner interposed an appeal to the Court of Appeals wherein in due course a decision was rendered on February 23, 1989 affirming in toto the appealed decision with costs against the petitioner. 6 Hence, the herein petition for review, the main thrust of which is that there being no lawful arrest or search and seizure, the items which were confiscated from the possession of the petitioner are inadmissible in evidence against him. The Solicitor General, in justifying the warrantless search of the buri bag then carried by the petitioner, argues that under Section 12, Rule 136 of the Rules of Court a person lawfully arrested may be searched for dangerous weapons or anything used as proof of a commission of an offense without a search warrant. It is further alleged that the arrest without a warrant of the petitioner was lawful under the circumstances. Section 5, Rule 113 of the 1985 Rules on Criminal Procedure provides as follows: SEC. 5. Arrest without warrant; when lawful A peace officer or a private person may, without a warrant, arrest a person: (a) When in his presence, the person to be arrested has committed is actually committing, or is attempting to commit an offense; (b) When an offense has in fact just been committed, and he has personal knowledge of facts indicating that the person to be arrested has committed it; and (c) When the person to be arrested is a prisoner who has escaped from a penal establishment or place where he is serving final judgment or temporarily confined while his case is pending, or has escaped while being transferred from one confinement to another. In cases falling under paragraphs (a) and (b) hereof, the person arrested without a warrant shall be forthwith delivered to the nearest police station or jail, and he shall be proceeded against in accordance with Rule 112, Section 7. (6a, 17a) From the foregoing provision of law it is clear that an arrest without a warrant may be effected by a peace officer or private person, among others, when in his presence the person to be arrested has committed, is actually committing, or is attempting to commit an offense; or when an offense has in fact just been

committed, and he has personal knowledge of the facts indicating that the person arrested has committed it. The Solicitor General argues that when the two policemen approached the petitioner, he was actually committing or had just committed the offense of illegal possession of firearms and ammunitions in the presence of the police officers and consequently the search and seizure of the contraband was incidental to the lawful arrest in accordance with Section 12, Rule 126 of the 1985 Rules on Criminal Procedure. We disagree. At the time the peace officers in this case identified themselves and apprehended the petitioner as he attempted to flee they did not know that he had committed, or was actually committing the offense of illegal possession of firearms and ammunitions. They just suspected that he was hiding something in the buri bag. They did now know what its contents were. The said circumstances did not justify an arrest without a warrant. However, there are many instances where a warrant and seizure can be effected without necessarily being preceded by an arrest, foremost of which is the "stop and search" without a search warrant at military or police checkpoints, the constitutionality or validity of which has been upheld by this Court in Valmonte vs. de Villa, 7 as follows: Petitioner Valmonte's general allegation to the effect that he had been stopped and searched without a search warrant by the military manning the checkpoints, without more, i.e., without stating the details of the incidents which amount to a violation of his light against unlawful search and seizure, is not sufficient to enable the Court to determine whether there was a violation of Valmonte's right against unlawful search and seizure. Not all searches and seizures are prohibited. Those which are reasonable are not forbidden. A reasonable search is not to be determined by any fixed formula but is to be resolved according to the facts of each case. Where, for example, the officer merely draws aside the curtain of a vacant vehicle which is parked on the public fair grounds, or simply looks into a vehicle or flashes a light therein, these do not constitute unreasonable search. The setting up of the questioned checkpoints in Valenzuela (and probably in other areas) may be considered as a security measure to enable the NCRDC to pursue its mission of establishing effective territorial defense and maintaining peace and order for the benefit of the public. Checkpoints may also be regarded as measures to thwart plots to destabilize the government in the interest of public security. In this connection, the Court may take judicial notice of the shift to urban centers and their suburbs of the insurgency movement, so clearly reflected in the increased killings in cities of police and military men by NPA "sparrow units," not to mention the abundance of unlicensed firearms and the alarming rise in

lawlessness and violence in such urban centers, not all of which are reported in media, most likely brought about by deteriorating economic conditions which all sum up to what one can rightly consider, at the very least, as abnormal times. Between the inherent right of the state to protect its existence and promote public welfare and an individual's right against a warrantless search which is however reasonably conducted, the former should prevail. True, the manning of checkpoints by the military is susceptible of abuse by the men in uniform in the same manner that all governmental power is susceptible of abuse. But, at the cost of occasional inconvenience, discomfort and even irritation to the citizen, the checkpoints during these abnormal times, when conducted within reasonable limits, are part of the price we pay for an orderly society and a peaceful community. (Emphasis supplied). Thus, as between a warrantless search and seizure conducted at military or police checkpoints and the search thereat in the case at bar, there is no question that, indeed, the latter is more reasonable considering that unlike in the former, it was effected on the basis of a probable cause. The probable cause is that when the petitioner acted suspiciously and attempted to flee with the buri bag there was a probable cause that he was concealing something illegal in the bag and it was the right and duty of the police officers to inspect the same. It is too much indeed to require the police officers to search the bag in the possession of the petitioner only after they shall have obtained a search warrant for the purpose. Such an exercise may prove to be useless, futile and much too late. In People vs. CFI of Rizal, 8 this Court held as follows: . . . In the ordinary cases where warrant is indispensably necessary, the mechanics prescribed by the Constitution and reiterated in the Rules of Court must be followed and satisfied. But We need not argue that there are exceptions. Thus in the extraordinary events where warrant is not necessary to effect a valid search or seizure, or when the latter cannot be performed except without warrant, what constitutes a reasonable or unreasonable search or seizure becomes purely a judicial question, determinable from the uniqueness of the circumstances involved, including the purpose of the search or seizure, the presence or absence of probable cause, the manner in which the search and seizure was made, the place or thing searched and the character of the articles procured. The Court reproduces with approval the following disquisition of the Solicitor General: The assailed search and seizure may still be justified as akin to a "stop and frisk" situation whose object is either to determine the identity of a suspicious individual

or to maintain the status quo momentarily while the police officer seeks to obtain more information. This is illustrated in the case of Terry vs. Ohio, 392 U.S. 1 (1968). In this case, two men repeatedly walked past a store window and returned to a spot where they apparently conferred with a third man. This aroused the suspicion of a police officer. To the experienced officer, the behaviour of the men indicated that they were sizing up the store for an armed robbery. When the police officer approached the men and asked them for their names, they mumbled a reply. Whereupon, the officer grabbed one of them, spun him around and frisked him. Finding a concealed weapon in one, he did the same to the other two and found another weapon. In the prosecution for the offense of carrying a concealed weapon, the defense of illegal search and seizure was put up. The United States Supreme Court held that "a police officer may in appropriate circumstances and in an appropriate manner approach a person for the purpose of investigating possible criminal behaviour even though there is no probable cause to make an arrest." In such a situation, it is reasonable for an officer rather than simply to shrug his shoulder and allow a crime to occur, to stop a suspicious individual briefly in order to determine his identity or maintain the status quo while obtaining more information. . . . Clearly, the search in the case at bar can be sustained under the exceptions heretofore discussed, and hence, the constitutional guarantee against unreasonable searches and seizures has not been violated. 9 WHEREFORE, the petition is DENIED with costs against petitioner. SO ORDERED.

[G.R. No. 130491. March 25, 1999] PEOPLE OF THE PHILIPPINES, Plaintiff-Appellee, vs. ROBERTO MENGOTE, Accused-Appellant. DECISION PER CURIAM: For automatic review by this court is the decision dated May 16, 1997 of the Regional Trial Court of Malolos, Bulacan, Branch 17, convicting Roberto Mengote of rape, as follows: WHEREFORE, premises considered, the court finds accused Roberto Mengote guilty beyond reasonable doubt of the crime of Rape, as defined and penalized under Article 335 of the Revised Penal Code, as amended by R.A. 7659, and hereby sentences him to suffer the penalty of Death and to pay the offended party Jenny Mengote the following: 1. P100,000.00 as moral damages and 2. P100,000.00 as exemplary damages. Roberto Mengote information: was charged with rape under the following

That on or about the 20th day of March, 1996, in the municipality of Hagonoy, province of Bulacan, Philippines, and within the jurisdiction of this Honorable Court, the above-named accused, being then the father of the offended party, did then and there wilfully, unlawfully and feloniously by means of force and intimidation and with lewd designs, have carnal knowledge of the offended party, Jenny Mengote, a twelve (12) year old girl against her will and without her consent. Contrary to law. The accused with the assistance of counsel de officio pleaded NOT GUILTY upon arraignment.[1] The prosecution presented the victim Jenny Mengote, her mother Dolores Mengote who assisted her when she filed a complaint for rape and Dr. Edgardo Gueco, who prepared the medico-legal report dated April 22, 1996 marked as Exh. D. Jenny Mengote is the daughter of the accused Roberto Mengote and

Dolores Mengote. She turned twelve on January 25, 1996, and was a Grade 6 student at the Sagrada Familia Elementary School in Hagonoy, Bulacan. She testified on June 27, 1996 on the incident that happened on March 20, 1996 between the hours of 7:00 oclock and 8:00 oclock in the evening. She was watching TV in her Tita Meloidas house, which is about 2 or 3 meters from her own home, when her father called her from the door of their house and ordered her to get his lighter upstairs. When she got the lighter, her father followed and embraced her from behind. She was surprised because her father was not in the habit of embracing her except when she was a small child. She evaded him by going to the place where they kept their clothes. Her father followed her again, embraced her and kissed her on the face, at the same time telling her to keep quiet and not to report to anybody (huwag akong magsusumbong). She left the place and went down to where their water jar was located in order to evade her father. Her father followed, pinched her ear and pulled her upstairs while still holding her ear. He again embraced and kissed her, touched her private part (referring to her vagina) while she was in a standing position, and proceeded to remove her T-shirt, her shorts and her panty. She protested huwag, huwag but her father told her sandali lang iyon. Thereafter, he laid her down on the floor, removed his pants and briefs and put his body on top of her, face down. She was then lying straight, and her father separated her right thigh from the left and inserted his penis into her vagina, which caused her pain. The penis penetrated about an inch into the vagina, and he was moving up and down, push and pull[2] about five times. She felt something come out from his penis, which was colored white. Her father noticed that her mother was coming, and he stood up and wiped his penis with a white rag and put on his shorts and briefs. When her mother entered, she asked her father what he had done to her. He said nothing and her mother got mad. Her mother saw the piece of rag that her father used in wiping his penis, smelled it, and asked him what it means; her father did not answer and left the house. She was also asked by her mother about what her father did to her. She did not answer because of her father' threat to kill them ("a papatayin kayo". She told her mother about the incident two weeks after, when their mother left the house to go to the house of a relative in another barangay.[3] The reason she did not reveal to her mother earlier what happened was that she was thinking of her brothers and sisters and her father might also do to her sister what she did to her. She was no longer afraid of her father after she reported the same to

her mother. Her mother promptly brought her to the police station to file a complaint.[4] In court, Jenny attested to the truth of the contents of her sworn statement. Asked about her answer to Question No. 8 wherein she stated that her father committed the act against her three times, she explained that on the two previous occasions that took place in January and February 1996, her father embraced and kissed her, but did not attempt to insert his penis into her private part. She did not tell anybody because she was afraid of her father who told her not to report these to anybody, and who in the past used to beat them when his orders were not followed.[5] Upon cross-examination, witness Jenny answered the questions propounded substantially as she testified during the direct examination.[6] A medico-legal examination was conducted by Dr. Edgardo Gueco, upon the request of the Chief of Police of Hagonoy, Bulacan. The extra-genital examination of the hymen revealed the presence of deep healed lacerations at 4 and 12 oclock and shallow healed laceration at 7 oclock positions.[7] The report carried the remarks subject is in nonvirgin state physically.[8] Dr. Gueco testified that the presence of the laceration of the hymen means that subject was no longer a virgin at the time of the examination and possible cause of said laceration is sexual intercourse, and that because the healing period is usually ten days, the laceration inflicted by the sexual intercourse on March 22nd could have already been healed.[9] Dolores Mengote identified her signature on the sworn complaint of her daughter,[10] and stated that it took a long time for her to give her consent to Jennys filing the complaint against her father because she did not want the people around them to know because she was ashamed,[11] and that when they filed the complaint she did not know that the penalty for rape is death.[12] The accused Roberto Mengote testified in his defense. He declared that he was admitting the crime and repents for what he did, but prays that a lower sentence be imposed upon him. He testified that he was only drunk; in the months of January and February when he first attempted to rape Jenny, he could not recall where his other daughters were because he was drunk.[13] As aforestated, the court a quo found the accused guilty beyond reasonable doubt of the crime of rape.

The accused-appellant raises the following assignment of errors in his brief: I THE COURT A QUO ERRED IN TOTALLY DISREGARDING THE DEFENSE PUT UP BY THE ACCUSED-APPELLANT. II THE TRIAL COURT GRAVELY ERRED IN NOT APPLYING THE SAFEGUARDS SET FORTH UNDER RULE 116, 1987 RULES ON CRIMINAL PROCEDURE. III THE COURT A QUO GRAVELY ERRED IN FINDING THAT THE GUILT OF THE ACCUSED-APPELLANT FOR THE CRIME CHARGED HAS BEEN PROVEN BEYOND REASONABLE DOUBT. Accused-appellant submits that the degree of his intoxication at the time of the incident affected his mental faculty to a degree that he was no longer conscious and in control of what he was doing; that his mental faculties were so far overcome by intoxication that it produces a mental condition of insanity. Appellant further claims that the trial court erred in not applying the safeguards imposed by Section 3, Rule 116 of the 1987 Rules on Criminal Procedure after a plea of guilty to a capital offense, in that it did not endeavor to conduct a searching inquiry as to the voluntariness and full comprehension of the consequences of his plea of guilt; thus, the accused admitted his guilt in his testimony in court in the belief that the penalty to be meted by the court is life imprisonment.[14] Finally, appellant contends that his guilt was not proven beyond reasonable doubt as there were glaring inconsistencies between the testimony of Jenny during the direct examination and that during the cross-examination, as well as between her testimony and the testimony of her mother. We find no merit in the appeal. We have scrutinized the evidence and reviewed the testimony of the complainant with great caution and are convinced that the trial court correctly held that the guilt of Roberto Mengote was established beyond reasonable doubt. The narration of Jenny is positive, categorical and full of details, free of any significant inconsistencies and clearly described the sexual assault wherein, as the trial court observed, her juvenile resistance proved no match to the strength and

evil determination of her father. We are not unmindful of the crucial importance in a rape case of determining the credibility of both the victim herself and her version as to how the crime charged was committed but we repose almost total reliance on the findings and conclusions of the trial court which had the clear advantage of a trial judge over an appellate court magistrate in the appreciation of testimonial evidence.[15] In the absence of any showing that the trial courts assessment of the credibility of the witness was flawed, we are bound by its assessment.[16] Furthermore, it is doctrinally settled that testimonies of rape victims who are of tender age are credible. The revelation of an innocent child whose chastity was abused deserves full credit, as the willingness of the complainant to face police investigation and to undergo the trouble and humiliation of a public trial is eloquent testimony of the truth of her complaint.[17] The trial court observed: It is an undeniable fact that because of their filial relationship, the accused exercised a great degree of moral ascendancy over his 12 year old daughter Jenny, so much so that even if she found her fathers embraces and kisses to be quite peculiar because it was not his wont to kiss and embrace his children even as a show of love (tsn, June 27, 1996, p. 12), she could not adequately repel his advances. Furthermore, her fathers practice of beating them up when he was mad (tsn, July 11, 1996, pp. 13-14) which was foremost in her young mind then disabled her to struggle against him. In this respect, the employment of force and intimidation by the accused has been fully established.chanroblesvirtualawlibrary The testimony of Dr. Edgardo Gueco who conducted the medical examination corroborated Jennys claim that she was sexually abused. The presence of deep healed lacerations in the hymen indicated that she was no longer a virgin and the possible cause of said lacerations is sexual intercourse.[18] Neither are we persuaded by appellants plea of insanity allegedly caused by intoxication that has affected his mental faculty to a degree that he was no longer conscious of what he was doing. The accused pleaded insanity quite late and obviously as an afterthought. More important, it was not substantiated. The law presumes every man to be sane and if the accused interposes the defense of mental incapacity, the burden of establishing such fact rests upon him.[19] Insanity must be proven by clear and positive evidence.[20] As an

exempting circumstance, insanity means that the accused must have been deprived completely of reason and freedom of the will at the time of the commission of the crime[21] or be incapable of entertaining criminal intent.[22] In this case, the appellant merely stated that he was not in my right senses at the time, because he was drunk. Thus: chanroblesvirtualawlibrary COURT: xxx xxx xxx Q: Can you tell us what compelled you to rape your daughter? A: I was not in my right senses at the time, your Honor Q: Why, were you under the influence of any liquor or drugs? A: I was drunk at that time your Honor. xxx xxx xxx COURT: Q: How many daughters do you have? A: Five (5), your Honor Q: Why did you specifically pick on Jenny to be your vctim? A: I did not intend to do it, your Honor. Q: You will recall that at that time Jenny was out of your house and you just called her? A: I dont remember that, your Honor Q: So, at that ime you were not also aware where your other daughters were? A: No, your honor.[23] (underscoring supplied) The above testimony of the accused clearly falls short of the degree of proof necessary to prove insanity. The appellants second assignment of error is not tenable. The records

show that appellant pleaded not guilty upon arraignment.[24] Before the prosecution started the presentation of its second witness, the defense counsel manifested that the accused would be changing his plea to that of guilty. The court ruled that considering that the offense charged is a grave offense, the presentation of the prosecutions evidence is still required to determine the guilt of the accused.[25] After the prosecution had rested its case, counsel for the accused manifested hat the accused intended his willingness to plead guilty. The records show: PROS. JOAQUIN: For the prosecution, your Honor. ATTY. DELA CRUZ: For the accused, ready, your Honor. Your Honor please, considering that the accused in this case intended his willingness to plead guilty to this case, your Honor COURT: Up to now have you not still understood the position of the court, in the first place a plea of guilty should not be unconditional. ATTY. DELA CRUZ: That is the wish of the accused, your Honor, and I could not be more than the accused. COURT: Is it the accused dictating to the court what penalty should be imposed upon him and that if the Court will not agree to his condition he will not plead guilty, is that it? ATTY. DELA CRUZ: I think he court has already made an Order that he could not be bound by the offer of the accused, your Honor. That is why the case went through and the prosecution rested its case, your Honor. So, now we are presenting the accused, your Honor.chanroblesvirtualawlibrary COURT:chanroblesvirtualawlibrary (to interpreter) Call the witness and swear in the witness.

INTERPRETER: (to witness) Do you swear to tell the truth, the whole truth and nothing but the truth before the Honorable Court? WITNESS: I do, mam. INTERPRETER: Please state your name and circumstances.chanroblesvirtualawlibrary WITNESS:chanroblesvirtualawlibrary I am ROBERTO MENGOTE, 40 years old, married, fisherman and residing at Sagrada Familia, Hagonoy, Bulacan.chanroblesvirtualawlibrary ATTY. DELA CRUZchanroblesvirtualawlibrary We are offering the testimony of the witness to initiate from this witness his decision as to whether he will admit in open Court guilty or not guilty to this case, anyway, he was already arraigned but up to now this representation is under doubt as to whether he is willing to admit or not, your Honor. With the kind permission of the Hon. Court?chanroblesvirtualawlibrary COURT:chanroblesvirtualawlibrary Kindly proceed.chanroblesvirtualawlibrary D. EXAM. OF WIT. R. MENGOTE BYchanroblesvirtualawlibrary ATTY. DELA CRUZ:chanroblesvirtualawlibrary Q: What is your relationship case?chanroblesvirtualawlibrary with the complainants in this other personal

A: My wife and my daughter, sir.chanroblesvirtualawlibrary Q: During the presentation of the prosecutions evidence they presented you as the one who molested your daughter, what can you say about that?chanroblesvirtualawlibrary A: That is not true, sir.chanroblesvirtualawlibrary

Q: What is the truth Mr. Witness?chanroblesvirtualawlibrary A: I am admitting the crime, sir, and repent to what I did and I am praying to the Hon. Court that a lower sentence be imposed upon me, sir.chanroblesvirtualawlibrary Q: Do you have other Court?chanroblesvirtualawlibrary things to say before this

A: No more, sir.chanroblesvirtualawlibrary ATTY. DELA CRUZ:chanroblesvirtualawlibrary I think that is all, your Honor.chanroblesvirtualawlibrary COURT:chanroblesvirtualawlibrary Q: Is it not true that the reason why you are repenting is because you are afraid to be imposed the death penalty?chanroblesvirtualawlibrary A: No your Honor.chanroblesvirtualawlibrary Q: In other words, even if you have heard your daughter say that she is willing to have the death penalty imposed on you, you are not still afraid?chanroblesvirtualawlibrary A: I have nothing to do if that is the penalty and I wish that is not the penalty, your Honor.[26] chanroblesvirtualawlibrary It is clear that appellant initially denied that he molested his daughter although he later stated that he was admitting the crime and was repenting what he did praying that a lower sentence be imposed and that the death penalty is not the penalty to be imposed. The trial court did not act favorably on his offer because the plea of guilty should be unconditional, i.e., not conditioned on his getting a lower penalty. It has been held that a plea of guilty made after arraignment and after trial had began does not entitle the accused to have such plea considered as a mitigating circumstance.[27] Moreover, in cases where the law prescribes a single indivisible penalty, it shall be applied regardless of any mitigating or aggravating circumstances that may have attended the commission of the deed.[28] chanroblesvirtualawlibrary At any rate, the court is convinced that the evidence of the prosecution has undoubtedly established the guilt of the accused beyond reasonable doubt and the accused was given full opportunity to

present his own evidence. The court did not rest its finding of guilt on the statement of the accused admitting the crime. The alleged inconsistencies in the testimony of the victim do not cast doubt on her credibility. They refer to minor and insubstanial details, e.g., whether the accused embraced Jenny first before kissing her[29] or kissed her first and then embraced her,[30] or whether or not Jenny tried to evade her father by going to the kitchen near the water jar.[31] They do not detract from the substance of her testimony that her father succeeded in performing the carnal act against her will. This Court has ruled in numerous cases that an errorless recollection of a harrowing incident cannot be expected of a witness especially when she is recounting details of an experience so humiliating and so painful as rape.[32] Minor errors in the testimony of a rape victim tend to buttress rather than weaken her credibility since that would indicate that her testimony was not contrived.[33] The crime of rape is defined and made punishable by Article 335 of the Revised Penal Code, as amended by Section 11 of R.A. No. 7659[34] which reads: chanroblesvirtualawlibrary SEC. 11 Article 335 of the same Code (Revised Penal Code, as amended) is hereby amended to read as follows: ART. 335. When and how rape is committed.- Rape is committed by having carnal knowledge of a woman under any of the following circumstances:chanroblesvirtualawlibrary 1. By using force or intimidation; 2. When the woman is deprived of reason or otherwise unconscious; andchanroblesvirtualawlibrary 3. When the woman is under twelve demented.chanroblesvirtualawlibrary The crime of rape shall perpetua.chanroblesvirtualawlibrary be years of by age or is

punished

reclusion

Whenever the crime of rape is committed with the use of a deadly weapon or by two or more persons, the penalty shall be reclusion perpetua to death.chanroblesvirtualawlibrary When by reason or on the occasion of the rape, the victim has become insane, the penalty shall be death.chanroblesvirtualawlibrary

When the rape is attempted or frustrated and a homicide is committed by reason or on the occasion thereof, the penalty shall be reclusion perpetua to death.chanroblesvirtualawlibrary When by reason or on the occasion of the rape, a homicide is committed, the penalty shall be death.chanroblesvirtualawlibrary The death penalty shall also be imposed if the crime of rape is committed with any of the following attendant circumstances: 1. When the victim is under eighteen (18) years of age and the offender is a parent, ascendant, step-parent, guardian, relative by consanguinity or affinity within the third civil degree, or the commonlaw spouse of the parent of the victim. 2. When the victim is under the custody of the police or military authorities. 3.When the rape is committed in full view of the husband, parent, any of the children or other relatives within the third degree of consanguinity. 4. When the victim is a religious or child below seven (7) years old. 5. When the offender knows that he is afflicted with Acquired Immune Deficiency Syndrome (AIDS) disease. 6. When committed by any member of the Armed Forces of the Philippines or the Philippine National Police or any law enforcement agency. 7. When by reason or on the occasion of the rape, the victim has suffered permanent physical mutilation. (underscoring supplied).chanroblesvirtualawlibrary Accordingly, the Court is constrained to affirm the death penalty imposed by the trial court. Four justices of the Court, however, have continued to maintain the unconstitutionality of Republic Act No. 7659 insofar as it prescribes the death penalty; nevertheless, they submit to the ruling of the majority to the effect that the law is constitutional and that the death penalty can be lawfully imposed in the case at bar. chanroblesvirtualawlibrary Relative to the monetary liability of accused-appellant, the Court, in line with prevailing jurisprudence, finds an award of P75,000.00[35] by way of indemnity and an amount of P50,000.00 as moral damages

to be in order. The award of P100,000.00 for exemplary damages is hereby deleted for lack of legal basis. chanroblesvirtualawlibrary WHEREFORE, the appealed decision of the trial court finding accusedappellant Roberto Mengote guilty beyond reasonable doubt of the crime charged and imposing upon him the penalty of death is AFFIRMED, with the modification that accused-appellant is ordered to pay P75,000.00 as civil indemnity, and P50,000.00 as moral damages. The award for exemplary damages is hereby deleted. chanroblesvirtualawlibrary In accordance with Section 25 of Republic Act No. 7659, amending Article 83 of the Revised Penal Code, upon the finality of he decision, let the records of this case be forthwith forwarded to the Office of the President for possible exercise of the pardoning power. No pronouncement on costs. chanroblesvirtualawlibrary SO ORDERED.

G.R. No. 123595 December 12, 1997 SAMMY MALACAT y MANDAR, petitioner, vs. COURT OF APPEALS, and PEOPLE OF THE PHILIPPINES, respondents.

DAVIDE, JR., J.: In an Information 1 filed on 30 August 1990, in Criminal Case No. 90-86748 before the Regional Trial Court (RTC) of Manila, Branch 5, petitioner Sammy Malacat y Mandar was charged with violating Section 3 of Presidential Decree No. 1866, 2 as follows: That on or about August 27, 1990, in the City of Manila, Philippines, the said accused did then and there willfully, unlawfully and knowingly keep, possess and/or acquire a hand grenade, without first securing the necessary license and/or permit therefor from the proper authorities. At arraignment 3 on 9 October 1990, petitioner, assisted by counsel de oficio, entered a plea of not guilty. At pre-trial on 11 March 1991, petitioner admitted the existence of Exhibits "A," "A-1," and "A-2," 4 while the prosecution admitted that the police authorities were not armed with a search warrant nor warrant of arrest at the time they arrested petitioner. 5 At trial on the merits, the prosecution presented the following police officers as its witnesses: Rodolfo Yu, the arresting officer; Josefino G. Serapio, the investigating officer; and Orlando Ramilo, who examined the grenade. Rodolfo Yu of the Western Police District, Metropolitan Police Force of the Integrated National Police, Police Station No. 3, Quiapo, Manila, testified that on 27 August 1990, at about 6:30 p.m., in response to bomb threats reported seven days earlier, he was on foot patrol with three other police officers (all of them in uniform) along Quezon Boulevard, Quiapo, Manila, near the Mercury Drug store at Plaza Miranda. They chanced upon two groups of Muslim-looking men, with each group, comprised of three to four men, posted at opposite sides of the corner of Quezon Boulevard near the Mercury Drug Store. These men were acting suspiciously with "[t]heir eyes. . . moving very fast." 6 Yu and his companions positioned themselves at strategic points and observed both groups for about thirty minutes. The police officers then approached one group of men, who then fled in different directions. As the policemen gave chase, Yu caught up with and apprehended petitioner. Upon searching petitioner, Yu found a fragmentation grenade tucked inside petitioner's "front waist line." 7 Yu's

companion, police officer Rogelio Malibiran, apprehended Abdul Casan from whom a .38 caliber revolver was recovered. Petitioner and Casan were then brought to Police Station No. 3 where Yu placed an "X" mark at the bottom of the grenade and thereafter gave it to his commander. 8 On cross-examination, Yu declared that they conducted the foot patrol due to a report that a group of Muslims was going to explode a grenade somewhere in the vicinity of Plaza Miranda. Yu recognized petitioner as the previous Saturday, 25 August 1990, likewise at Plaza Miranda, Yu saw petitioner and 2 others attempt to detonate a grenade. The attempt was aborted when Yu and other policemen chased petitioner and his companions; however, the former were unable to catch any of the latter. Yu further admitted that petitioner and Casan were merely standing on the corner of Quezon Boulevard when Yu saw them on 27 August 1990. Although they were not creating a commotion, since they were supposedly acting suspiciously, Yu and his companions approached them. Yu did not issue any receipt for the grenade he allegedly recovered from petitioner. 9 Josefino G. Serapio declared that at about 9:00 a.m. of 28 August 1990, petitioner and a certain Abdul Casan were brought in by Sgt. Saquilla 10 for investigation. Forthwith, Serapio conducted the inquest of the two suspects, informing them of their rights to remain silent and to be assisted by competent and independent counsel. Despite Serapio's advice, petitioner and Casan manifested their willingness to answer questions even without the assistance of a lawyer. Serapio then took petitioner's uncounselled confession (Exh. "E"), there being no PAO lawyer available, wherein petitioner admitted possession of the grenade. Thereafter, Serapio prepared the affidavit of arrest and booking sheet of petitioner and Casan. Later, Serapio turned over the grenade to the Intelligence and Special Action Division (ISAD) of the Explosive Ordinance Disposal Unit for examination. 11 On cross-examination, Serapio admitted that he took petitioner's confession knowing it was inadmissible in evidence. 12 Orlando Ramilo, a member of the Bomb Disposal Unit, whose principal duties included, among other things, the examination of explosive devices, testified that on 22 March 1991, he received a request dated 19 March 1991 from Lt. Eduardo Cabrera and PO Diosdado Diotoy for examination of a grenade. Ramilo then affixed an orange tag on the subject grenade detailing his name, the date and time he received the specimen. During the preliminary examination of the grenade, he "[f]ound that [the] major components consisting of [a] high filler and fuse assembly [were] all present," and concluded that the grenade was "[l]ive and capable of exploding." On even date, he issued a certification stating his findings, a copy of which he forwarded to Diotoy on 11 August 1991. 13 Petitioner was the lone defense witness. He declared that he arrived in Manila on 22 July 1990 and resided at the Muslim Center in Quiapo, Manila. At around 6:30

in the evening of 27 August 1990, he went to Plaza Miranda to catch a breath of fresh air. Shortly after, several policemen arrived and ordered all males to stand aside. The policemen searched petitioner and two other men, but found nothing in their possession. However, he was arrested with two others, brought to and detained at Precinct No. 3, where he was accused of having shot a police officer. The officer showed the gunshot wounds he allegedly sustained and shouted at petitioner "[i]to ang tama mo sa akin." This officer then inserted the muzzle of his gun into petitioner's mouth and said, "[y]ou are the one who shot me." Petitioner denied the charges and explained that he only recently arrived in Manila. However, several other police officers mauled him, hitting him with benches and guns. Petitioner was once again searched, but nothing was found on him. He saw the grenade only in court when it was presented. 14 The trial court ruled that the warrantless search and seizure of petitioner was akin to it a "stop and frisk," where a "warrant and seizure can be effected without necessarily being preceded by an arrest" and "whose object is either to maintain the status quo momentarily while the police officer seeks to obtain more information." 15 Probable cause was not required as it was not certain that a crime had been committed, however, the situation called for an investigation, hence to require probable cause would have been "premature." 16 The RTC emphasized that Yu and his companions were "[c]onfronted with an emergency, in which the delay necessary to obtain a warrant, threatens the destruction of evidence" 17 and the officers "[h]ad to act in haste," as petitioner and his companions were acting suspiciously, considering the time, place and "reported cases of bombing." Further, petitioner's group suddenly ran away in different directions as they saw the arresting officers approach, thus "[i]t is reasonable for an officer to conduct a limited search, the purpose of which is not necessarily to discover evidence of a crime, but to allow the officer to pursue his investigation without fear of violence." 18 The trial court then ruled that the seizure of the grenade from petitioner was incidental to a lawful arrest, and since petitioner "[l]ater voluntarily admitted such fact to the police investigator for the purpose of bombing the Mercury Drug Store," concluded that sufficient evidence existed to establish petitioner's guilt beyond reasonable doubt. In its decision 19 dated 10 February 1994 but promulgated on 15 February 1994, the trial court thus found petitioner guilty of the crime of illegal possession of explosives under Section 3 of P.D. No. 186, and sentenced him to suffer: [T]he penalty of not less than SEVENTEEN (17) YEARS, FOUR (4) MONTHS AND ONE (1) DAY OF RECLUSION TEMPORAL, as minimum, and not more than THIRTY (30) YEARS OF RECLUSION PERPETUA, as maximum. On 18 February 1994, petitioner filed a notice of appeal 20 indicating that he was appealing to this Court. However, the record of the case was forwarded to the

Court of Appeals which docketed it as CA-G.R. CR No. 15988 and issued a notice to file briefs. 21 In his Appellant's Brief 22 filed with the Court of Appeals, petitioner asserted that: 1. THE LOWER COURT ERRED IN HOLDING THAT THE SEARCH UPON THE PERSON OF ACCUSED-APPELLANT AND THE SEIZURE OF THE ALLEGED HANDGRENADE FROM HIM "WAS AN APPROPRIATE INCIDENT TO HIS ARREST." 2. THE LOWER COURT ERRED IN ADMITTING AS EVIDENCE AGAINST ACCUSED-APPELLANT THE HANDGRENADE ALLEGEDLY SEIZED FROM HIM AS IT WAS A PRODUCT OF AN UNREASONABLE AND ILLEGAL SEARCH. In sum, petitioner argued that the warrantless arrest was invalid due to absence of any of the conditions provided for in Section 5 of Rule 113 of the Rules of Court, citing People vs. Mengote. 23 As such, the search was illegal, and the hand grenade seized, inadmissible in evidence. In its Brief for the Appellee, the Office of the Solicitor General agreed with the trial court and prayed that its decision be affirmed in toto. 24 In its decision of 24 January 1996, 25 the Court of Appeals affirmed the trial court, noting, first, that petitioner abandoned his original theory before the court a quo that the grenade was "planted" by the police officers; and second, the factual finding of the trial court that the grenade was seized from petitioner's possession was not raised as an issue. Further, respondent court focused on the admissibility in evidence of Exhibit "D," the hand grenade seized from petitioner. Meeting the issue squarely, the Court of Appeals ruled that the arrest was lawful on the ground that there was probable cause for the arrest as petitioner was "attempting to commit an offense," thus: We are at a loss to understand how a man, who was in possession of a live grenade and in the company of other suspicious character[s] with unlicensed firearm[s] lurking in Plaza Miranda at a time when political tension ha[d] been enkindling a series of terroristic activities, [can] claim that he was not attempting to commit an offense. We need not mention that Plaza Miranda is historically notorious for being a favorite bomb site especially during times of political upheaval. As the mere possession of an unlicensed grenade is by itself an offense, Malacat's posture is simply too preposterous to inspire belief. In so doing, the Court of Appeals took into account petitioner's failure to rebut the testimony of the prosecution witnesses that they received intelligence reports of a bomb threat at Plaza Miranda; the fact that PO Yu chased petitioner two days prior to the latter's arrest, or on 27 August 1990; and that petitioner and his

companions acted suspiciously, the "accumulation" of which was more than sufficient to convince a reasonable man that an offense was about to be committed. Moreover, the Court of Appeals observed: The police officers in such a volatile situation would be guilty of gross negligence and dereliction of duty, not to mention of gross incompetence, if they [would] first wait for Malacat to hurl the grenade, and kill several innocent persons while maiming numerous others, before arriving at what would then be an assured but moot conclusion that there was indeed probable cause for an arrest. We are in agreement with the lower court in saying that the probable cause in such a situation should not be the kind of proof necessary to convict, but rather the practical considerations of everyday life on which a reasonable and prudent mind, and not legal technicians, will ordinarily act. Finally, the Court of Appeals held that the rule laid down in People v. Mengote, 26 which petitioner relied upon, was inapplicable in light of "[c]rucial differences," to wit: [In Mengote] the police officers never received any intelligence report that someone [at] the corner of a busy street [would] be in possession of a prohibited article. Here the police officers were responding to a [sic] public clamor to put a check on the series of terroristic bombings in the Metropolis, and, after receiving intelligence reports about a bomb threat aimed at the vicinity of the historically notorious Plaza Miranda, they conducted foot patrols for about seven days to observe suspicious movements in the area. Furthermore, in Mengote, the police officers [had] no personal knowledge that the person arrested has committed, is actually committing, or is attempting to commit an offense. Here, PO3 Yu [had] personal knowledge of the fact that he chased Malacat in Plaza Miranda two days before he finally succeeded in apprehending him. Unable to accept his conviction, petitioner forthwith filed the instant petition and assigns the following errors: 1. THE RESPONDENT COURT ERRED IN AFFIRMING THE FINDING OF THE TRIAL COURT THAT THE WARRANTIES ARREST OF PETITIONER WAS VALID AND LEGAL. 2. THE RESPONDENT COURT ERRED IN HOLDING THAT THE RULING IN PEOPLE VS. MENGOTE DOES NOT FIND APPLICATION IN THE INSTANT CASE. In support thereof, petitioner merely restates his arguments below regarding the validity of the warrantless arrest and search, then disagrees with the finding of the Court of Appeals that he was "attempting to commit a crime," as the evidence for the prosecution merely disclosed that he was "standing at the corner of Plaza Miranda and Quezon Boulevard" with his eyes "moving very fast" and "looking at

every person that come (sic) nearer (sic) to them." Finally, petitioner points out the factual similarities between his case and that of People v. Mengote to demonstrate that the Court of Appeals miscomprehended the latter. In its Comment, the Office of the Solicitor General prays that we affirm the challenged decision.. For being impressed with merit, we resolved to give due course to the petition. The challenged decision must immediately fall on jurisdictional grounds. To repeat, the penalty imposed by the trial court was: [N]ot less than SEVENTEEN (17) YEARS, FOUR (4) MONTHS AND ONE (1) DAY OF RECLUSION TEMPORAL, as minimum, and not more than THIRTY (30) YEARS OF RECLUSION PERPETUA, as maximum. The penalty provided by Section 3 of P.D. No. 1866 upon any person who shall unlawfully possess grenades is reclusion temporal in its maximum period to reclusion perpetua. For purposes of determining appellate jurisdiction in criminal cases, the maximum of the penalty, and not the minimum, is taken into account. Since the maximum of the penalty is reclusion perpetua, the appeal therefrom should have been to us, and not the Court of Appeals, pursuant to Section 9(3) of the Judiciary Reorganization Act of 1980 (B.P. Blg. 129), 27 in relation to Section 17 of the Judiciary Act of 1948, 28 Section 5(2) of Article VIII of the Constitution 29 and Section 3(c) of Rule 122 of the Rules of Court. 30 The term "life imprisonment" as used in Section 9 of B.P. Blg. 129, the Judiciary Act of 1948, and Section 3 of Rule 122 must be deemed to include reclusion perpetua in view of Section 5(2) of Article VIII of the Constitution. Petitioner's Notice of Appeal indicated that he was appealing from the trial court's decision to this Court, yet the trial court transmitted the record to the Court of Appeals and the latter proceeded to resolve the appeal. We then set aside the decision of the Court of Appeals for having been rendered without jurisdiction, and consider the appeal as having been directly brought to us, with the petition for review as petitioner's Brief for the Appellant, the comment thereon by the Office of the Solicitor General as the Brief for the Appellee and the memoranda of the parties as their Supplemental Briefs. Deliberating on the foregoing pleadings, we find ourselves convinced that the prosecution failed to establish petitioner's guilt with moral certainty. First, serious doubt surrounds the story of police officer Yu that a grenade was found in and seized from petitioner's possession. Notably, Yu did not identify, in

court, the grenade he allegedly seized. According to him, he turned it over to his commander after putting an "X" mark at its bottom; however, the commander was not presented to corroborate this claim. On the other hand, the grenade presented in court and identified by police officer Ramilo referred to what the latter received from Lt. Eduardo Cabrera and police officer Diotoy not immediately after petitioner's arrest, but nearly seven (7) months later, or on 19 March 1991; further, there was no evidence whatsoever that what Ramilo received was the very same grenade seized from petitioner. In his testimony, Yu never declared that the grenade passed on to Ramilo was the grenade the former confiscated from petitioner. Yu did not, and was not made to, identify the grenade examined by Ramilo, and the latter did not claim that the grenade he examined was that seized from petitioner. Plainly, the law enforcement authorities failed to safeguard and preserve the chain of evidence so crucial in cases such as these. Second, if indeed petitioner had a grenade with him, and that two days earlier he was with a group about to detonate an explosive at Plaza Miranda, and Yu and his fellow officers chased, but failed to arrest them, then considering that Yu and his three fellow officers were in uniform and therefore easily cognizable as police officers, it was then unnatural and against common experience that petitioner simply stood there in proximity to the police officers. Note that Yu observed petitioner for thirty minutes and must have been close enough to petitioner in order to discern petitioner's eyes "moving very fast." Finally, even assuming that petitioner admitted possession of the grenade during his custodial investigation by police officer Serapio, such admission was inadmissible in evidence for it was taken in palpable violation of Section 12(1) and (3) of Article III of the Constitution, which provide as follows: Sec. 12 (1). Any person under investigation for the commission of an offense shall have the right to be informed of his right to remain silent and to have competent and independent counsel preferably of his own choice. If the person cannot afford the services of counsel, he must be provided with one. These rights cannot be waived except in writing and in the presence of counsel. xxx xxx xxx (3) Any confession or admission obtained in violation of this or Section 17 hereof shall be inadmissible in evidence against him. Serapio conducted the custodial investigation on petitioner the day following his arrest. No lawyer was present and Serapio could not have requested a lawyer to assist petitioner as no PAO lawyer was then available. Thus, even if petitioner consented to the investigation and waived his rights to remain silent and to counsel, the waiver was invalid as it was not in writing, neither was it executed in the presence of counsel.

Even granting ex gratia that petitioner was in possession of a grenade, the arrest and search of petitioner were invalid, as will be discussed below. The general rule as regards arrests, searches and seizures is that a warrant is needed in order to validly effect the same. 31 The Constitutional prohibition against unreasonable arrests, searches and seizures refers to those effected without a validly issued warrant, 32 subject to certain exceptions. As regards valid warrantless arrests, these are found in Section 5, Rule 113 of the Rules of Court, which reads, in part: Sec. 5. Arrest, without warrant; when lawful A peace officer or a private person may, without a warrant, arrest a person: (a) When, in his presence, the person to be arrested has committed, is actually committing, or is attempting to commit an offense; (b) When an offense has in fact just been committed, and he has personal knowledge of facts indicating that the person to be arrested has committed it; and (c) When the person to be arrested is a prisoner who has escaped . . . A warrantless arrest under the circumstances contemplated under Section 5(a) has been denominated as one "in flagrante delicto," while that under Section 5(b) has been described as a "hot pursuit" arrest. Turning to valid warrantless searches, they are limited to the following: (1) customs searches; (2) search of moving vehicles; (3) seizure of evidence in plain view; (4) consent searches; 33 (5) a search incidental to a lawful arrest; 34 and (6) a "stop and frisk." 35 In the instant petition, the trial court validated the warrantless search as a "stop and frisk" with "the seizure of the grenade from the accused [as an appropriate incident to his arrest," hence necessitating a brief discussion on the nature of these exceptions to the warrant requirement. At the outset, we note that the trial court confused the concepts of a "stop-andfrisk" and of a search incidental to a lawful arrest. These two types of warrantless searches differ in terms of the requisite quantum of proof before they may be validly effected and in their allowable scope. In a search incidental to a lawful arrest, as the precedent arrest determines the validity of the incidental search, the legality of the arrest is questioned in a large majority of these cases, e.g., whether an arrest was merely used as a pretext for conducting a search. 36 In this instance, the law requires that there first be a lawful arrest before a search can be made the process cannot be reversed. 37 At bottom, assuming a valid arrest, the arresting officer may search the person of

the arrestee and the area within which the latter may reach for a weapon or for evidence to destroy, and seize any money or property found which was used in the commission of the crime, or the fruit of the crime, or that which may be used as evidence, or which might furnish the arrestee with the means of escaping or committing violence. 38 Here, there could have been no valid in flagrante delicto or hot pursuit arrest preceding the search in light of the lack of personal knowledge on the part of Yu, the arresting officer, or an overt physical act, on the part of petitioner, indicating that a crime had just been committed, was being committed or was going to be committed. Having thus shown the invalidity of the warrantless arrest in this case, plainly, the search conducted on petitioner could not have been one incidental to a lawful arrest. We now proceed to the justification for and allowable scope of a "stop-and-frisk" as a "limited protective search of outer clothing for weapons," as laid down in Terry, thus: We merely hold today that where a police officer observes unusual conduct which leads him reasonably to conclude in light of his experience that criminal activity may be afoot and that the persons with whom he is dealing may be armed and presently dangerous, where in the course of investigating this behavior he identifies himself as a policeman and makes reasonable inquiries, and where nothing in the initial stages of the encounter serves to dispel his reasonable fear for his own or others' safety, he is entitled for the protection of himself and others in the area to conduct a carefully limited search of the outer clothing of such persons in an attempt to discover weapons which might be used to assault him. Such a search is a reasonable search under the Fourth Amendment . . . 39 Other notable points of Terry are that while probable cause is not required to conduct a "stop and frisk," 40 it nevertheless holds that mere suspicion or a hunch will not validate a "stop and frisk." A genuine reason must exist, in light of the police officer's experience and surrounding conditions, to warrant the belief that the person detained has weapons concealed about him. 41 Finally, a "stopand-frisk" serves a two-fold interest: (1) the general interest of effective crime prevention and detection, which underlies the recognition that a police officer may, under appropriate circumstances and in an appropriate manner, approach a person for purposes of investigating possible criminal behavior even without probable cause; and (2) the more pressing interest of safety and selfpreservation which permit the police officer to take steps to assure himself that the person with whom he deals is not armed with a deadly weapon that could unexpectedly and fatally be used against the police officer.

Here, there are at least three (3) reasons why the "stop-and-frisk" was invalid: First, we harbor grave doubts as to Yu's claim that petitioner was a member of the group which attempted to bomb Plaza Miranda two days earlier. This claim is neither supported by any police report or record nor corroborated by any other police officer who allegedly chased that group. Aside from impairing Yu's credibility as a witness, this likewise diminishes the probability that a genuine reason existed so as to arrest and search petitioner. If only to further tarnish the credibility of Yu's testimony, contrary to his claim that petitioner and his companions had to be chased before being apprehended, the affidavit of arrest (Exh. "A") expressly declares otherwise, i.e., upon arrival of five (5) other police officers, petitioner and his companions were "immediately collared." Second, there was nothing in petitioner's behavior or conduct which could have reasonably elicited even mere suspicion other than that his eyes were "moving very fast" an observation which leaves us incredulous since Yu and his teammates were nowhere near petitioner and it was already 6:30 p.m., thus presumably dusk. Petitioner and his companions were merely standing at the corner and were not creating any commotion or trouble, as Yu explicitly declared on cross-examination: Q And what were they doing? A They were merely standing. Q You are sure of that? A Yes, sir. Q And when you saw them standing, there were nothing or they did not create any commotion. A None, sir. Q Neither did you see them create commotion? A None, sir. 42 Third, there was at all no ground, probable or otherwise, to believe that petitioner was armed with a deadly weapon. None was visible to Yu, for as he admitted, the alleged grenade was "discovered" "inside the front waistline" of petitioner, and from all indications as to the distance between Yu and petitioner, any telltale bulge, assuming that petitioner was indeed hiding a grenade, could not have been visible to Yu. In fact, as noted by the trial court:

When the policemen approached the accused and his companions, they were not yet aware that a handgrenade was tucked inside his waistline. They did not see any bulging object in [sic] his person. 43 What is unequivocal then in this case are blatant violations of petitioner's rights solemnly guaranteed in Sections 2 and 12(1) of Article III of the Constitution. WHEREFORE, the challenged decision of the Seventeenth Division of the Court of Appeals in CA-G.R. CR No. 15988 is SET ASIDE for lack of jurisdiction on the part of said Court and, on ground of reasonable doubt, the decision of 10 February 1994 of Branch 5 of the Regional Trial Court of Manila is REVERSED and petitioner SAMMY MALACAT y MANDAR is hereby ACQUITTED and ORDERED immediately released from detention, unless his further detention is justified for any other lawful cause. Costs de oficio. SO ORDERED.

G.R.No. 74869 July 6, 1988 PEOPLE OF THE PHILIPPINES, plaintiff-appellee, vs. IDEL AMINNUDIN y AHNI, defendant-appellant. The Solicitor General for plaintiff-appellee. Herminio T. Llariza counsel de-officio for defendant-appellant.

CRUZ, J.: The accused-appellant claimed his business was selling watches but he was nonetheless arrested, tried and found guilty of illegally transporting marijuana. The trial court, disbelieving him, held it was high time to put him away and sentenced him to life imprisonment plus a fine of P20,000.00. 1 Idel Aminnudin was arrested on June 25, 1984, shortly after disembarking from the M/V Wilcon 9 at about 8:30 in the evening, in Iloilo City. The PC officers who were in fact waiting for him simply accosted him, inspected his bag and finding what looked liked marijuana leaves took him to their headquarters for investigation. The two bundles of suspect articles were confiscated from him and later taken to the NBI laboratory for examination. When they were verified as marijuana leaves, an information for violation of the Dangerous Drugs Act was filed against him. 2 Later, the information was amended to include Farida Ali y Hassen, who had also been arrested with him that same evening and likewise investigated. 3 Both were arraigned and pleaded not guilty. 4 Subsequently, the fiscal filed a motion to dismiss the charge against Ali on the basis of a sworn statement of the arresting officers absolving her after a 'thorough investigation." 5 The motion was granted, and trial proceeded only against the accused-appellant, who was eventually convicted . 6 According to the prosecution, the PC officers had earlier received a tip from one of their informers that the accused-appellant was on board a vessel bound for Iloilo City and was carrying marijuana. 7 He was Identified by name. 8 Acting on this tip, they waited for him in the evening of June 25, 1984, and approached him as he descended from the gangplank after the informer had pointed to him. 9 They detained him and inspected the bag he was carrying. It was found to contain three kilos of what were later analyzed as marijuana leaves by an NBI forensic examiner, 10 who testified that she conducted microscopic, chemical and chromatographic tests on them. On the basis of this finding, the corresponding charge was then filed against Aminnudin. In his defense, Aminnudin disclaimed the marijuana, averring that all he had in his bag was his clothing consisting of a jacket, two shirts and two pairs of pants.

11

He alleged that he was arbitrarily arrested and immediately handcuffed. His bag was confiscated without a search warrant. At the PC headquarters, he was manhandled to force him to admit he was carrying the marijuana, the investigator hitting him with a piece of wood in the chest and arms even as he parried the blows while he was still handcuffed. 12 He insisted he did not even know what marijuana looked like and that his business was selling watches and sometimes cigarettes. 13 He also argued that the marijuana he was alleged to have been carrying was not properly Identified and could have been any of several bundles kept in the stock room of the PC headquarters. 14 The trial court was unconvinced, noting from its own examination of the accused that he claimed to have come to Iloilo City to sell watches but carried only two watches at the time, traveling from Jolo for that purpose and spending P107.00 for fare, not to mention his other expenses. 15 Aminnudin testified that he kept the two watches in a secret pocket below his belt but, strangely, they were not discovered when he was bodily searched by the arresting officers nor were they damaged as a result of his manhandling. 16 He also said he sold one of the watches for P400.00 and gave away the other, although the watches belonged not to him but to his cousin, 17 to a friend whose full name he said did not even know. 18 The trial court also rejected his allegations of maltreatment, observing that he had not sufficiently proved the injuries sustained by him. 19 There is no justification to reverse these factual findings, considering that it was the trial judge who had immediate access to the testimony of the witnesses and had the opportunity to weigh their credibility on the stand. Nuances of tone or voice, meaningful pauses and hesitation, flush of face and dart of eyes, which may reveal the truth or expose the lie, are not described in the impersonal record. But the trial judge sees all of this, discovering for himself the truant fact amidst the falsities. The only exception we may make in this case is the trial court's conclusion that the accused-appellant was not really beaten up because he did not complain about it later nor did he submit to a medical examination. That is hardly fair or realistic. It is possible Aminnudin never had that opportunity as he was at that time under detention by the PC authorities and in fact has never been set free since he was arrested in 1984 and up to the present. No bail has been allowed for his release. There is one point that deserves closer examination, however, and it is Aminnudin's claim that he was arrested and searched without warrant, making the marijuana allegedly found in his possession inadmissible in evidence against him under the Bill of Rights. The decision did not even discuss this point. For his part, the Solicitor General dismissed this after an all-too-short argument that the arrest of Aminnudin was valid because it came under Rule 113, Section 6(b) of the Rules of Court on warrantless arrests. This made the search also valid as incidental to a lawful arrest.

It is not disputed, and in fact it is admitted by the PC officers who testified for the prosecution, that they had no warrant when they arrested Aminnudin and seized the bag he was carrying. Their only justification was the tip they had earlier received from a reliable and regular informer who reported to them that Aminnudin was arriving in Iloilo by boat with marijuana. Their testimony varies as to the time they received the tip, one saying it was two days before the arrest, 20 another two weeks 21 and a third "weeks before June 25." 22 On this matter, we may prefer the declaration of the chief of the arresting team, Lt. Cipriano Querol, Jr., who testified as follows: Q You mentioned an intelligence report, you mean with respect to the coming of Idel Aminnudin on June 25, 1984? A Yes, sir. Q When did you receive this intelligence report? A Two days before June 25, 1984 and it was supported by reliable sources. Q Were you informed of the coming of the Wilcon 9 and the possible trafficking of marijuana leaves on that date? A Yes, sir, two days before June 25, 1984 when we received this information from that particular informer, prior to June 25, 1984 we have already reports of the particular operation which was being participated by Idel Aminnudin. Q You said you received an intelligence report two days before June 25, 1984 with respect to the coming of Wilcon 9? A Yes, sir. Q Did you receive any other report aside from this intelligence report? A Well, I have received also other reports but not pertaining to the coming of Wilcon 9. For instance, report of illegal gambling operation. COURT: Q Previous to that particular information which you said two days before June 25, 1984, did you also receive daily report regarding the activities of Idel Aminnudin A Previous to June 25, 1984 we received reports on the activities of Idel Aminnudin. Q What were those activities?

A Purely marijuana trafficking. Q From whom did you get that information? A It came to my hand which was written in a required sheet of information, maybe for security reason and we cannot Identify the person. Q But you received it from your regular informer? A Yes, sir. ATTY. LLARIZA: Q Previous to June 25, 1984, you were more or less sure that Idel Aminnudin is coming with drugs? A Marijuana, sir. Q And this information respecting Idel Aminnudin's coming to Iloilo with marijuana was received by you many days before you received the intelligence report in writing? A Not a report of the particular coming of Aminnudin but his activities. Q You only knew that he was coming on June 25,1984 two days before? A Yes, sir. Q You mean that before June 23, 1984 you did not know that minnudin was coming? A Before June 23,1984, I, in my capacity, did not know that he was coming but on June 23, 1984 that was the time when I received the information that he was coming. Regarding the reports on his activities, we have reports that he was already consummated the act of selling and shipping marijuana stuff. COURT: Q And as a result of that report, you put him under surveillance? A Yes, sir. Q In the intelligence report, only the name of Idel Aminnudin was mentioned? A Yes, sir.

Q Are you sure of that? A On the 23rd he will be coming with the woman. Q So that even before you received the official report on June 23, 1984, you had already gathered information to the effect that Idel Aminnudin was coming to Iloilo on June 25, 1984? A Only on the 23rd of June. Q You did not try to secure a search warrant for the seizure or search of the subject mentioned in your intelligence report? A No, more. Q Why not? A Because we were very very sure that our operation will yield positive result. Q Is that your procedure that whenever it will yield positive result you do not need a search warrant anymore? A Search warrant is not necessary. 23 That last answer is a cavalier pronouncement, especially as it comes from a mere lieutenant of the PC. The Supreme Court cannot countenance such a statement. This is still a government of laws and not of men. The mandate of the Bill of Rights is clear: Sec. 2. The right of the people to be secure in their persons, houses, papers and effects against unreasonable searches and seizures of whatever nature and for any purpose shall be inviolable, and no search warrant or warrant of arrest shall issue except upon probable cause to be determined personally by the judge after examination under oath or affirmation of the complainant and the witnesses he may produce, and particularly describing the place to be searched and the persons or things to be seized. In the case at bar, there was no warrant of arrest or search warrant issued by a judge after personal determination by him of the existence of probable cause. Contrary to the averments of the government, the accused-appellant was not caught in flagrante nor was a crime about to be committed or had just been committed to justify the warrantless arrest allowed under Rule 113 of the Rules of Court. Even expediency could not be invoked to dispense with the obtention of the warrant as in the case of Roldan v. Arca, 24 for example. Here it was held that vessels and aircraft are subject to warrantless searches and seizures for violation

of the customs law because these vehicles may be quickly moved out of the locality or jurisdiction before the warrant can be secured. The present case presented no such urgency. From the conflicting declarations of the PC witnesses, it is clear that they had at least two days within which they could have obtained a warrant to arrest and search Aminnudin who was coming to Iloilo on the M/V Wilcon 9. His name was known. The vehicle was Identified. The date of its arrival was certain. And from the information they had received, they could have persuaded a judge that there was probable cause, indeed, to justify the issuance of a warrant. Yet they did nothing. No effort was made to comply with the law. The Bill of Rights was ignored altogether because the PC lieutenant who was the head of the arresting team, had determined on his own authority that a "search warrant was not necessary." In the many cases where this Court has sustained the warrantless arrest of violators of the Dangerous Drugs Act, it has always been shown that they were caught red-handed, as a result of what are popularly called "buy-bust" operations of the narcotics agents. 25 Rule 113 was clearly applicable because at the precise time of arrest the accused was in the act of selling the prohibited drug. In the case at bar, the accused-appellant was not, at the moment of his arrest, committing a crime nor was it shown that he was about to do so or that he had just done so. What he was doing was descending the gangplank of the M/V Wilcon 9 and there was no outward indication that called for his arrest. To all appearances, he was like any of the other passengers innocently disembarking from the vessel. It was only when the informer pointed to him as the carrier of the marijuana that he suddenly became suspect and so subject to apprehension. It was the furtive finger that triggered his arrest. The Identification by the informer was the probable cause as determined by the officers (and not a judge) that authorized them to pounce upon Aminnudin and immediately arrest him. Now that we have succeeded in restoring democracy in our country after fourteen years of the despised dictatorship, when any one could be picked up at will, detained without charges and punished without trial, we will have only ourselves to blame if that kind of arbitrariness is allowed to return, to once more flaunt its disdain of the Constitution and the individual liberties its Bill of Rights guarantees. While this is not to say that the accused-appellant is innocent, for indeed his very own words suggest that he is lying, that fact alone does not justify a finding that he is guilty. The constitutional presumption is that he is innocent, and he will be so declared even if his defense is weak as long as the prosecution is not strong enough to convict him. Without the evidence of the marijuana allegedly seized from Aminnudin, the case of the prosecution must fall. That evidence cannot be admitted, and should never

have been considered by the trial court for the simple fact is that the marijuana was seized illegally. It is the fruit of the poisonous tree, to use Justice Holmes' felicitous phrase. The search was not an incident of a lawful arrest because there was no warrant of arrest and the warrantless arrest did not come under the exceptions allowed by the Rules of Court. Hence, the warrantless search was also illegal and the evidence obtained thereby was inadmissible. The Court strongly supports the campaign of the government against drug addiction and commends the efforts of our law-enforcement officers against those who would inflict this malediction upon our people, especially the susceptible youth. But as demanding as this campaign may be, it cannot be more so than the compulsions of the Bill of Rights for the protection of the liberty of every individual in the realm, including the basest of criminals. The Constitution covers with the mantle of its protection the innocent and the guilty alike against any manner of high- handedness from the authorities, however praiseworthy their intentions. Those who are supposed to enforce the law are not justified in disregarding the rights of the individual in the name of order. Order is too high a price for the loss of liberty. As Justice Holmes, again, said, "I think it a less evil that some criminals should escape than that the government should play an ignoble part." It is simply not allowed in the free society to violate a law to enforce another, especially if the law violated is the Constitution itself. We find that with the exclusion of the illegally seized marijuana as evidence against the accused-appellant, his guilt has not been proved beyond reasonable doubt and he must therefore be discharged on the presumption that he is innocent. ACCORDINGLY, the decision of the trial court is REVERSED and the accusedappellant is ACQUITTED. It is so ordered.

G.R. No. 91107 June 19, 1991 THE PEOPLE OF THE PHILIPPINES,Plainti ff- Appellee, vs. MIKAEL MALMSTEDT, *defendant-appellant. PADILLA,J.: In an information dated 15 June 1989, accused-appellant Mikael Malmstedt (hereinafter referred to as the accused) was charged before the Regional Trial Court (RTC) of La Trinidad, Benguet, Branch 10, in Criminal Case No. 89-CR-0663, for violation of Section 4, Art. II of Republic Act 6425, as amended, otherwise known as the Dangerous Drugs Act of 1972, as amended. The factual background of the case is as follows: Accused Mikael Malmstedt, a Swedish national, entered the Philippines for the third time in December 1988 as a tourist. He had visited the country sometime in 1982 and 1985. In the evening of 7 May 1989, accused left for Baguio City. Upon his arrival thereat in the morning of the following day, he took a bus to Sagada and stayed in that place for two (2) days. At around 7:00 o'clock in the morning of 11 May 1989, accused went to the Nangonogan bus stop in Sagada to catch the first available trip to Baguio City. From Baguio City, accused planned to take a late afternoon trip to Angeles City, then proceed to Manila to catch his flight out of the country, scheduled on 13 May 1989. From Sagada, accused took a Skyline bus with body number 8005 and Plate number AVC 902.1 At about 8: 00 o'clock in the morning of that same day (11 May 1989), Captain Alen Vasco, the Commanding Officer of the First Regional Command (NARCOM) stationed at Camp Dangwa, ordered his men to set up a temporary checkpoint at Kilometer 14, Acop, Tublay, Mountain Province, for the purpose of checking all vehicles coming from the Cordillera Region. The order to establish a checkpoint in the said area was prompted by persistent reports that vehicles coming from Sagada were transporting marijuana and other prohibited drugs. Moreover, information was received by the Commanding Officer of NARCOM, that same morning, that a Caucasian coming from Sagada had in his possession prohibited drugs.2 The group composed of seven (7) NARCOM officers, in coordination with Tublay Police Station, set up a checkpoint at the designated area at about 10:00 o'clock in the morning and inspected all vehicles coming from the Cordillera Region. At about 1:30 o'clock in the afternoon, the bus where accused was riding was stopped. Sgt. Fider and CIC Galutan boarded the bus and announced that they were members of the NARCOM and that they would conduct an inspection. The two (2)

NARCOM officers started their inspection from the front going towards the rear of the bus. Accused who was the sole foreigner riding the bus was seated at the rear thereof. During the inspection, CIC Galutan noticed a bulge on accused's waist. Suspecting the bulge on accused's waist to be a gun, the officer asked for accused's passport and other identification papers. When accused failed to comply, the officer required him to bring out whatever it was that was bulging on his waist. The bulging object turned out to be a pouch bag and when accused opened the same bag, as ordered, the officer noticed four (4) suspicious-looking objects wrapped in brown packing tape, prompting the officer to open one of the wrapped objects. The wrapped objects turned out to contain hashish, a derivative of marijuana. Thereafter, accused was invited outside the bus for questioning. But before he alighted from the bus, accused stopped to get two (2) travelling bags from the luggage carrier. Upon stepping out of the bus, the officers got the bags and opened them. A teddy bear was found in each bag. Feeling the teddy bears, the officer noticed that there were bulges inside the same which did not feel like foam stuffing. It was only after the officers had opened the bags that accused finally presented his passport. Accused was then brought to the headquarters of the NARCOM at Camp Dangwa, La Trinidad, Benguet for further investigation. At the investigation room, the officers opened the teddy bears and they were found to also contain hashish. Representative samples were taken from the hashish found among the personal effects of accused and the same were brought to the PC Crime Laboratory for chemical analysis. In the chemistry report, it was established that the objects examined were hashish. a prohibited drug which is a derivative of marijuana. Thus, an information was filed against accused for violation of the Dangerous Drugs Act. During the arraignment, accused entered a plea of "not guilty." For his defense, he raised the issue of illegal search of his personal effects. He also claimed that the hashish was planted by the NARCOM officers in his pouch bag and that the two (2) travelling bags were not owned by him, but were merely entrusted to him by an Australian couple whom he met in Sagada. He further claimed that the Australian couple intended to take the same bus with him but because there were no more seats available in said bus, they decided to take the next ride and asked accused to take charge of the bags, and that they would meet each other at the Dangwa Station. Likewise, accused alleged that when the NARCOM officers demanded for his passport and other Identification papers, he handed to one of the officers his pouch bag which was hanging on his neck containing, among others, his passport, return ticket to Sweden and other papers. The officer in turn handed it to his companion who brought the bag outside the bus. When said officer came back, he charged the accused that there was hashish in the bag. He was told to get off the bus and his

picture was taken with the pouch bag placed around his neck. The trial court did not give credence to accused's defense. The claim of the accused that the hashish was planted by the NARCOM officers, was belied by his failure to raise such defense at the earliest opportunity. When accused was investigated at the Provincial Fiscal's Office, he did not inform the Fiscal or his lawyer that the hashish was planted by the NARCOM officers in his bag. It was only two (2) months after said investigation when he told his lawyer about said claim, denying ownership of the two (2) travelling bags as well as having hashish in his pouch bag. In a decision dated 12 October 1989, the trial court found accused guilty beyond reasonable doubt for violation of the Dangerous Drugs Act, specifically Section 4, Art. II of RA 6425, as amended.3 The dispositive portion of the decision reads as follows: WHEREFORE, finding the guilt of the accused Mikael Malmstedt established beyond reasonable doubt, this Court finds him GUILTY of violation of Section 4, Article 11 of Republic Act 6425, as amended, and hereby sentences him to suffer the penalty of life imprisonment and to pay a fine of Twenty Thousand Pesos (P20,000.00), with subsidiary imprisonment in case of insolvency and to pay the costs. Let the hashish subject of this case be turned over to the First Narcotics Regional Unit at Camp Bado; Dangwa, La Trinidad Benguet for proper disposition under Section 20, Article IV of Republic Act 6425, as amended. SO ORDERED.4 Seeking the reversal of the decision of the trial court finding him guilty of the crime charged, accused argues that the search of his personal effects was illegal because it was made without a search warrant and, therefore, the prohibited drugs which were discovered during the illegal search are not admissible as evidence against him. The Constitution guarantees the right of the people to be secure in their persons, houses, papers and effects against unreasonable searches and seizures.5 However, where the search is made pursuant to a lawful arrest, there is no need to obtain a search warrant. A lawful arrest without a warrant may be made by a peace officer or a private person under the following circumstances.6 Sec. 5 Arrest without warrant; when lawful. A peace officer or a private person may, without a warrant, arrest a person:

(a) When, in his presence, the person to be arrested has committed is actually committing, or is attempting to commit an offense; chanrobles virtual law library (b) When an offense has in fact just been committed, and he has personal knowledge of facts indicating that the person to be arrested has committed it; and chanrobles virtual law library (c) When the person to be arrested is a prisoner who has escaped from a penal establishment or place where he is serving final judgment or temporarily confined while his case is pending, or has escaped while being transferred from one confinement to another. In cases falling under paragraphs (a) and (b) hereof, the person arrested without a warrant shall be forthwith delivered to the nearest police station or jail, and he shall be proceeded against in accordance with Rule 112, Section 7. (6a 17a). Accused was searched and arrested while transporting prohibited drugs (hashish). A crime was actually being committed by the accused and he was caught inflagrante delicto. Thus, the search made upon his personal effects falls squarely under paragraph (1) of the foregoing provisions of law, which allow a warrantless search incident to a lawful arrest.7 While it is true that the NARCOM officers were not armed with a search warrant when the search was made over the personal effects of accused, however, under the circumstances of the case, there was sufficient probable cause for said officers to believe that accused was then and there committing a crime. Probable cause has been defined as such facts and circumstances which could lead a reasonable, discreet and prudent man to believe that an offense has been committed, and that the objects sought in connection with the offense are in the place sought to be searched.8 The required probable cause that will justify a warrantless search and seizure is not determined by any fixed formula but is resolved according to the facts of each case. 9 Warrantless search of the personal effects of an accused has been declared by this Court as valid, because of existence of probable cause, where the smell of marijuana emanated from a plastic bag owned by the accused,10 or where the accused was acting suspiciously, 11 and attempted to flee. 12

Aside from the persistent reports received by the NARCOM that vehicles coming from Sagada were transporting marijuana and other prohibited drugs, their Commanding Officer also received information that a Caucasian coming from Sagada on that particular day had prohibited drugs in his possession. Said information was received by the Commanding Officer of NARCOM the very same morning that accused came down by bus from Sagada on his way to Baguio City. When NARCOM received the information, a few hours before the apprehension of herein accused, that a Caucasian travelling from Sagada to Baguio City was carrying with him prohibited drugs, there was no time to obtain a search warrant. In the Tangliben case,13 the police authorities conducted a surveillance at the Victory Liner Terminal located at Bgy. San Nicolas, San Fernando Pampanga, against persons engaged in the traffic of dangerous drugs, based on information supplied by some informers. Accused Tangliben who was acting suspiciously and pointed out by an informer was apprehended and searched by the police authorities. It was held that when faced with on-the-spot information, the police officers had to act quickly and there was no time to secure a search warrant. It must be observed that, at first, the NARCOM officers merely conducted a routine check of the bus (where accused was riding) and the passengers therein, and no extensive search was initially made. It was only when one of the officers noticed a bulge on the waist of accused, during the course of the inspection, that accused was required to present his passport. The failure of accused to present his identification papers, when ordered to do so, only managed to arouse the suspicion of the officer that accused was trying to hide his identity. For is it not a regular norm for an innocent man, who has nothing to hide from the authorities, to readily present his identification papers when required to do so? The receipt of information by NARCOM that a Caucasian coming from Sagada had prohibited drugs in his possession, plus the suspicious failure of the accused to produce his passport, taken together as a whole, led the NARCOM officers to reasonably believe that the accused was trying to hide something illegal from the authorities. From these circumstances arose a probable cause which justified the warrantless search that was made on the personal effects of the accused. In other words, the acts of the NARCOM officers in requiring the accused to open his pouch bag and in opening one of the wrapped objects found inside said bag (which was discovered to contain hashish) as well as the two (2) travelling bags containing two (2) teddy bears with hashish stuffed inside them, were prompted by accused's own attempt to hide his identity by refusing to present his passport, and by the information received by the NARCOM that a Caucasian coming from Sagada had prohibited drugs in his possession. To deprive the NARCOM agents of the ability and facility to act accordingly, including, to search even without warrant, in the light of such circumstances, would be to sanction impotence and ineffectiveness in law enforcement, to the detriment of society. WHEREFORE, premises considered, the appealed judgment of conviction by the trial court is hereby AFFIRMED. Costs against the accused-appellant.

SO ORDERED.

G.R. No. 120431 April 1, 1998 RODOLFO ESPANO, accused-petitioner, vs. COURT OF APPEALS and PEOPLE OF THE PHILIPPINES, respondents.

ROMERO, J.: This is a petition for review of the decision of the Court of Appeals in CAG.R. CR No. 13976 dated January 16, 1995, 1 which affirmed in toto the judgment of the Regional Trial Court of Manila, Branch 1, convincing petitioner Rodolfo Espano for violation of Article II, Section 8 of Republic Act No. 6425, as amended, otherwise known as the Dangerous Drugs Act.

Petitioner was charged under the following information: That on or about July 14, 1991, in the City of Manila, Philippines, the said accused not being authorized by law to possess or use any prohibited drug, did then and there willfully, unlawfully and knowingly have in his possession and under his custody and control twelve (12) plastic cellophane (bags) containing crushed flowering tops, marijuana weighing 5.5 grams which is a prohibited drug. Contrary to law. 2
The evidence for the prosecution, based on the testimony of Pat. Romeo Pagilagan, shows that on July 14, 1991, at about 12:30 a.m., he and other police officers, namely, Pat. Wilfredo Aquino, Simplicio Rivera, and Erlindo Lumboy of the Western Police District (WPD), Narcotics Division went to Zamora and Pandacan Streets, Manila to confirm reports of drug pushing in the area. They saw petitioner selling "something" to another person. After the alleged buyer left, they approached petitioner, identified themselves as policemen, and frisked him. The search yielded two plastic cellophane tea bags of marijuana. When asked if he had more marijuana, he replied that there was more in his house. The policemen went to his residence where they found ten more cellophane tea bags of marijuana. Petitioner was brought to the police headquarters where he was charged with possession of prohibited drugs. On July 24, 1991, petitioner posted bail 3 and the trial court issued his order of release on July 29, 1991. 4

Annabelle Alip, forensic chemist of the WPD Criminal Investigation Laboratory Section, testified that the articles sent to her by Pat. Wilfredo Aquino regarding the apprehension of a certain Rodolfo Espano for examination tested positive for marijuana, with a total weight of 5.5 grams. By way of defense, petitioner testified that on said evening, he was sleeping in his house and was awakened only when the policemen handcuffed him. He alleged that the policemen were looking for his brother-in-law Lauro, and when they could not find the latter, he was instead brought to the police station for investigation and later indicted for possession of prohibited drugs. His wife Myrna corroborated his story. The trial court rejected petitioner's, defense as a "mere afterthought" and found the version of the prosecution "more credible and trustworthy." Thus, on August 14, 1992, the trial court rendered a decision, convicting petitioner of the crime charged, the dispositive portion of which reads: WHEREFORE there being proof beyond reasonable doubt, the court finds the accused Rodolfo Espano y Valeria guilty of the crime of violation of Section 8, Article II, in relation to Section 2 (e-L) (I) of Republic Act No. 6425 as amended by Batas Pambansa Blg. 179, and pursuant to law hereby sentences him to suffer imprisonment of six (6) years and one (1) day to twelve (12) years and to pay a fine of P6,000.00 with subsidiary imprisonment in case of default plus costs. The marijuana is declared forfeited in favor of government and shall be turned over to the Dangerous Drugs Board without delay. SO ORDERED. 5 Petitioner appealed the decision to the Court of Appeals. The appellate court, however, affirmed the decision of the trial court in toto. Hence, this petition.

Petitioner contends that the trial and appellate courts erred in convicting him on the basis of the following: (a) the pieces of evidence seized were inadmissible; (b) the superiority of his constitutional right to be presumed innocent over the doctrine of presumption of regularity, (c) he was denied the constitutional right of confrontation and to compulsory process; and (d) his conviction was based on evidence which was irrelevant and not properly identified. After a careful examination of the records of the case, this Court finds no compelling reason sufficient to reverse the decisions of the trial and appellate courts.
First, it is a well settled doctrine that findings of trial courts on the credibility of witnesses deserve a high degree of respect. Having observed the deportment of witnesses during the trial, the trial judge is in a better position to determine the issue of credibility and, thus, his findings will not be disturbed during appeal in the absence of any clear showing that he had overlooked, misunderstood or misapplied some facts or circumstances of weight and substance which could have altered the conviction of the appellants. 6

In this case, the findings of the trial court that the prosecution witnesses were more credible than those of the defense must stand. Petitioner failed to show that Pat. Pagilagan, in testifying against him, was motivated by reasons other than his duty to curb drug abuse and had any intent to falsely impute to him such a serious crime as possession of prohibited drugs. In the absence of such ill motive, the presumption of regularity in the performance of his official duty must prevail.
In People v. Velasco, 7 this Court reiterated the doctrine of presumption of regularity in the performance of official duty which provides:

. . . Appellant failed to establish that Pat. Godoy and the other members of the buy-bust team are policemen engaged in mulcting or other unscrupulous activities who were motivated either by the desire to extort money or exact personal vengeance, or by sheer whim and caprice, when they entrapped her. And in the absence of proof of any intent on the part of the police authorities to falsely impute such a serious crime against appellant, as in this case, the presumption of regularity in the performance of official duty, . . . , must prevail over

the self-serving and uncorroborated claim of appellant that she had been framed. 8
Furthermore, the defense set up by petitioner does not deserve any consideration. He simply contended that he was in his house sleeping at the time of the incident. This Court has consistently held that alibi is the weakest of all defenses; and for it to prosper, the accused has the burden of proving that he was not at the scene of the crime at the time of its commission and that it was physically impossible for him to be there. Moreover, the "claim of a 'frame-up', like alibi, is a defense that has been invariably viewed by the Court with disfavor for it can just as easily be concocted but difficult to prove, and is a common and standard line of defense in most prosecutions arising from violations of the Dangerous Drugs Act." 9 No clear and convincing evidence was presented by petitioner to prove his defense of alibi.

Second, petitioner contends that the prosecution's failure to present the alleged informant in court cast a reasonable doubt which warrants his acquittal. This is again without merit, since failure of the prosecution to produce the informant in court is of no moment especially when he is not even the best witness to establish the fact that a buy-bust operation had indeed been conducted. In this case, Pat. Pagilagan, one of the policemen who apprehended petitioner, testified on the actual incident of July 14, 1991, and identified him as the one they caught in possession of prohibited drugs. Thus, We find that the prosecution had satisfactorily proved its case against appellants. There is no compelling reason for us to overturn the finding of the trial court that the testimony of Sgt. Gamboa, the lone witness for the prosecution, was straightforward spontaneous and convincing. The testimony of a sole witness, if credible and positive and satisfies the court beyond reasonable doubt, is sufficient to convict. 10 Thus on the basis of Pat. Pagilagan's testimony, the prosecution was able to prove that petitioner indeed committed the crime charged; consequently, the finding of conviction was proper. Lastly, the issue on the admissibility of the marijuana seized should likewise be ruled upon. Rule 113 Section 5(a) of the Rules of Court provides:

A peace officer or a private person may, without a warrant, arrest a person: a. when, in his presence, the person to be arrested has committed, is actually committing, or is attempting to commit an offense; xxx xxx xxx Petitioner's arrest falls squarely under the aforecited rule. He was caught in flagranti as a result of a buy-bust operation conducted by police officers on the basis of information received regarding the illegal trade of drugs within the area of Zamora and Pandacan Streets, Manila. The police officer saw petitioner handing over something to an alleged buyer. After the buyer left, they searched him and discovered two cellophanes of marijuana. His arrest was, therefore, lawful and the two cellophane bags of marijuana seized were admissible in evidence, being the fruits of the crime. As for the ten cellophane bags of marijuana found at petitioner's residence, however, the same are inadmissible in evidence. The 1987 Constitution guarantees freedom against unreasonable searches and seizures under Article III, Section 2 which provides: The right of the people to be secure in their persons, houses, papers and effects against unreasonable searches and seizures of whatever nature and for any purpose shall be inviolable, and no search warrant or warrant of arrest shall issue except upon probable cause to be determined personally by the judge after examination under oath or affirmation of the complainant and the witnesses he may produce, and particularly describing the place to be searched and the persons or things to be seized.
An exception to the said rule is a warrantless search incidental to a lawful arrest for dangerous weapons or anything which may be used as proof of the commission of an offense. 11 It may extend beyond the person of the one arrested to include the premises or surroundings under his immediate control. In this case, the ten cellophane bags of marijuana seized at petitioner's house after his arrest at Pandacan and Zamora Streets do not fall under the said exceptions.

In the case of People v. Lua, 12 this Court held:

As regards the brick of marijuana found inside the appellant's house, the trial court correctly ignored it apparently in view of its inadmissibility. While initially the arrest as well as the body search was lawful, the warrantless search made inside the appellant's house became unlawful since the police operatives were not armed with a search warrant. Such search cannot fall under "search made incidental to a lawful arrest," the same being limited to body search and to that point within reach or control of the person arrested, or that which may furnish him with the means of committing violence or of escaping. In the case at bar, appellant was admittedly outside his house when he was arrested. Hence, it can hardly be said that the inner portion of his house was within his reach or control. The articles seized from petitioner during his arrest were valid under the doctrine of search made incidental to a lawful arrest. The warrantless search made in his house, however, which yielded ten cellophane bags of marijuana became unlawful since the police officers were not armed with a search warrant at the time. Moreover, it was beyond the reach and control of petitioner.
In sum, this Court finds petitioner Rodolfo Espano guilty beyond reasonable doubt of violating Article II, Section 8, in relation to Section 2 (e-L) (I) of Republic Act No. 6425, as amended. Under the said provision, the penalty imposed is six years and one day to twelve years and a fine ranging from six thousand to twelve thousand pesos. With the passage of Republic Act No. 7659, which took effect on December 31, 1993, the imposable penalty shall now depend on the quantity of drugs recovered. Under the provisions of Republic Act No. 7629, Section 20, and as interpreted in People v. Simon 13 and People v. Lara, 14 if the quantity of marijuana involved is less than 750 grams, the imposable penalty ranges from prision correccional to reclusion temporal. Taking into consideration that petitioner is not a habitual delinquent, the amendatory provision is favorable to him and the quantity of marijuana involved is less than 750 grams, the penalty imposed under Republic Act No. 7659 should be applied. There being no mitigating nor aggravating circumstances, the imposable penalty shall be prision correccional in its medium period. Applying the Indeterminate Sentence Law, the maximum penalty shall be taken from the medium period of prision correccional, which is two (2) years, four (4) months and one (1) day to four (4) years and two (2) months, while the minimum shall be taken from the penalty next lower in

degree, which is one (1) month and one (1) day to six (6) months of arresto mayor.

WHEREFORE, the instant petition is hereby DENIED. The decision of the Court of Appeals in C.A.-G.R. CR No. 13976 dated January 16, 1995 is AFFIRMED with the MODIFICATION that petitioner Rodolfo Espano is sentenced to suffer an indeterminate penalty of TWO (2) months and ONE (1) day of arresto mayor, as minimum to TWO (2) years, FOUR (4) months and ONE (1) day of prision correccional, as maximum. SO ORDERED.

G.R. No. L-27360

February 28, 1968

HON. RICARDO G. PAPA, as Chief of Police of Manila; HON. JUAN PONCE ENRILE, as Commissioner of Customs; PEDRO PACIS, as Collector of Customs of the Port of Manila; and MARTIN ALAGAO, as Patrolman of the Manila Police Department, petitioners, vs. REMEDIOS MAGO and HILARION U. JARENCIO, as Presiding Judge of Branch 23, Court of First Instance of Manila, respondents. Office of the Solicitor General for petitioners. Juan T. David for respondents. ZALDIVAR, J.: This is an original action for prohibition and certiorari, with preliminary injunction filed by Ricardo Papa, Chief of Police of Manila; Juan once Enrile, Commissioner of Customs; Pedro Pacis, Collector of Customs of the Port of Manila; and Martin Alagao, a patrolman of the Manila Police Department, against Remedios Mago and Hon. Hilarion Jarencio, Presiding Judge of Branch 23 of the Court of First Instance of Manila, praying for the annulment of the order issued by respondent Judge in Civil Case No. 67496 of the Court of First Instance of Manila under date of March 7, 1967, which authorized the release under bond of certain goods which were seized and held by petitioners in connection with the enforcement of the Tariff and Customs Code, but which were claimed by respondent Remedios Mago, and to prohibit respondent Judge from further proceeding in any manner whatsoever in said Civil Case No. 67496. Pending the determination of this case this Court issued a writ of preliminary injunction restraining the respondent Judge from executing, enforcing and/or implementing the questioned order in Civil Case No. 67496 and from proceeding with said case. Petitioner Martin Alagao, head of the counter-intelligence unit of the Manila Police Department, acting upon a reliable information received on November 3, 1966 to the effect that a certain shipment of personal effects, allegedly misdeclared and undervalued, would be released the following day from the customs zone of the port of Manila and loaded on two trucks, and upon orders of petitioner

Ricardo Papa, Chief of Police of Manila and a duly deputized agent of the Bureau of Customs, conducted surveillance at gate No. 1 of the customs zone. When the trucks left gate No. 1 at about 4:30 in the afternoon of November 4, 1966, elements of the counter-intelligence unit went after the trucks and intercepted them at the Agrifina Circle, Ermita, Manila. The load of the two trucks consisting of nine bales of goods, and the two trucks, were seized on instructions of the Chief of Police. Upon investigation, a person claimed ownership of the goods and showed to the policemen a "Statement and Receipts of Duties Collected in Informal Entry No. 147-5501", issued by the Bureau of Customs in the name of a certain Bienvenido Naguit. Claiming to have been prejudiced by the seizure and detention of the two trucks and their cargo, Remedios Mago and Valentin B. Lanopa filed with the Court of First Instance of Manila a petition "for mandamus with restraining order or preliminary injunction, docketed as Civil Case No. 67496, alleging, among others, that Remedios Mago was the owner of the goods seized, having purchased them from the Sta. Monica Grocery in San Fernando, Pampanga; that she hired the trucks owned by Valentin Lanopa to transport, the goods from said place to her residence at 1657 Laon Laan St., Sampaloc, Manila; that the goods were seized by members of the Manila Police Department without search warrant issued by a competent court; that anila Chief of Police Ricardo Papa denied the request of counsel for Remedios Mago that the bales be not opened and the goods contained therein be not examined; that then Customs Commissioner Jacinto Gavino had illegally assigned appraisers to examine the goods because the goods were no longer under the control and supervision of the Commissioner of Customs; that the goods, even assuming them to have been misdeclared and, undervalued, were not subject to seizure under Section 2531 of the Tariff and Customs Code because Remedios Mago had bought them from another person without knowledge that they were imported illegally; that the bales had not yet been opened, although Chief of Police Papa had arranged with the Commissioner of Customs regarding the disposition of the goods, and that unless restrained their constitutional rights would be violated and they would truly suffer irreparable injury. Hence, Remedios Mago and Valentin Lanopa prayed for the issuance of a restraining order, ex parte, enjoining the above-named police and customs authorities, or their agents, from

opening the bales and examining the goods, and a writ of mandamus for the return of the goods and the trucks, as well as a judgment for actual, moral and exemplary damages in their favor. On November 10, 1966, respondent Judge Hilarion Jarencio issued an order ex parte restraining the respondents in Civil Case No. 67496 now petitioners in the instant case before this Court from opening the nine bales in question, and at the same time set the hearing of the petition for preliminary injunction on November 16, 1966. However, when the restraining order was received by herein petitioners, some bales had already been opened by the examiners of the Bureau of Customs in the presence of officials of the Manila Police Department, an assistant city fiscal and a representative of herein respondent Remedios Mago. Under date of November 15, 1966, Remedios Mago filed an amended petition in Civil Case No. 67496, including as party defendants Collector of Customs Pedro Pacis of the Port of Manila and Lt. Martin Alagao of the Manila Police Department. Herein petitioners (defendants below) filed, on November 24, 1966, their "Answer with Opposition to the Issuance of a Writ of Preliminary Injunction", denying the alleged illegality of the seizure and detention of the goods and the trucks and of their other actuations, and alleging special and affirmative defenses, to wit: that the Court of First Instance of Manila had no jurisdiction to try the case; that the case fell within the exclusive jurisdiction of the Court of Tax Appeals; that, assuming that the court had jurisdiction over the case, the petition stated no cause of action in view of the failure of Remedios Mago to exhaust the administrative remedies provided for in the Tariff and Customs Code; that the Bureau of Customs had not lost jurisdiction over the goods because the full duties and charges thereon had not been paid; that the members of the Manila Police Department had the power to make the seizure; that the seizure was not unreasonable; and the persons deputized under Section 2203 (c) of the Tariff and Customs Code could effect search, seizures and arrests in inland places in connection with the enforcement of the said Code. In opposing the issuance of the writ of preliminary injunction, herein petitioners averred in the court below that the writ could not be granted for the reason that Remedios Mago was not entitled to the main reliefs she prayed for; that the release of the goods, which were

subject to seizure proceedings under the Tariff and Customs Code, would deprive the Bureau of Customs of the authority to forfeit them; and that Remedios Mago and Valentin Lanopa would not suffer irreparable injury. Herein petitioners prayed the court below for the lifting of the restraining order, for the denial of the issuance of the writ of preliminary injunction, and for the dismissal of the case. At the hearing on December 9, 1966, the lower Court, with the conformity of the parties, ordered that an inventory of the goods be made by its clerk of court in the presence of the representatives of the claimant of the goods, the Bureau of Customs, and the AntiSmuggling Center of the Manila Police Department. On December 13, 1966, the above-named persons filed a "Compliance" itemizing the contents of the nine bales. Herein respondent Remedios Mago, on December 23, 1966, filed an ex parte motion to release the goods, alleging that since the inventory of the goods seized did not show any article of prohibited importation, the same should be released as per agreement of the patties upon her posting of the appropriate bond that may be determined by the court. Herein petitioners filed their opposition to the motion, alleging that the court had no jurisdiction to order the release of the goods in view of the fact that the court had no jurisdiction over the case, and that most of the goods, as shown in the inventory, were not declared and were, therefore, subject to forfeiture. A supplemental opposition was filed by herein petitioners on January 19, 1967, alleging that on January 12, 1967 seizure proceedings against the goods had been instituted by the Collector of Customs of the Port of Manila, and the determination of all questions affecting the disposal of property proceeded against in seizure and forfeiture proceedings should thereby be left to the Collector of Customs. On January 30, 1967, herein petitioners filed a manifestation that the estimated duties, taxes and other charges due on the goods amounted to P95,772.00. On February 2, 1967, herein respondent Remedios Mago filed an urgent manifestation and reiteration of the motion for the release under bond of the goods. On March 7, 1967, the respondent Judge issued an order releasing the goods to herein respondent Remedios Mago upon her filing of a bond in the amount of P40,000.00, and on March 13, 1967,

said respondent filed the corresponding bond. On March 13, 1967, herein petitioner Ricardo Papa, on his own behalf, filed a motion for reconsideration of the order of the court releasing the goods under bond, upon the ground that the Manila Police Department had been directed by the Collector of Customs of the Port of Manila to hold the goods pending termination of the seizure proceedings. Without waiting for the court's action on the motion for reconsideration, and alleging that they had no plain, speedy and adequate remedy in the ordinary course of law, herein petitioners filed the present action for prohibition and certiorari with preliminary injunction before this Court. In their petition petitioners alleged, among others, that the respondent Judge acted without jurisdiction in ordering the release to respondent Remedios Mago of the disputed goods, for the following reasons: (1) the Court of First Instance of Manila, presided by respondent Judge, had no jurisdiction over the case; (2) respondent Remedios Mago had no cause of action in Civil Case No. 67496 of the Court of First Instance of Manila due to her failure to exhaust all administrative remedies before invoking judicial intervention; (3) the Government was not estopped by the negligent and/or illegal acts of its agent in not collecting the correct taxes; and (4) the bond fixed by respondent Judge for the release of the goods was grossly insufficient. In due time, the respondents filed their answer to the petition for prohibition and certiorari in this case. In their answer, respondents alleged, among others: (1) that it was within the jurisdiction of the lower court presided by respondent Judge to hear and decide Civil Case No. 67496 and to issue the questioned order of March 7, 1967, because said Civil Case No. 67496 was instituted long before seizure, and identification proceedings against the nine bales of goods in question were instituted by the Collector of Customs; (2) that petitioners could no longer go after the goods in question after the corresponding duties and taxes had been paid and said goods had left the customs premises and were no longer within the control of the Bureau of Customs; (3) that respondent Remedios Mago was purchaser in good faith of the goods in question so that those goods can not be the subject of seizure and forfeiture proceedings; (4) that

the seizure of the goods was affected by members of the Manila Police Department at a place outside control of jurisdiction of the Bureau of Customs and affected without any search warrant or a warrant of seizure and detention; (5) that the warrant of seizure and detention subsequently issued by the Collector of Customs is illegal and unconstitutional, it not being issued by a judge; (6) that the seizing officers have no authority to seize the goods in question because they are not articles of prohibited importation; (7) that petitioners are estopped to institute the present action because they had agreed before the respondent Judge that they would not interpose any objection to the release of the goods under bond to answer for whatever duties and taxes the said goods may still be liable; and (8) that the bond for the release of the goods was sufficient. The principal issue in the instant case is whether or not, the respondent Judge had acted with jurisdiction in issuing the order of March 7, 1967 releasing the goods in question. The Bureau of Customs has the duties, powers and jurisdiction, among others, (1) to assess and collect all lawful revenues from imported articles, and all other dues, fees, charges, fines and penalties, accruing under the tariff and customs laws; (2) to prevent and suppress smuggling and other frauds upon the customs; and (3) to enforce tariff and customs laws. 1 The goods in question were imported from Hongkong, as shown in the "Statement and Receipts of Duties Collected on Informal Entry". 2 As long as the importation has not been terminated the imported goods remain under the jurisdiction of the Bureau of customs. Importation is deemed terminated only upon the payment of the duties, taxes and other charges upon the articles, or secured to be paid, at the port of entry and the legal permit for withdrawal shall have been granted. 3 The payment of the duties, taxes, fees and other charges must be in full. 4 The record shows, by comparing the articles and duties stated in the aforesaid "Statement and Receipts of Duties Collected on Informal Entry" with the manifestation of the Office of the Solicitor General 5 wherein it is stated that the estimated duties, taxes and other charges on the goods subject of this case amounted to P95,772.00 as evidenced by the report of the appraiser of the Bureau

of Customs, that the duties, taxes and other charges had not been paid in full. Furthermore, a comparison of the goods on which duties had been assessed, as shown in the "Statement and Receipts of Duties Collected on Informal Entry" and the "compliance" itemizing the articles found in the bales upon examination and inventory, 6 shows that the quantity of the goods was underdeclared, presumably to avoid the payment of duties thereon. For example, Annex B (the statement and receipts of duties collected) states that there were 40 pieces of ladies' sweaters, whereas Annex H (the inventory contained in the "compliance") states that in bale No. 1 alone there were 42 dozens and 1 piece of ladies' sweaters of assorted colors; in Annex B, only 100 pieces of watch bands were assessed, but in Annex H, there were in bale No. 2, 209 dozens and 5 pieces of men's metal watch bands (white) and 120 dozens of men's metal watch band (gold color), and in bale No. 7, 320 dozens of men's metal watch bands (gold color); in Annex B, 20 dozens only of men's handkerchief were declared, but in Annex H it appears that there were 224 dozens of said goods in bale No. 2, 120 dozens in bale No. 6, 380 dozens in bale No. 7, 220 dozens in bale No. 8, and another 200 dozens in bale No. 9. The articles contained in the nine bales in question, were, therefore, subject to forfeiture under Section 2530, pars. e and m, (1), (3), (4), and (5) of the Tariff and Customs Code. And this Court has held that merchandise, the importation of which is effected contrary to law, is subject to forfeiture, 7 and that goods released contrary to law are subject to seizure and forfeiture. 8 Even if it be granted, arguendo, that after the goods in question had been brought out of the customs area the Bureau of Customs had lost jurisdiction over the same, nevertheless, when said goods were intercepted at the Agrifina Circle on November 4, 1966 by members of the Manila Police Department, acting under directions and orders of their Chief, Ricardo C. Papa, who had been formally deputized by the Commissioner of Customs, 9 the Bureau of Customs had regained jurisdiction and custody of the goods. Section 1206 of the Tariff and Customs Code imposes upon the Collector of Customs the duty to hold possession of all imported articles upon which duties, taxes, and other charges have not been paid or secured to be paid, and to dispose of the same according to law. The goods in question, therefore, were under the custody and at the disposal of the Bureau of Customs at the time the petition for mandamus, docketed as Civil

Case No. 67496, was filed in the Court of First Instance of Manila on November 9, 1966. The Court of First Instance of Manila, therefore, could not exercise jurisdiction over said goods even if the warrant of seizure and detention of the goods for the purposes of the seizure and forfeiture proceedings had not yet been issued by the Collector of Customs. The ruling in the case of "Alberto de Joya, et al. v. Hon. Gregorio Lantin, et al.," G.R. No. L-24037, decided by this Court on April 27, 1967, is squarely applicable to the instant case. In the De Joya case, it appears that Francindy Commercial of Manila bought from Ernerose Commercial of Cebu City 90 bales of assorted textiles and rags, valued at P117,731.00, which had been imported and entered thru the port of Cebu. Ernerose Commercial shipped the goods to Manila on board an inter-island vessel. When the goods where about to leave the customs premises in Manila, on October 6, 1964, the customs authorities held them for further verification, and upon examination the goods were found to be different from the declaration in the cargo manifest of the carrying vessel. Francindy Commercial subsequently demanded from the customs authorities the release of the goods, asserting that it is a purchaser in good faith of those goods; that a local purchaser was involved so the Bureau of Customs had no right to examine the goods; and that the goods came from a coastwise port. On October 26, 1964, Francindy Commercial filed in the Court of First Instance of Manila a petition for mandamus against the Commissioner of Customs and the Collector of Customs of the port of Manila to compel said customs authorities to release the goods. Francindy Commercial alleged in its petition for mandamus that the Bureau of Customs had no jurisdiction over the goods because the same were not imported to the port of Manila; that it was not liable for duties and taxes because the transaction was not an original importation; that the goods were not in the hands of the importer nor subject to importer's control, nor were the goods imported contrary to law with its (Francindy Commercial's) knowledge; and that the importation had been terminated. On November 12, 1964, the Collector of Customs of Manila issued a warrant of seizure and identification against the goods. On December 3, 1964, the Commissioner of Customs and the Collector of Customs, as

respondents in the mandamus case, filed a motion to dismiss the petition on the grounds of lack of jurisdiction, lack of cause of action, and in view of the pending seizure and forfeiture proceedings. The Court of First Instance held resolution on the motion to dismiss in abeyance pending decision on the merits. On December 14, 1964, the Court of First Instance of Manila issued a preventive and mandatory injunction, on prayer by Francindy Commercial, upon a bond of P20,000.00. The Commissioner of Customs and the Collector of Customs sought the lifting of the preliminary and mandatory injunction, and the resolution of their motion to dismiss. The Court of First Instance of Manila, however, on January 12, 1965, ordered them to comply with the preliminary and mandatory injunction, upon the filing by Francindy Commercial of an additional bond of P50,000.00. Said customs authorities thereupon filed with this Court, on January 14, 1965, a petition for certiorari and prohibition with preliminary injunction. In resolving the question raised in that case, this Court held: This petition raises two related issues: first, has the Customs bureau jurisdiction to seize the goods and institute forfeiture proceedings against them? and (2) has the Court of First Instance jurisdiction to entertain the petition for mandamus to compel the Customs authorities to release the goods? Francindy Commercial contends that since the petition in the Court of first Instance was filed (on October 26, 1964) ahead of the issuance of the Customs warrant of seizure and forfeiture (on November 12, 1964),the Customs bureau should yield the jurisdiction of the said court. The record shows, however, that the goods in question were actually seized on October 6, 1964, i.e., before Francindy Commercial sued in court. The purpose of the seizure by the Customs bureau was to verify whether or not Custom duties and taxes were paid for their importation. Hence, on December 23, 1964, Customs released 22 bales thereof, for the same were found to have been released regularly from the Cebu Port (Petition Annex "L"). As to goods imported illegally or released irregularly from Customs custody, these are subject to seizure under Section 2530 m. of the Tariff and Customs Code (RA 1957).

The Bureau of Customs has jurisdiction and power, among others to collect revenues from imported articles, fines and penalties and suppress smuggling and other frauds on customs; and to enforce tariff and customs laws (Sec. 602, Republic Act 1957). The goods in question are imported articles entered at the Port of Cebu. Should they be found to have been released irregularly from Customs custody in Cebu City, they are subject to seizure and forfeiture, the proceedings for which comes within the jurisdiction of the Bureau of Customs pursuant to Republic Act 1937. Said proceeding should be followed; the owner of the goods may set up defenses therein (Pacis v. Averia, L-22526, Nov. 20, 1966.) From the decision of the Commissioner of Customs appeal lies to the Court of Tax Appeals, as provided in Sec. 2402 of Republic Act 1937 and Sec. 11 of Republic Act, 1125. To permit recourse to the Court of First Instance in cases of seizure of imported goods would in effect render ineffective the power of the Customs authorities under the Tariff and Customs Code and deprive the Court of Tax Appeals of one of its exclusive appellate jurisdictions. As this Court has ruled in Pacis v. Averia, supra, Republic Acts 1937 and 1125 vest jurisdiction over seizure and forfeiture proceedings exclusively upon the Bureau of Customs and the Court of Tax Appeals. Such law being special in nature, while the Judiciary Act defining the jurisdiction of Courts of First Instance is a general legislation, not to mention that the former are later enactments, the Court of First Instance should yield to the jurisdiction of the Customs authorities. It is the settled rule, therefore, that the Bureau of Customs acquires exclusive jurisdiction over imported goods, for the purposes of enforcement of the customs laws, from the moment the goods are actually in its possession or control, even if no warrant of seizure or detention had previously been issued by the Collector of Customs in connection with seizure and forfeiture proceedings. In the present case, the Bureau of Customs actually seized the goods in question on November 4, 1966, and so from that date the Bureau of Customs acquired jurisdiction over the goods for the purposes of the enforcement of the tariff and customs laws, to the exclusion of the regular courts. Much less then would the Court of First Instance of

Manila have jurisdiction over the goods in question after the Collector of Customs had issued the warrant of seizure and detention on January 12, 1967. 10 And so, it cannot be said, as respondents contend, that the issuance of said warrant was only an attempt to divest the respondent Judge of jurisdiction over the subject matter of the case. The court presided by respondent Judge did not acquire jurisdiction over the goods in question when the petition for mandamus was filed before it, and so there was no need of divesting it of jurisdiction. Not having acquired jurisdiction over the goods, it follows that the Court of First Instance of Manila had no jurisdiction to issue the questioned order of March 7, 1967 releasing said goods. Respondents also aver that petitioner Martin Alagao, an officer of the Manila Police Department, could not seize the goods in question without a search warrant. This contention cannot be sustained. The Chief of the Manila Police Department, Ricardo G. Papa, having been deputized in writing by the Commissioner of Customs, could, for the purposes of the enforcement of the customs and tariff laws, effect searches, seizures, and arrests, 11 and it was his duty to make seizure, among others, of any cargo, articles or other movable property when the same may be subject to forfeiture or liable for any fine imposed under customs and tariff laws. 12 He could lawfully open and examine any box, trunk, envelope or other container wherever found when he had reasonable cause to suspect the presence therein of dutiable articles introduced into the Philippines contrary to law; and likewise to stop, search and examine any vehicle, beast or person reasonably suspected of holding or conveying such article as aforesaid. 13 It cannot be doubted, therefore, that petitioner Ricardo G. Papa, Chief of Police of Manila, could lawfully effect the search and seizure of the goods in question. The Tariff and Customs Code authorizes him to demand assistance of any police officer to effect said search and seizure, and the latter has the legal duty to render said assistance. 14 This was what happened precisely in the case of Lt. Martin Alagao who, with his unit, made the search and seizure of the two trucks loaded with the nine bales of goods in question at the Agrifina Circle. He was given authority by the Chief of Police to make the interception of the cargo.
15

Petitioner Martin Alagao and his companion policemen had

authority to effect the seizure without any search warrant issued by a competent court. The Tariff and Customs Code does not require said warrant in the instant case. The Code authorizes persons having police authority under Section 2203 of the Tariff and Customs Code to enter, pass through or search any land, inclosure, warehouse, store or building, not being a dwelling house; and also to inspect, search and examine any vessel or aircraft and any trunk, package, or envelope or any person on board, or to stop and search and examine any vehicle, beast or person suspected of holding or conveying any dutiable or prohibited article introduced into the Philippines contrary to law, without mentioning the need of a search warrant in said cases. 16 But in the search of a dwelling house, the Code provides that said "dwelling house may be entered and searched only upon warrant issued by a judge or justice of the peace. . . ." 17 It is our considered view, therefor, that except in the case of the search of a dwelling house, persons exercising police authority under the customs law may effect search and seizure without a search warrant in the enforcement of customs laws. Our conclusion finds support in the case of Carroll v. United States, 39 A.L.R., 790, 799, wherein the court, considering a legal provision similar to Section 2211 of the Philippine Tariff and Customs Code, said as follows: Thus contemporaneously with the adoption of the 4th Amendment, we find in the first Congress, and in the following second and fourth Congresses, a difference made as to the necessity for a search warrant between goods subject to forfeiture, when concealed in a dwelling house of similar place, and like goods in course of transportation and concealed in a movable vessel, where readily they could be put out of reach of a search warrant. . . . Again, by the 2d section of the Act of March 3, 1815 (3 Stat. at L.231, 232, chap. 94), it was made lawful for customs officers not only to board and search vessels within their own and adjoining districts, but also to stop, search and examine any vehicle, beast or person on which or whom they should suspect there was merchandise which was subject to duty, or had been introduced into the United States in any manner contrary to law, whether by the person in charge of the vehicle or beast or otherwise, and if they

should find any goods, wares, or merchandise thereon, which they had probably cause to believe had been so unlawfully brought into the country, to seize and secure the same, and the vehicle or beast as well, for trial and forfeiture. This Act was renewed April 27, 1816 (3 Sta. at L. 315, chap. 100), for a year and expired. The Act of February 28, 1865, revived 2 of the Act of 1815, above described, chap. 67, 13 Stat. at L. 441. The substance of this section was reenacted in the 3d section of the Act of July 18, 1866, chap. 201, 14 Stat. at L. 178, and was thereafter embodied in the Revised Statutes as 3061, Comp. Stat. 5763, 2 Fed. Stat. Anno. 2d ed. p. 1161. Neither 3061 nor any of its earlier counterparts has ever been attacked as unconstitutional. Indeed, that section was referred to and treated as operative by this court in Von Cotzhausen v. Nazro, 107 U.S. 215, 219, 27 L. ed. 540, 541, 2 Sup. Ct. Rep. 503. . . . In the instant case, we note that petitioner Martin Alagao and his companion policemen did not have to make any search before they seized the two trucks and their cargo. In their original petition, and amended petition, in the court below Remedios Mago and Valentin Lanopa did not even allege that there was a search. 18 All that they complained of was, That while the trucks were on their way, they were intercepted without any search warrant near the Agrifina Circle and taken to the Manila Police Department, where they were detained. But even if there was a search, there is still authority to the effect that no search warrant would be needed under the circumstances obtaining in the instant case. Thus, it has been held that: The guaranty of freedom from unreasonable searches and seizures is construed as recognizing a necessary difference between a search of a dwelling house or other structure in respect of which a search warrant may readily be obtained and a search of a ship, motorboat, wagon, or automobile for contraband goods, where it is not practicable to secure a warrant because the vehicle can be quickly moved out of the locality or jurisdiction in which the warrant must be sought. (47 Am. Jur., pp. 513-514, citing Carroll v. United States, 267 U.S. 132, 69 L. ed., 543, 45 S. Ct., 280, 39 A.L.R., 790;

People v. Case, 320 Mich., 379, 190 N.W., 389, 27 A.L.R., 686.) In the case of People v. Case (320 Mich., 379, 190 N.W., 389, 27 A.L.R., 686), the question raised by defendant's counsel was whether an automobile truck or an automobile could be searched without search warrant or other process and the goods therein seized used afterwards as evidence in a trial for violation of the prohibition laws of the State. Same counsel contended the negative, urging the constitutional provision forbidding unreasonable searches and seizures. The Court said: . . . Neither our state nor the Federal Constitution directly prohibits search and seizure without a warrant, as is sometimes asserted. Only "unreasonable" search and seizure is forbidden. . . . . . . The question whether a seizure or a search is unreasonable in the language of the Constitution is a judicial and not a legislative question; but in determining whether a seizure is or is not unreasonable, all of the circumstances under which it is made must be looked to. The automobile is a swift and powerful vehicle of recent development, which has multiplied by quantity production and taken possession of our highways in battalions until the slower, animaldrawn vehicles, with their easily noted individuality, are rare. Constructed as covered vehicles to standard form in immense quantities, and with a capacity for speed rivaling express trains, they furnish for successful commission of crime a disguising means of silent approach and swift escape unknown in the history of the world before their advent. The question of their police control and reasonable search on highways or other public places is a serious question far deeper and broader than their use in so-called "bootleging" or "rum running," which is itself is no small matter. While a possession in the sense of private ownership, they are but a vehicle constructed for travel and transportation on highways. Their active use is not in homes or on private premises, the privacy of which the law especially guards from search and seizure without process. The baffling extent to which they are successfully utilized to facilitate commission of crime of all degrees, from those against morality, chastity, and decency, to robbery, rape, burglary, and murder, is a

matter of common knowledge. Upon that problem a condition, and not a theory, confronts proper administration of our criminal laws. Whether search of and seizure from an automobile upon a highway or other public place without a search warrant is unreasonable is in its final analysis to be determined as a judicial question in view of all the circumstances under which it is made. Having declared that the seizure by the members of the Manila Police Department of the goods in question was in accordance with law and by that seizure the Bureau of Customs had acquired jurisdiction over the goods for the purpose of the enforcement of the customs and tariff laws, to the exclusion of the Court of First Instance of Manila, We have thus resolved the principal and decisive issue in the present case. We do not consider it necessary, for the purposes of this decision, to discuss the incidental issues raised by the parties in their pleadings. WHEREFORE, judgment is hereby rendered, as follows: (a) Granting the writ of certiorari and prohibition prayed for by petitioners; (b) Declaring null and void, for having been issued without jurisdiction, the order of respondent Judge Hilarion U. Jarencio, dated March 7, 1967, in Civil Code No. 67496 of the Court of First Instance of Manila; (c) Declaring permanent the preliminary injunction issued by this Court on March 31, 1967 restraining respondent Judge from executing, enforcing and/or implementing his order of March 7, 1967 in Civil Case No. 67496 of the Court of First Instance of Manila, and from proceeding in any manner in said case; (d) Ordering the dismissal of Civil Case No. 67496 of the Court of First Instance of Manila; and
1wph1.t

(e) Ordering the private respondent, Remedios Mago, to pay the costs. It is so ordered.

G.R. No. 96177 January 27, 1993 PEOPLE OF THE PHILIPPINES, Plaintiff-Appellee, vs. MARI MUSA y HANTATALU, Accused-Appellant. ROMERO, J.: The appellant, Mari Musa, seeks, in this appeal, the reversal of the decision, dated August 31, 1990, 1 of the Regional Trial Court (RTC) of Zamboanga City, Branch XII, finding him guilty of selling marijuana in violation of Article II, Section 4 of Republic Act No. 6425, as amended, otherwise known as the Dangerous Drugs Act of 1972. The information filed on December 15, 1989 against the appellant reads: That on or about December 14, 1989, in the City of Zamboanga, Philippines, and within the jurisdiction of this Honorable Court, the above-named accused, not being authorized by law, did then and there, wilfully, unlawfully and feloniously sell to one SGT. AMADO ANI, two (2) wrappers containing dried marijuana leaves, knowing the same to be a prohibited drug.
cha nro blesvi rtua lawlib rary chan roble s virtual law l ibrary

CONTRARY TO LAW. 2

chan roble s virtual law l ibra ry

Upon his arraignment on January 11, 1990, the appellant pleaded not guilty. 3
chan roble s virtual law lib rary

At the trial, the prosecution presented three (3) witnesses, namely: (1) Sgt. Amado Ani, Jr. of the 9th Narcotics Command (NARCOM) of Zamboanga City, who acted as poseur-buyer in the buy-bust operation made against the appellant; (2) T/Sgt. Jesus Belarga, also of the 9th Narcotics Command of Zamboanga City, who was the NARCOM team leader of the buy-bust operation; and (3) Athena Elisa P. Anderson, the Document Examiner and Forensic Chemist of PC-INP Crime Laboratory of Regional Command (RECOM) 9. The evidence of the prosecution was summarized by the trial court as follows: Prosecution evidence shows that in the morning of December 13, 1989, T/Sgt. Jesus Belarga, leader of a NARCOTICS COMMAND (NARCOM) team based at Calarian, Zamboanga City, instructed Sgt. Amado Ani to conduct surveillance and test buy on a certain Mari Musa of Suterville, Zamboanga City. Information received from civilian informer was that this Mari Musa was engaged in selling marijuana in said place. So Sgt. Amado Ani, another NARCOM agent, proceeded to Suterville, in company with a NARCOM civilian informer, to the house

of Mari Musa to which house the civilian informer had guided him. The same civilian informer had also described to him the appearance of Mari Musa. Amado Ani was able to buy one newspaper-wrapped dried marijuana (Exh. "E") for P10.00. Sgt. Ani returned to the NARCOM office and turned over the newspaper-wrapped marijuana to T/Sgt. Jesus Belarga. Sgt. Belarga inspected the stuff turned over to him and found it to be marijuana.
c han roblesv irt ualawli bra ry chan rob les vi rtual law lib rary

The next day, December 14, 1989, about 1:30 P.M., a buy-bust was planned. Sgt. Amado Ani was assigned as the poseur buyer for which purpose he was given P20.00 (with SN GA955883) by Belarga. The buy-bust money had been taken by T/Sgt. Jesus Belarga from M/Sgt. Noh Sali Mihasun, Chief of Investigation Section, and for which Belarga signed a receipt (Exh. "L" & "L-l" ) The team under Sgt. Foncargas was assigned as back-up security. A pre-arranged signal was arranged consisting of Sgt. Ani's raising his right hand, after he had succeeded to buy the marijuana. The two NARCOM teams proceeded to the target site in two civilian vehicles. Belarga's team was composed of Sgt. Belarga, team leader, Sgt. Amado Ani, poseur buyer, Sgt. Lego and Sgt. Biong.
li bra ry

Arriving at the target site, Sgt. Ani proceeded to the house of Mari Musa, while the rest of the NARCOM group positioned themselves at strategic places about 90 to 100 meters from Mari Musa's house. T/Sgt. Belarga could see what went on between Ani and suspect Mari Musa from where he was. Ani approached Mari Musa, who came out of his house, and asked Ani what he wanted. Ani said he wanted some more stuff. Ani gave Mari Musa the P20.00 marked money. After receiving the money, Mari Musa went back to his house and came back and gave Amado Ani two newspaper wrappers containing dried marijuana. Ani opened the two wrappers and inspected the contents. Convinced that the contents were marijuana, Ani walked back towards his companions and raised his right hand. The two NARCOM teams, riding the two civilian vehicles, sped towards Sgt. Ani. Ani joined Belarga's team and returned to the house. At the time Sgt. Ani first approached Mari Musa, there were four persons inside his house: Mari Musa, another boy, and two women, one of whom Ani and Belarga later came to know to be Mari Musa's wife. The second time, Ani with the NARCOM team returned to Mari Musa's house, the woman, who was later known as Mari Musa's wife, slipped away from the house. Sgt. Belarga frisked Mari Musa but could not find the P20.00 marked money with him. Mari Musa was then asked where the P20.00 was and he told the NARCOM team he has

given the money to his wife (who had slipped away). Sgt. Belarga also found a plastic bag containing dried marijuana inside it somewhere in the kitchen. Mari Musa was then placed under arrest and brought to the NARCOM office. At Suterville, Sgt. Ani turned over to Sgt. Belarga the two newspaper-wrapped marijuana he had earlier bought from Mari Musa (Exhs. "C" & "D").
chan rob lesvi rtualaw lib rary c hanrobles vi rt ual law li bra ry

In the NARCOM office, Mari Musa first gave his name as Hussin Musa. Later on, Mari Musa gave his true name - Mari Musa. T/Sgt. Jesus Belarga turned over the two newspaper-wrapped marijuana (bought at the buy-bust), the one newspaper-wrapped marijuana (bought at the test-buy) and the plastic bag containing more marijuana (which had been taken by Sgt. Lego inside the kitchen of Mari Musa) to the PC Crime Laboratory, Zamboanga City, for laboratory examination. The turnover of the marijuana specimen to the PC Crime Laboratory was by way of a letter-request, dated December 14, 1989 (Exh. "B"), which was stamped "RECEIVED" by the PC Crime Laboratory (Exh. "B1") on the same day.
chan rob lesvi rtualaw lib rary c han robles v irt ual law li bra ry

Mrs. Athena Elisa P. Anderson, the Forensic Chemist of the PC Crime Laboratory, examined the marijuana specimens subjecting the same to her three tests. All submitted specimens she examined gave positive results for the presence of marijuana. Mrs. Anderson reported the results of her examination in her Chemistry Report D-100-89, dated December 14, 1989, (Exh. "J", "J-1", "J-2", "J-3", "J-4" and "J-5"). Mrs. Anderson identified in court the two newspaper wrapped marijuana bought at the buy-bust on December 14, 1989, through her initial and the weight of each specimen written with red ink on each wrapper (Exhs. "C-1" and "D-1"). She also identified the one newspaper-wrapped marijuana bought at the test-buy on December 13, 1989, through her markings (Exh. "E-1"). Mrs. Anderson also identified her Chemistry Report (Exh. "J" & sub-markings.)
c han robles v irt ual law lib rary

T. Sgt. Belarga identified the two buy-bust newspaper wrapped marijuana through his initial, the words "buy-bust" and the words "December 14, 1989, 2:45 P.M." (written on Exhs. "C" and "D"). Belarga also identified the receipt of the P20 marked money (with SN GA955883) (Exh. "L"), dated December 14, 1989, and his signature thereon (Exh. "L-1"). He also identified the letter-request, dated December 14, 1989, addressed to the PC Crime Laboratory (Exh. "B") and his signature thereon (Exh. "B-2") and the stamp of the PC Crime Laboratory marked "RECEIVED" (Exh. "B-1"). 4
chan roble s virt ual law l ibra ry

For the defense, the following testified as witnesses: (1) the accusedappellant Mari H. Musa; and (2) Ahara R. Musa, his wife. The trial

court summarized the version of the defense, thus: [O]n December 14, 1989, at about 1:30 in the afternoon, Mari Musa was in his house at Suterville, Zamboanga City. With him were his wife, Ahara Musa, known as Ara, his one-year old child, a woman manicurist, and a male cousin named Abdul Musa. About 1:30 that afternoon, while he was being manicured at one hand, his wife was inside the one room of their house, putting their child to sleep. Three NARCOM agents, who introduced themselves as NARCOM agents, dressed in civilian clothes, got inside Mari Musa's house whose door was open. The NARCOM agents did not ask permission to enter the house but simply announced that they were NARCOM agents. The NARCOM agents searched Mari Musa's house and Mari Musa asked them if they had a search warrant. The NARCOM agents were just silent. The NARCOM agents found a red plastic bag whose contents, Mari Musa said, he did not know. He also did not know if the plastic bag belonged to his brother, Faisal, who was living with him, or his father, who was living in another house about ten arms-length away. Mari Musa, then, was handcuffed and when Mari Musa asked why, the NARCOM agents told him for clarification.
cha nrob lesvi rtua lawlib rary chan robles v irt ual law l ibra ry

Mari Musa was brought in a pick-up, his wife joining him to the NARCOM Office at Calarian, Zamboanga City. Inside the NARCOM Office, Mari Musa was investigated by one NARCOM agent which investigation was reduced into writing. The writing or document was interpreted to Mari Musa in Tagalog. The document stated that the marijuana belonged to Mari Musa and Mari Musa was asked to sign it. But Mari Musa refused to sign because the marijuana did not belong to him. Mari Musa said he was not told that he was entitled to the assistance of counsel, although he himself told the NARCOM agents he wanted to be assisted by counsel.
chan rob lesvi rtualaw lib rary c han robles v irt ual law li bra ry

Mari Musa said four bullets were then placed between the fingers of his right hand and his fingers were pressed which felt very painful. The NARCOM agents boxed him and Mari Musa lost consciousness. While Mari Musa was maltreated, he said his wife was outside the NARCOM building. The very day he was arrested (on cross-examination Mari Musa said it was on the next day), Mari Musa was brought to the Fiscal's Office by three NARCOM agents. The fiscal asked him if the marijuana was owned by him and he said "not." After that single question, Mari Musa was brought to the City Jail. Mari Musa said he did not tell the fiscal that he had been maltreated by the NARCOM agents because he was afraid he might be maltreated in the fiscal's office.
chan roblesv irtualawli bra ry c han robles virtual law libra ry

Mari Musa denied the NARCOM agents' charge that he had sold two

wrappers of marijuana to them; that he had received from them a P20.00 bill which he had given to his wife. He did not sell marijuana because he was afraid that was against the law and that the person selling marijuana was caught by the authorities; and he had a wife and a very small child to support. Mari Musa said he had not been arrested for selling marijuana before. 5
chan rob les vi rtual law lib rary

After trial, the trial court rendered the assailed decision with the following disposition: WHEREFORE, finding accused Mari Musa y Hantatalu guilty beyond reasonable doubt of selling marijuana and pursuant to Sec. 4, Art II of Rep. Act No. 6425, he is sentenced to life imprisonment and to pay the fine of P20,000.00, the latter imposed without subsidiary imprisonment. 6
c han robles v irt ual law l ibra ry

In this appeal, the appellant contends that his guilt was not proved beyond reasonable doubt and impugns the credibility of the prosecution witnesses.
chan roblesv irt ualawli bra ry chan rob les vi rtual law lib rary

The appellant claims that the testimony of Sgt. Ani, the poseur-buyer, is not credible because: (1) prior to the buy-bust operation, neither Sgt. Ani nor the other NARCOM agents were personally known by the appellant or vice-versa; and (2) there was no witness to the alleged giving of the two wrappers of marijuana by the appellant to Sgt. Ani.
chan roble s virtual law libra ry

Sgt. Ani testified that on December 13, 1989, upon instruction by T/Sgt. Jesus Belarga, he conducted a test-buy operation on the appellant whereby he bought one wrapper of marijuana for P15.00 from the latter. 7 He reported the successful operation to T/Sgt. Belarga on the same day. 8 Whereupon, T/Sgt. Belarga conducted a conference to organize a buy-bust operation for the following day. 9
cha nrob les vi rtua l law lib rary

On December 14, 1989, at 1:30 p.m., two NARCOM teams in separate vehicles headed by T/Sgt. Belarga and a certain Sgt. Foncardas went to the place of operation, which was the appellant's house located in Laquian Compound, Suterville, Zamboanga City. Sgt. Ani was with the team of T/Sgt. Belarga, whose other members were Sgts. Lego and Biong. 10 Sgt. Ani was given a marked P20.00 bill by T/Sgt. Belarga, which was to be used in the operation.
chan roble svirtualawl i brary chan robles v irt ual law l ibra ry

Upon reaching the place, the NARCOM agents positioned themselves at strategic places. 11 Sgt. Ani approached the house. Outside the house, the appellant asked Sgt. Ani what he wanted. Sgt. Ani asked him for some more marijuana. 12 Sgt. Ani gave him the marked P20.00 bill and

the appellant went inside the house and brought back two paper wrappers containing marijuana which he handed to Sgt. Ani. 13 From his position, Sgt. Ani could see that there were other people in the house. 14
cha nro bles vi rtua l law lib ra ry

After the exchange, Sgt. Ani approached the other NARCOM agents and made the pre-arranged signal of raising his right hand. 15 The NARCOM agents, accompanied by Sgt. Ani, went inside the house and made the arrest. The agents searched the appellant and unable to find the marked money, they asked him where it was. The appellant said that he gave it to his wife. 16
c han robles v irt ual law l ibra ry

The Court, after a careful reading of the record, finds the testimony of Sgt. Ani regarding the buy-bust operation, which resulted in the apprehension, prosecution and subsequent conviction of the appellant, to be direct, lucid and forthright. Being totally untainted by contradictions in any of the material points, it deserves credence.
chan roble svirtualawl ibra ry cha nrob les vi rtua l law lib rary

The contention that the appellant could not have transacted with Sgt. Ani because they do not know each other is without merit. The day before the buy-bust operation, Sgt. Ani conducted a test-buy and he successfully bought a wrapper of marijuana from the appellant. Through this previous transaction, Sgt. Ani was able to gain the appellant's confidence for the latter to sell more marijuana to Sgt. Ani the following day, during the buy-bust operation. Moreover, the Court has held that what matters is not an existing familiarity between the buyer and the seller, for quite often, the parties to the transaction may be strangers, but their agreement and the acts constituting the sale and delivery of the marijuana. 17
chan roble s virtual law lib rary

The appellant, again to cast doubt on the credibility of Sgt. Ani, argues that it was impossible for the appellant to sell marijuana while his wife, cousin and manicurist were present. But the place of the commission of the crime of selling prohibited drugs has been held to be not crucial 18 and the presence of other people apart from the buyer and seller will not necessarily prevent the consummation of the illegal sale. As the Court observed in People v. Paco, 19 these factors may sometimes camouflage the commission of the crime. In the instant case, the fact that the other people inside the appellant's house are known to the appellant may have given him some assurance that these people will not report him to the authorities.
chan roble svi rtualaw lib rary c hanrobles vi rt ual law li bra ry

The appellant, besides assailing Sgt. Ani's credibility, also questions the credibility of T/Sgt. Belarga. The appellant submits that since T/Sgt. Belarga admitted that he was about 90 meters away from Sgt.

Ani and the appellant, he could not have possibly witnessed the sale. The appellant invokes People v. Ale 20 where the Court observed that from a distance of 10-15 meters, a policeman cannot distinguish between marijuana cigarette from ordinary ones by the type of rolling done on the cigarette sticks. And since T/Sgt. Belarga allegedly did not see the sale, the appellant contends that the uncorroborated testimony of Sgt. Ani can not stand as basis for his conviction. People v. Ale does not apply here because the policeman in that case testified that he and his companion were certain that the appellant therein handed marijuana cigarettes to the poseur-buyer based on the appearance of the cigarette sticks. The Court rejected this claim, stating that: This Court cannot give full credit to the testimonies of the prosecution witnesses marked as they are with contradictions and tainted with inaccuracies. Bian testified that they were able to tell that the four cigarettes were marijuana cigarettes because according to him, the rolling of ordinary cigarettes are different from those of marijuana cigarettes. (tsn, November 13, 1984, p. 10). It is however, incredible to believe that they could discern the type of rolling done on those cigarettes from the distance where they were observing the alleged sale of more or less 10 to 15 meters. 21 In the case at bar, however, T/Sgt. Belarga did not positively claim that he saw the appellant hand over marijuana to Sgt. Ani. What he said was that there was an exchange of certain articles between the two. The relevant portion of T/Sgt. Belarga's testimony reads: 22 Q Now, do you remember whether Sgt. Ani was able to reach the house of Mari Musa?
chanroble s virtual law lib rary

A Yes, ma'am.

chan roblesv irtualawl ibra ry cha nrob les vi rtua l law lib rary

Q After reaching Mari Musa, did you see what happened (sic)? A Yes, ma'am.
chan roblesv irtualawl ibra ry cha nrob les vi rtua l law lib rary

chanro bles vi rtua l law lib ra ry

Q Could you please tell us?

c hanro bles vi rtua l law lib ra ry

A From our vehicle the stainless owner type jeep where Sgt. Lego, Sgt. Biong were boarded, I saw that Sgt. Ani proceeded to the house near the road and he was met by one person and later known as Mari Musa

who was at the time wearing short pants and later on I saw that Sgt. Ani handed something to him, thereafter received by Mari Musa and went inside the house and came back later and handed something to Sgt. Ani. Contrary to the contention of the appellant, it was not impossible for T/Sgt. Belarga to have seen, from a distance of 90-100 meters, Sgt. Ani hand to the appellant "something" and for the latter to give to the former "something."
chanro bles vi rtua l law lib ra ry

Notwithstanding the fact that T/Sgt. Belarga could not have been certain that what Sgt. Ani received from the appellant was marijuana because of the distance, his testimony, nevertheless, corroborated the direct evidence, which the Court earlier ruled to be convincing, presented by Sgt. Ani on the following material points: (1) T/Sgt. Belarga instructed Sgt. Ani to conduct a surveillance and test-buy operation on the appellant at Suterville, Zamboanga City on December 13, 1989; 23 (2) later that same day, Sgt. Ani went back to their office and reported a successful operation and turned over to T/Sgt. Belarga one wrapper of marijuana; 24 (3) T/Sgt. Belarga then organized a team to conduct a buy-bust operation the following day; 25 (4) on December 14, 1989, T/Sgt. Belarga led a team of NARCOM agents who went to Suterville, Zamboanga City; 26 (5) T/Sgt. Belarga gave a P20.00 marked bill to Sgt. Ani which was to be used in the buy-bust operation; 27 (6) upon the arrival of the NARCOM agents in Suterville, Zamboanga City, Sgt. Ani proceeded to the house of the appellant while some agents stayed in the vehicles and others positioned themselves in strategic places; 28 the appellant met Sgt. Ani and an exchange of articles took place. 29
cha nrob les vi rtua l law lib rary

The corroborative testimony of T/Sgt. Belarga strengthens the direct evidence given by Sgt. Ani. Additionally, the Court has ruled that the fact that the police officers who accompanied the poseur-buyer were unable to see exactly what the appellant gave the poseur-buyer because of their distance or position will not be fatal to the prosecution's case 30 provided there exists other evidence, direct or circumstantial, e.g., the testimony of the poseur-buyer, which is sufficient to prove the consummation of the sale of the prohibited drug
chanrobles vi rt ual law li bra ry

The appellant next assails the seizure and admission as evidence of a plastic bag containing marijuana which the NARCOM agents found in the appellant's kitchen. It appears that after Sgt. Ani gave the prearranged signal to the other NARCOM agents, the latter moved in and arrested the appellant inside the house. They searched him to retrieve the marked money but didn't find it. Upon being questioned, the

appellant said that he gave the marked money to his wife. 31 Thereafter, T/Sgt. Belarga and Sgt. Lego went to the kitchen and noticed what T/Sgt. Belarga described as a "cellophane colored white and stripe hanging at the corner of the kitchen." 32 They asked the appellant about its contents but failing to get a response, they opened it and found dried marijuana leaves. At the trial, the appellant questioned the admissibility of the plastic bag and the marijuana it contains but the trial court issued an Order ruling that these are admissible in evidence. 33
chan rob les vi rtual law lib rary

Built into the Constitution are guarantees on the freedom of every individual against unreasonable searches and seizures by providing in Article III, Section 2, the following: The right of the people to be secure in their persons, houses, papers, and effects against unreasonable searches and seizures of whatever nature and for any purpose shall be inviolable, and no search warrant or warrant of arrest shall issue except upon probable cause to be determined personally by the judge after examination under oath or affirmation of the complainant and the witness he may produce, and particularly describing the place to be searched and the persons or things to be seized. Furthermore, the Constitution, in conformity with the doctrine laid down in Stonehill v. Diokno, 34 declares inadmissible, any evidence obtained in violation of the freedom from unreasonable searches and seizures. 35
chan robles v irt ual law l ibra ry

While a valid search warrant is generally necessary before a search and seizure may be effected, exceptions to this rule are recognized. Thus, in Alvero v. Dizon, 36 the Court stated that. "[t]he most important exception to the necessity for a search warrant is the right of search and seizure as an incident to a lawful arrest." 37
cha nro bles vi rtua l law lib ra ry

Rule 126, Section 12 of the Rules of Court expressly authorizes a warrantless search and seizure incident to a lawful arrest, thus: Sec. 12. Search incident to lawful arrest. - A person lawfully arrested may be searched for dangerous weapons or anything which may be used as proof of the commission of an offense, without a search warrant. There is no doubt that the warrantless search incidental to a lawful arrest authorizes the arresting officer to make a search upon the person of the person arrested. As early as 1909, the Court has ruled

that "[a]n officer making an arrest may take from the person arrested any money or property found upon his person which was used in the commission of the crime or was the fruit of the crime or which might furnish the prisoner with the means of committing violence or of escaping, or which may be used as evidence in the trial of the cause . . . " 38 Hence, in a buy-bust operation conducted to entrap a drugpusher, the law enforcement agents may seize the marked money found on the person of the pusher immediately after the arrest even without arrest and search warrants. 39
cha nro bles vi rtua l law lib ra ry

In the case at bar, the NARCOM agents searched the person of the appellant after arresting him in his house but found nothing. They then searched the entire house and, in the kitchen, found and seized a plastic bag hanging in a corner.
chan rob lesvi rtualaw lib rary c hanrobles vi rt ual law li bra ry

The warrantless search and seizure, as an incident to a suspect's lawful arrest, may extend beyond the person of the one arrested to include the premises or surroundings under his immediate control. 40 Objects in the "plain view" of an officer who has the right to be in the position to have that view are subject to seizure and may be presented as evidence. 41
cha nrob les vi rtual law lib rary

In Ker v. California 42 police officers, without securing a search warrant but having information that the defendant husband was selling marijuana from his apartment, obtained from the building manager a passkey to defendants' apartment, and entered it. There they found the defendant husband in the living room. The defendant wife emerged from the kitchen, and one of the officers, after identifying himself, observed through the open doorway of the kitchen, a small scale atop the kitchen sink, upon which lay a brick-shaped package containing green leafy substance which he recognized as marijuana. The package of marijuana was used as evidence in prosecuting defendants for violation of the Narcotic Law. The admissibility of the package was challenged before the U.S. Supreme Court, which held, after observing that it was not unreasonable for the officer to walk to the doorway of the adjacent kitchen on seeing the defendant wife emerge therefrom, that "the discovery of the brick of marijuana did not constitute a search, since the officer merely saw what was placed before him in full view. 43 The U.S. Supreme Court ruled that the warrantless seizure of the marijuana was legal on the basis of the "plain view" doctrine and upheld the admissibility of the seized drugs as part of the prosecution's evidence. 44
chan roble s virtual law lib rary

The "plain view" doctrine may not, however, be used to launch unbridled searches and indiscriminate seizures nor to extend a general

exploratory search made solely to find evidence of defendant's guilt. The "plain view" doctrine is usually applied where a police officer is not searching for evidence against the accused, but nonetheless inadvertently comes across an incriminating object. 45 Furthermore, the U.S. Supreme Court stated the following limitations on the application of the doctrine:
chan roble s virtual law l ibra ry

What the "plain view" cases have in common is that the police officer in each of them had a prior justification for an intrusion in the course of which he came inadvertently across a piece of evidence incriminating the accused. The doctrine serves to supplement the prior justification - whether it be a warrant for another object, hot pursuit, search incident to lawful arrest, or some other legitimate reason for being present unconnected with a search directed against the accused - and permits the warrantless seizure. Of course, the extension of the original justification is legitimate only where it is immediately apparent to the police that they have evidence before them; the "plain view" doctrine may not be used to extend a general exploratory search from one object to another until something incriminating at last emerges. 46
l ibra ry

It has also been suggested that even if an object is observed in "plain view," the "plain view" doctrine will not justify the seizure of the object where the incriminating nature of the object is not apparent from the "plain view" of the object. 47 Stated differently, it must be immediately apparent to the police that the items that they observe may be evidence of a crime, contraband, or otherwise subject to seizure.
chanro blesvi rt ualawlib ra ry chan roble s virtual law lib rary

In the instant case, the appellant was arrested and his person searched in the living room. Failing to retrieve the marked money which they hoped to find, the NARCOM agents searched the whole house and found the plastic bag in the kitchen. The plastic bag was, therefore, not within their "plain view" when they arrested the appellant as to justify its seizure. The NARCOM agents had to move from one portion of the house to another before they sighted the plastic bag. Unlike Ker vs. California, where the police officer had reason to walk to the doorway of the adjacent kitchen and from which position he saw the marijuana, the NARCOM agents in this case went from room to room with the obvious intention of fishing for more evidence.
chan roble svi rtualawl ib rary c hanro bles vi rtua l law li bra ry

Moreover, when the NARCOM agents saw the plastic bag hanging in one corner of the kitchen, they had no clue as to its contents. They had to ask the appellant what the bag contained. When the appellant refused to respond, they opened it and found the marijuana. Unlike Ker v. California, where the marijuana was visible to the police officer's

eyes, the NARCOM agents in this case could not have discovered the inculpatory nature of the contents of the bag had they not forcibly opened it. Even assuming then, that the NARCOM agents inadvertently came across the plastic bag because it was within their "plain view," what may be said to be the object in their "plain view" was just the plastic bag and not the marijuana. The incriminating nature of the contents of the plastic bag was not immediately apparent from the "plain view" of said object. It cannot be claimed that the plastic bag clearly betrayed its contents, whether by its distinctive configuration, its transprarency, or otherwise, that its contents are obvious to an observer. 48
cha n

We, therefore, hold that under the circumstances of the case, the "plain view" doctrine does not apply and the marijuana contained in the plastic bag was seized illegally and cannot be presented in evidence pursuant to Article III, Section 3(2) of the Constitution.
chan roble svi rtualaw lib rary c hanrobles vi rt ual law li bra ry

The exclusion of this particular evidence does not, however, diminish, in any way, the damaging effect of the other pieces of evidence presented by the prosecution to prove that the appellant sold marijuana, in violation of Article II, Section 4 of the Dangerous Drugs Act of 1972. We hold that by virtue of the testimonies of Sgt. Ani and T/Sgt. Belarga and the two wrappings of marijuana sold by the appellant to Sgt. Ani, among other pieces of evidence, the guilt of the appellant of the crime charged has been proved beyond reasonable doubt.
chan roble svirtualawl ibra ry ch anro bles vi rtua l law lib ra ry

WHEREFORE, the appeal is DISMISSED and the judgment of the Regional Trial Court AFFIRMED.
c han roblesv irt ualawli bra ry chan rob les vi rtual law lib rary

SO ORDERED.

G.R. No. 83988 September 29, 1989 RICARDO C. VALMONTE AND UNION OF LAWYERS AND ADVOCATES FOR PEOPLE'S RIGHTS (ULAP), petitioners, vs. GEN. RENATO DE VILLA AND NATIONAL CAPITAL REGION DISTRICT COMMAND, respondents. Ricardo C. Valmonte for himself and his co-petitioners.

PADILLA, J.: This is a petition for prohibition with preliminary injunction and/or temporary restraining order, seeking the declaration of checkpoints in Valenzuela, Metro Manila or elsewhere, as unconstitutional and the dismantling and banning of the same or, in the alternative, to direct the respondents to formulate guidelines in the implementation of checkpoints, for the protection of the people. Petitioner Ricardo C. Valmonte sues in his capacity as citizen of the Republic, taxpayer, member of the Integrated Bar of the Philippines (IBP), and resident of Valenzuela, Metro Manila; while petitioner Union of Lawyers and Advocates for People's Rights (ULAP) sues in its capacity as an association whose members are all members of the IBP. The factual background of the case is as follows: On 20 January 1987, the National Capital Region District Command (NCRDC) was activated pursuant to Letter of Instruction 02/87 of the Philippine General Headquarters, AFP, with the mission of conducting security operations within its area of responsibility and peripheral areas, for the purpose of establishing an effective territorial defense, maintaining peace and order, and providing an atmosphere conducive to the social, economic and political development of the National Capital Region. 1 As part of its duty to maintain peace and order, the NCRDC installed checkpoints in various parts of Valenzuela, Metro Manila. Petitioners aver that, because of the installation of said checkpoints,

the residents of Valenzuela are worried of being harassed and of their safety being placed at the arbitrary, capricious and whimsical disposition of the military manning the checkpoints, considering that their cars and vehicles are being subjected to regular searches and check-ups, especially at night or at dawn, without the benefit of a search warrant and/or court order. Their alleged fear for their safety increased when, at dawn of 9 July 1988, Benjamin Parpon, a supply officer of the Municipality of Valenzuela, Bulacan, was gunned down allegedly in cold blood by the members of the NCRDC manning the checkpoint along McArthur Highway at Malinta, Valenzuela, for ignoring and/or refusing to submit himself to the checkpoint and for continuing to speed off inspire of warning shots fired in the air. Petitioner Valmonte also claims that, on several occasions, he had gone thru these checkpoints where he was stopped and his car subjected to search/check-up without a court order or search warrant. Petitioners further contend that the said checkpoints give the respondents a blanket authority to make searches and/or seizures without search warrant or court order in violation of the Constitution; 2 and, instances have occurred where a citizen, while not killed, had been harassed. Petitioners' concern for their safety and apprehension at being harassed by the military manning the checkpoints are not sufficient grounds to declare the checkpoints as per se illegal. No proof has been presented before the Court to show that, in the course of their routine checks, the military indeed committed specific violations of petitioners' right against unlawful search and seizure or other rights. In a case filed by the same petitioner organization, Union of Lawyers and Advocates for People's Right (ULAP) vs. Integrated National Police, 3 it was held that individual petitioners who do not allege that any of their rights were violated are not qualified to bring the action, as real parties in interest. The constitutional right against unreasonable searches and seizures is a personal right invocable only by those whose rights have been infringed, 4 or threatened to be infringed. What constitutes a reasonable or unreasonable search and seizure in any particular case is purely a judicial question, determinable from a consideration

of the circumstances involved. 5 Petitioner Valmonte's general allegation to the effect that he had been stopped and searched without a search warrant by the military manning the checkpoints, without more, i.e., without stating the details of the incidents which amount to a violation of his right against unlawful search and seizure, is not sufficient to enable the Court to determine whether there was a violation of Valmonte's right against unlawful search and seizure. Not all searches and seizures are prohibited. Those which are reasonable are not forbidden. A reasonable search is not to be determined by any fixed formula but is to be resolved according to the facts of each case. 6 Where, for example, the officer merely draws aside the curtain of a vacant vehicle which is parked on the public fair grounds, 7 or simply looks into a vehicle, 8 or flashes a light therein, 9 these do not constitute unreasonable search. The setting up of the questioned checkpoints in Valenzuela (and probably in other areas) may be considered as a security measure to enable the NCRDC to pursue its mission of establishing effective territorial defense and maintaining peace and order for the benefit of the public. Checkpoints may also be regarded as measures to thwart plots to destabilize the government, in the interest of public security. In this connection, the Court may take judicial notice of the shift to urban centers and their suburbs of the insurgency movement, so clearly reflected in the increased killings in cities of police and military men by NPA "sparrow units," not to mention the abundance of unlicensed firearms and the alarming rise in lawlessness and violence in such urban centers, not all of which are reported in media, most likely brought about by deteriorating economic conditions which all sum up to what one can rightly consider, at the very least, as abnormal times. Between the inherent right of the state to protect its existence and promote public welfare and an individual's right against a warrantless search which is however reasonably conducted, the former should prevail. True, the manning of checkpoints by the military is susceptible of abuse by the men in uniform, in the same manner that all governmental power is susceptible of abuse. But, at the cost of

occasional inconvenience, discomfort and even irritation to the citizen, the checkpoints during these abnormal times, when conducted within reasonable limits, are part of the price we pay for an orderly society and a peaceful community. Finally, on 17 July 1988, military and police checkpoints in Metro Manila were temporarily lifted and a review and refinement of the rules in the conduct of the police and military manning the checkpoints was ordered by the National Capital Regional Command Chief and the Metropolitan Police Director. 10 WHEREFORE, the petition is DISMISSED. SO ORDERED.

Você também pode gostar